100% found this document useful (4 votes)
2K views70 pages

Rheumatology MCQs for Medical Students

This document contains multiple choice questions about rheumatology. It provides explanations for each answer. The questions cover topics like carpal tunnel syndrome, lumbar disc prolapse, ankylosing spondylitis, McArdle's disease, and elbow pain conditions. The document tests knowledge of clinical features, expected neurological deficits, complications, and underlying pathophysiology.

Uploaded by

M
Copyright
© © All Rights Reserved
We take content rights seriously. If you suspect this is your content, claim it here.
Available Formats
Download as PDF, TXT or read online on Scribd
100% found this document useful (4 votes)
2K views70 pages

Rheumatology MCQs for Medical Students

This document contains multiple choice questions about rheumatology. It provides explanations for each answer. The questions cover topics like carpal tunnel syndrome, lumbar disc prolapse, ankylosing spondylitis, McArdle's disease, and elbow pain conditions. The document tests knowledge of clinical features, expected neurological deficits, complications, and underlying pathophysiology.

Uploaded by

M
Copyright
© © All Rights Reserved
We take content rights seriously. If you suspect this is your content, claim it here.
Available Formats
Download as PDF, TXT or read online on Scribd
You are on page 1/ 70

RHEUMATOLOGY MCQs

Q-1 A. Common peroneal nerve


Which one of the following is not associated with carpal B. Lateral cutaneous nerve of the thigh
tunnel syndrome? C. L5
D. L3
A. Tinel's sign E. L4
B. Compression of the median nerve
C. Wasting of the hypothenar eminence ANSWER:
D. Flexion of the wrist reproduces symptoms L4
E. Weakness of thumb abduction

ANSWER: EXPLANATION:
Wasting of the hypothenar eminence LOWER BACK PAIN: PROLAPSED DISC
A prolapsed lumbar disc usually produces clear dermatomal
EXPLANATION: leg pain associated with neurological deficits.
CARPAL TUNNEL SYNDROME
Carpal tunnel syndrome is caused by compression of median Features
nerve in the carpal tunnel. • leg pain usually worse than back
• pain often worse when sitting
History
• pain/pins and needles in thumb, index, middle finger The table below demonstrates the expected features
• unusually the symptoms may 'ascend' proximally according to the level of compression:
• patient shakes his hand to obtain relief, classically at night Site of compression Features
L3 nerve root Sensory loss over anterior thigh
Examination
compression Weak quadriceps
• weakness of thumb abduction (abductor pollicis brevis) Reduced knee reflex
• wasting of thenar eminence (NOT hypothenar) Positive femoral stretch test
• Tinel's sign: tapping causes paraesthesia L4 nerve root Sensory loss anterior aspect of knee
• Phalen's sign: flexion of wrist causes symptoms compression Weak quadriceps
Reduced knee reflex
Causes Positive femoral stretch test
• idiopathic L5 nerve root Sensory loss dorsum of foot
• pregnancy compression Weakness in foot and big toe dorsiflexion
Reflexes intact
• oedema e.g. heart failure
Positive sciatic nerve stretch test
• lunate fracture
S1 nerve root Sensory loss posterolateral aspect of leg and lateral
• rheumatoid arthritis compression aspect of foot
Weakness in plantar flexion of foot
Electrophysiology Reduced ankle reflex
• motor + sensory: prolongation of the action potential Positive sciatic nerve stretch test

Treatment Management
• corticosteroid injection • similar to that of other musculoskeletal lower back pain:
• wrist splints at night analgesia, physiotherapy, exercises
• surgical decompression (flexor retinaculum division) • if symptoms persist then referral for consideration of MRI
is appropriate
Q-2
A 40-year-old man presents with pain in his lower back and Q-3
'sciatica' for the past three days. He describes bending down Which one of the following is most recognised as a potential
to pick up a washing machine when he felt 'something go'. complication in a patient with ankylosing spondylitis?
He now has severe pain radiating from his back down the A. Heart block
right leg. On examination he describes paraesthesia over the B. Aortic stenosis
anterior aspect of the right knee and the medial aspect of his C. Achalasia
calf. Power is intact and the right knee reflex is diminished. D. Diabetes mellitus
The femoral stretch test is positive on the right side. Which E. Bronchiectasis
nerve root is most likely to be affected?
ANSWER: ANSWER:
Heart block McArdle's syndrome
EXPLANATION:
Ankylosing spondylitis features - the 'A's EXPLANATION:
Repeated cramping and myoglobinuria after short bouts of
• Apical fibrosis
exercise can point towards McArdle's disease
• Anterior uveitis
The time of onset, the 'second wind' phenomenon, and
• Aortic regurgitation
recurrent symptoms are suggestive of McArdle's syndrome.
• Achilles tendonitis
• AV node block MCARDLE'S DISEASE
• Amyloidosis Overview
ANKYLOSING SPONDYLITIS: FEATURES • autosomal recessive type V glycogen storage disease
Ankylosing spondylitis is a HLA-B27 associated • caused by myophosphorylase deficiency
spondyloarthropathy. It typically presents in males (sex ratio • this causes decreased muscle glycogenolysis
3:1) aged 20-30 years old.
Features
Features • muscle pain and stiffness following exercise
• typically a young man who presents with lower back pain • muscle cramps
and stiffness of insidious onset • myoglobinuria
• stiffness is usually worse in the morning and improves • low lactate levels during exercise
with exercise
• the patient may experience pain at night which improves Q-5
on getting up A 40-year-old woman complains of a permanent 'funny-
bone' sensation in her right elbow. This is accompanied by
Clinical examination
tingling in the little and ring finger. Her symptoms are worse
• reduced lateral flexion when the elbow is bent for prolonged periods. What is the
• reduced forward flexion - Schober's test - a line is drawn most likely diagnosis?
10 cm above and 5 cm below the back dimples (dimples
of Venus). The distance between the two lines should A. Cubital tunnel syndrome
increase by more than 5 cm when the patient bends as B. Lateral epicondylitis
far forward as possible C. Medial epicondylitis
• reduced chest expansion D. Median nerve entrapment syndrome
E. Radial tunnel syndrome
Other features - the 'A's
• Apical fibrosis
ANSWER:
• Anterior uveitis
Cubital tunnel syndrome
• Aortic regurgitation
• Achilles tendonitis EXPLANATION:
• AV node block ELBOW PAIN
• Amyloidosis The table below details some of the characteristic features of
• and cauda equina syndrome conditions causing elbow pain:
• peripheral arthritis (25%, more common if female)
Features
Q-4
A 26-year-old man presents to his GP complaining of
recurrent cramping at the very start of his weekly five-a-side • pain and tenderness localised to the
football game. He is usually able to continue playing and the lateral epicondyle
cramps always resolve after the first 10 minutes of play. • pain worse on resisted wrist extension
with the elbow extended or supination
After these games, his urine can sometimes have the
of the forearm with the elbow extended
appearance of 'coca-cola'.
• episodes typically last between 6
What is the most likely underlying diagnosis? months and 2 years. Patients tend to
Lateral epicondylitis have acute pain for 6-12 weeks
A. March haematuria (tennis elbow)
B. McArdle's syndrome Medial epicondylitis Features
C. Sickle cell trait (golfer's elbow)
D. G6PD deficiency
E. Hereditary spherocytosis • pain and tenderness localised to the medial
Features EXPLANATION:
REACTIVE ARTHRITIS: FEATURES
Reactive arthritis is one of the HLA-B27 associated
• pain and tenderness localised to the seronegative spondyloarthropathies. It encompasses Reiter's
lateral epicondyle syndrome, a term which described a classic triad of urethritis,
• pain worse on resisted wrist extension conjunctivitis and arthritis following a dysenteric illness during
with the elbow extended or supination the Second World War. Later studies identified patients who
of the forearm with the elbow extended
developed symptoms following a sexually transmitted
• episodes typically last between 6 infection (post-STI, now sometimes referred to as sexually
months and 2 years. Patients tend to
have acute pain for 6-12 weeks
acquired reactive arthritis, SARA).
Lateral epicondylitis
(tennis elbow)
Reactive arthritis is defined as an arthritis that develops
epicondyle
following an infection where the organism cannot be
• pain is aggravated by wrist flexion and
pronation
recovered from the joint.
• symptoms may be accompanied by
numbness / tingling in the 4th and 5th Features
finger due to ulnar nerve involvement typically develops within 4 weeks of initial infection -
symptoms generally last around 4-6 months
Radial tunnel Most commonly due to compression of the posterior arthritis is typically an asymmetrical oligoarthritis of lower
syndrome interosseous branch of the radial nerve. It is thought limbs
to be a result of overuse. dactylitis
symptoms of urethritis
Features eye: conjunctivitis (seen in 10-30%), anterior uveitis
skin: circinate balanitis (painless vesicles on the coronal
margin of the prepuce), keratoderma blenorrhagica (waxy
• symptoms are similar to lateral yellow/brown papules on palms and soles)
epicondylitis making it difficult to diagnose
• however, the pain tends to be around 4-5 Around 25% of patients have recurrent episodes whilst 10% of
cm distal to the lateral epicondyle
patients develop chronic disease
• symptoms may be worsened by extending
the elbow and pronating the forearm
'Can't see, pee or climb a tree'
Cubital tunnel Due to the compression of the ulnar nerve.
syndrome
Features

• initially intermittent tingling in the 4th and


5th finger
• may be worse when the elbow is resting on
a firm surface or flexed for extended
periods
• later numbness in the 4th and 5th finger
with associated weakness

Olecranon bursitis Swelling over the posterior aspect of the elbow. Keratoderma blenorrhagica
There may be associated pain, warmth and erythema.
It typically affects middle-aged male patients.
Q-7
Q-6 A 30-year-old intravenous drug user is diagnosed as having
Each one of the following is seen in reactive arthritis, except: osteomyelitis of the right tibia. What is the most likely
A. Urethritis causative organism?
B. Keratoderma blenorrhagica
C. Conjunctivitis A. Salmonella species
D. Aseptic meningoencephalitis B. Haemophilus influenzae
E. Circinate balanitis C. Staphylococcus aureus
D. Enterobacter species
ANSWER: E. Streptococcus pyogenes
Aseptic meningoencephalitis
ANSWER: Drug causes
Staphylococcus aureus • diuretics: thiazides, furosemide
• ciclosporin
EXPLANATION:
• alcohol
OSTEOMYELITIS
• cytotoxic agents
Osteomyelitis describes an infection of the bone.
• pyrazinamide
• aspirin: it was previously thought that only high-dose
Staph. aureus is the most common cause except in patients
aspirin could precipitate gout. However, a systematic
with sickle-cell anaemia where Salmonella species
review (see link) showed that low-dose (e.g. 75mg) also
predominate.
increases the risk of gout attacks. This obviously needs to
be balanced against the cardiovascular benefits of aspirin
Predisposing conditions
and the study showed patients coprescribed allopurinol
• diabetes mellitus
were not at an increased risk
• sickle cell anaemia
• intravenous drug user
Q-9
• immunosuppression due to either medication or HIV What is the most common target of pANCA?
• alcohol excess
A. Plasminogen activator inhibitor-1
Investigations B. Elastase
• MRI is the imaging modality of choice, with a sensitivity of C. Myeloperoxidase
90-100% D. Serine proteinase 3
E. Cathepsin G
Management
• flucloxacillin for 6 weeks ANSWER:
• clindamycin if penicillin-allergic Myeloperoxidase

Q-8 EXPLANATION:
A 58-year-old man presents with a 3-day history of painful Myeloperoxidase is a neutrophil protein whose primary role
swelling of the 1st metatarsophalangeal joint of his right is the generation of oxygen free radicals.
foot. He has a history of hypertension,
hypercholesterolaemia and osteoarthritis. His list of ANCA
medications includes amlodipine, ibuprofen, omeprazole, There are two main types of anti-neutrophil cytoplasmic
indapamide and atorvastatin. antibodies (ANCA) - cytoplasmic (cANCA) and perinuclear
(pANCA)
Which of his list of medications is most likely to have
precipitated his acute attack of gout? For the exam, remember:
• cANCA - granulomatosis with polyangiitis (Wegener's
A. Amlodipine granulomatosis)
B. Ibuprofen • pANCA - Churg-Strauss syndrome + others (see below)
C. Omeprazole
D. Indapamide cANCA
E. Atorvastatin • most common target serine proteinase 3 (PR3)
ANSWER: • some correlation between cANCA levels and disease
Indapamide activity
• granulomatosis with polyangiitis, positive in > 90%
EXPLANATION: • microscopic polyangiitis, positive in 40%
Thiazide diuretics can precipitate an attack of gout
Thiazide diuretics, such as indapamide, chlortalidone and pANCA
bendroflumethiazide, can precipitate an acute attack of gout • most common target is myeloperoxidase (MPO)
by reducing renal excretion of uric acid. The other drugs are • cannot use level of pANCA to monitor disease activity
less associated. • associated with immune crescentic glomerulonephritis
(positive in c. 80% of patients)
GOUT: DRUG CAUSES
• microscopic polyangiitis, positive in 50-75%
Gout is a form of microcrystal synovitis caused by the
• Churg-Strauss syndrome, positive in 60%
deposition of monosodium urate monohydrate in the
synovium. It is caused by chronic hyperuricaemia (uric acid > • primary sclerosing cholangitis, positive in 60-80%
0.45 mmol/l). • granulomatosis with polyangiitis, positive in 25%
Other causes of positive ANCA (usually pANCA) • adalimumab: monoclonal antibody, subcutaneous
• inflammatory bowel disease (UC > Crohn's) administration
• connective tissue disorders: RA, SLE, Sjogren's • adverse effects of TNF blockers include reactivation of
• autoimmune hepatitis latent tuberculosis and demyelination

Q-10 Infliximab is also used in active Crohn's disease unresponsive


Which one of the following cells secretes the majority of to steroids
tumour necrosis factor in humans?
Q-11
A. Neutrophils Stephen is a 57-year-old man who has late ankylosing
B. Macrophages spondylitis. What pulmonary feature might you see on his
C. Natural killer cells chest radiograph?
D. Killer-T cells
E. Helper-T cells A. Apical fibrosis
B. Bilateral hilar lymphadenopathy
ANSWER: C. Peripheral granulomas
Macrophages D. Basal fibrosis
E. Increased pulmonary vasculature markings
EXPLANATION:
TUMOUR NECROSIS FACTOR ANSWER:
Tumour necrosis factor (TNF) is a pro-inflammatory cytokine Apical fibrosis
with multiple roles in the immune system
EXPLANATION:
TNF is secreted mainly by macrophages and has a number of May be able to see apical fibrosis on chest x-ray in later
effects on the immune system, acting mainly in a paracrine ankylosing spodylitis
fashion: You may be able to see apical fibrosis on chest x-ray in later
• activates macrophages and neutrophils ankylosing spondylitis.
• acts as costimulator for T cell activation
• key mediator of bodies response to Gram negative Peripheral granulomas form after inflammation. Causes
septicaemia include infection, TB and sarcoidosis.
• similar properties to IL-1
• anti-tumour effect (e.g. phospholipase activation) Bilateral hilar lymphadenopathy is often related to
sarcoidosis and possibly lymphoma.
TNF-alpha binds to both the p55 and p75 receptor. These
receptors can induce apoptosis. It also cause activation of Increased pulmonary vasculature markings can be related to
NFkB congestion or pulmonary hypertension.

Endothelial effects include increase expression of selectins ANKYLOSING SPONDYLITIS: INVESTIGATION AND
and increased production of platelet activating factor, IL-1 and MANAGEMENT
prostaglandins Ankylosing spondylitis is a HLA-B27 associated
spondyloarthropathy. It typically presents in males (sex ratio
TNF promotes the proliferation of fibroblasts and their 3:1) aged 20-30 years old.
production of protease and collagenase. It is thought
fragments of receptors act as binding points in serum Investigation
Inflammatory markers (ESR, CRP) are typically raised although
Systemic effects include pyrexia, increased acute phase normal levels do not exclude ankylosing spondylitis.
proteins and disordered metabolism leading to cachexia
HLA-B27 is of little use in making the diagnosis as it is positive
TNF is important in the pathogenesis of rheumatoid arthritis - in:
TNF blockers (e.g. infliximab, etanercept) are now licensed for • 90% of patients with ankylosing spondylitis
treatment of severe rheumatoid • 10% of normal patients

TNF blockers Plain x-ray of the sacroiliac joints is the most useful
• infliximab: monoclonal antibody, IV administration investigation in establishing the diagnosis. Radiographs may
• etanercept: fusion protein that mimics the inhibitory be normal early in disease, later changes include:
effects of naturally occurring soluble TNF receptors, • sacroilitis: subchondral erosions, sclerosis
subcutaneous administration • squaring of lumbar vertebrae
• 'bamboo spine' (late & uncommon)
• syndesmophytes: due to ossification of outer fibers of
annulus fibrosus
• chest x-ray: apical fibrosis

Fusion of bilateral sacroiliac joints. Sacroiliitis may present as sclerosis of joint


margins which can be asymmetrical at early stage of disease, but is bilateral and
symmetrical in late disease

40-year-old male. There is typical appearance of bamboo spine with a single


central radiodense line related to ossification of supraspinous and interspinous
ligaments which is called dagger sign. Ankylosing is detectable in both sacroiliac
joints

Syndesmophytes and squaring of vertebral bodies. Squaring of anterior vertebral


margins is due to osteitis of anterior corners. Syndesmophytes are due to
ossification of outer fibers of annulus fibrosus

Spirometry may show a restrictive defect due to a


combination of pulmonary fibrosis, kyphosis and ankylosis of
the costovertebral joints.

Ankylosing spondylitis with well formed syndesmophytes Management


The following is partly based on the 2010 EULAR guidelines
(please see the link for more details):
• encourage regular exercise such as swimming
• physiotherapy
• NSAIDs are the first-line treatment
• the disease-modifying drugs which are used to treat
rheumatoid arthritis (such as sulphasalazine) are only
really useful if there is peripheral joint involvement
• the 2010 EULAR guidelines suggest: 'Anti-TNF therapy
should be given to patients with persistently high disease
activity despite conventional treatments'
• research is ongoing to see whether anti-TNF therapies
such as etanercept and adalimumab should be used
earlier in the course of the disease

Lateral cervical spine. Complete fusion of anterior and posterior elements in


ankylosing spondylitis, so called bamboo spine
Q-12 Q-13
A 32-year-old woman presents with a facial rash that A 79-year-old man presents with a history of lower back pain
appears to be worse in summer. She has also noticed and right hip pain. Blood tests reveal the following:
intermittent swelling of the small joints of her hands and
describes suffering from Raynaud's syndrome for 5 years. Calcium 2.20 mmol/l
Phosphate 0.8 mmol/l
Given the likely diagnosis which of these auto-antibodies is ALP 890 u/L
most likely to be positive?
What is the most likely diagnosis?
A. Anti-dsDNA
B. Anti-nuclear cytoplasmic antibody A. Primary hyperparathyroidism
C. Anti-nuclear antibody B. Chronic kidney disease
D. Anti-Sm C. Osteomalacia
E. Anti-Ro D. Osteoporosis
E. Paget's disease
ANSWER:
Anti-nuclear antibody ANSWER:
Paget's disease
EXPLANATION:
This lady presents with a photosensitive rash, Raynaud's EXPLANATION:
syndrome and small joint arthritis - giving a likely diagnosis Paget's disease - old man, bone pain, raised ALP
of systemic lupus erythematosus (SLE). The normal calcium and phosphate combined with a raised
alkaline phosphate points to a diagnosis of Paget's
Anti-nuclear (ANA) antibody is the most likely antibody to be
present. Approximately 95% of people with SLE have a PAGET'S DISEASE OF THE BONE
positive ANA, however it is not very specific. Paget's disease is a disease of increased but uncontrolled
bone turnover. It is thought to be primarily a disorder of
Anti-Sm are the most specific antibodies for SLE, but only osteoclasts, with excessive osteoclastic resorption followed by
around 35% of patients with SLE will be positive. Anti-dsDNA increased osteoblastic activity. Paget's disease is common (UK
antibodies are positive in around 70% of SLE cases. prevalence 5%) but symptomatic in only 1 in 20 patients

Anti-Ro antibodies are present in 15% of patients with SLE, Predisposing factors
but are more frequently associated with Sjörgren's • increasing age
syndrome. • male sex
• northern latitude
Anti-nuclear cytoplasmic antibodies are not associated with • family history
SLE. They are associated with inflammatory bowel disease
and vasculitis such as Wegener's granulomatosis. Clinical features - only 5% of patients are symptomatic
• bone pain (e.g. pelvis, lumbar spine, femur)
SYSTEMIC LUPUS ERYTHEMATOSUS: INVESTIGATIONS • classical, untreated features: bowing of tibia, bossing of
Immunology skull
• 99% are ANA positive • raised alkaline phosphatase (ALP) - calcium* and
• 20% are rheumatoid factor positive phosphate are typically normal
• anti-dsDNA: highly specific (> 99%), but less sensitive • skull x-ray: thickened vault, osteoporosis circumscripta
(70%)
• anti-Smith: most specific (> 99%), sensitivity (30%) Indications for treatment include bone pain, skull or long bone
• also: anti-U1 RNP, SS-A (anti-Ro) and SS-B (anti-La) deformity, fracture, periarticular Paget's
• bisphosphonate (either oral risedronate or IV
Monitoring zoledronate)
• ESR: during active disease the CRP is characteristically • calcitonin is less commonly used now
normal - a raised CRP may indicate underlying infection
• complement levels (C3, C4) are low during active disease Complications
(formation of complexes leads to consumption of • deafness (cranial nerve entrapment)
complement) • bone sarcoma (1% if affected for > 10 years)
• anti-dsDNA titres can be used for disease monitoring (but • fractures
note not present in all patients) • skull thickening
• high-output cardiac failure
A. Limited cutaneous systemic sclerosis
B. Mixed connective tissue disease
C. Dermatomyositis
D. Polymyositis
E. Diffuse cutaneous systemic sclerosis

ANSWER:
Polymyositis

EXPLANATION:
Anti-Jo 1 antibodies are more commonly seen in polymyositis
than dermatomyositis

The radiograph demonstrates marked thickening of the calvarium. There are also
EXTRACTABLE NUCLEAR ANTIGENS
ill-defined sclerotic and lucent areas throughout. These features are consistent
with Paget's disease. Overview
• specific nuclear antigens
• usually associated with being ANA positive

Examples
• anti-Ro: Sjogren's syndrome, SLE, congenital heart block
• anti-La: Sjogren's syndrome
• anti-Jo 1: polymyositis
• anti-scl-70: diffuse cutaneous systemic sclerosis
• anti-centromere: limited cutaneous systemic sclerosis

Q-15
Which one of the following drugs is least likely to cause
gout?
Pelvic x-ray from an elderly man with Paget's disease. There is a smooth cortical
expansion of the left hemipelvic bones with diffuse increased bone density and A. Lithium
coarsening of trabeculae.
B. Bendroflumethiazide
C. Alcohol
D. Pyrazinamide
E. Furosemide

NSWER:
Lithium

EXPLANATION:
Lithium was actually used to treat gout in the 19th century

Please see Q-8 for Gout: Drug Causes

Q-16
A 64-year-old female is referred to rheumatology out-
patients by her GP with a history of arthritis in both hands.
Which one of the following x-ray findings would most favour
Isotope bone scan from a patient with Paget's disease showing a typical
a diagnosis of rheumatoid arthritis over other possible
distribution in the spine, asymmetrical pelvic disease and proximal long bones. causes?

*usually normal in this condition but hypercalcaemia may occur with A. Loss of joint space
prolonged immobilisation B. Periarticular osteopenia
C. Subchondral sclerosis
Q-14
D. Osteophytes
An autoantibody screen reveals that a patient is positive for
E. Subchondral cysts
anti-Jo 1 antibodies. What is the most likely underlying
diagnosis?
ANSWER: A. Equivalent of prednisolone 10 mg or more each day for 6
Periarticular osteopenia months
B. Equivalent of prednisolone 7.5 mg or more each day for
EXPLANATION: 6 weeks
Periarticular osteopenia and osteoporosis would point C. Equivalent of prednisolone 5 mg or more each day for 6
towards a diagnosis of rheumatoid arthritis (RA). Loss of weeks
joint space is common in both RA and osteoarthritis D. Equivalent of prednisolone 7.5 mg or more each day for
3 months
RHEUMATOID ARTHRITIS: X-RAY CHANGES E. Equivalent of prednisolone 10 mg or more each day for 6
weeks
Early x-ray findings
• loss of joint space ANSWER:
• juxta-articular osteoporosis Equivalent of prednisolone 7.5 mg or more each day for 3
• soft-tissue swelling months

Late x-ray findings EXPLANATION:


• periarticular erosions OSTEOPOROSIS: GLUCOCORTICOID-INDUCED
• subluxation We know that one of the most important risk factors for
osteoporosis is the use of corticosteroids. As these drugs are
Q-17 so widely used in clinical practice it is important we manage
Osteogenesis imperfecta is due to a defect in which one of this risk appropriately.
the following proteins?
The most widely followed guidelines are based around the
A. Fibrillin 2002 Royal College of Physicians (RCP) 'Glucocorticoid-
B. Elastin induced osteoporosis: A concise guide to prevention and
C. Type I collagen treatment'.
D. Type IV collagen
E. Polycystin-1 The risk of osteoporosis is thought to rise significantly once a
patient is taking the equivalent of prednisolone 7.5mg a day
ANSWER: for 3 or more months. It is important to note that we should
Type I collagen manage patients in an anticipatory, i.e. if it likely that the
patient will have to take steroids for at least 3 months then
EXPLANATION: we should start bone protection straight away, rather than
OSTEOGENESIS IMPERFECTA waiting until 3 months has elapsed. A good example is a
Osteogenesis imperfecta (more commonly known as brittle patient with newly diagnosed polymyalgia rheumatica. As it is
bone disease) is a group of disorders of collagen metabolism very likely they will be on a significant dose of prednisolone
resulting in bone fragility and fractures. The most common, for greater than 3 months bone protection should be
and milder, form of osteogenesis imperfecta is type 1 commenced immediately.

Overview Management of patients at risk of corticosteroid-induced


• autosomal dominant osteoporosis
• abnormality in type 1 collagen due to decreased synthesis
of pro-alpha 1 or pro-alpha 2 collagen polypeptides The RCP guidelines essentially divide patients into two groups.
1. Patients over the age of 65 years or those who've
Features previously had a fragility fracture should be offered bone
• presents in childhood protection.
• fractures following minor trauma 2. Patients under the age of 65 years should be offered a
bone density scan, with further management dependent:
• blue sclera
• deafness secondary to otosclerosis
T score Management
• dental imperfections are common
Greater than 0 Reassure
Between 0 and -1.5 Repeat bone density scan in 1-3 years
Q-18
Less than -1.5 Offer bone protection
What is the minimum steroid intake a patient should be
taking before they are offered osteoporosis prophylaxis?
The first-line treatment is alendronate. Patients should also be
calcium and vitamin D replete.
Q-19 Investigation
A 54-year-old lady presents with one-month history of • plain x-ray findings may be normal initially. Osteopenia
worsening right groin pain. The pain is worse on walking and and microfractures may be seen early on. Collapse of the
she has developed a limp. She has a background of systemic articular surface may result in the crescent sign
lupus erythematosus and is on long-term • MRI is the investigation of choice. It is more sensitive than
hydroxychloroquine with a recent course of prednisolone for radionuclide bone scanning
an acute flare. On examination, the patient has pain and
reduced range of movement particularly on internal rotation Management
of the right hip. • joint replacement may be necessary

Which of the following is the most sensitive investigation to Q-20


confirm the likely diagnosis? A 71-year-old man presents with an erythematous, swollen
first metatarsophalangeal joint on the left foot. This is
A. Pelvic x-ray causing him considerable pain and he is having difficulty
B. MRI right hip walking. He has never had any previous similar episodes. His
C. CT hip past medical history includes atrial fibrillation and type 2
D. Anti-phospholipid antibodies diabetes mellitus and his current medications are warfarin,
E. Right hip aspiration and microscopy metformin and simvastatin. What is the most appropriate
treatment of this episode?
ANSWER:
MRI right hip A. Intra-articular corticosteroid
B. Colchicine
EXPLANATION: C. Ibuprofen
This patient has avascular necrosis of her right femoral head D. Diclofenac
due to both her history of systemic lupus erythematosus and E. Prednisolone
recent steroid use. The most sensitive investigation is MRI
ANSWER:
scanning. X-rays may show increased density of the femoral
Colchicine
head in advanced cases. In practice, x-rays would usually be
done first-line. EXPLANATION:
NSAIDs should be avoided in elderly patients taking warfarin
Explanations for other options: due to the risk of a life-threatening gastrointestinal
1. In practice, pelvic x-rays would usually be done first-line haemorrhage. Oral steroids are an option but would upset
but they may be normal in early cases of avascular necrosis. his diabetic control.
3. CT scan of the hip is not as sensitive as MRI scanning.
4. Antiphospholipid antibodies are a possible cause of Whilst anticoagulation is not a contraindication to joint
avascular necrosis but would not confirm the diagnosis. injection it would make this option less attractive
5. Hip aspiration and microscopy would not be useful in
diagnosing avascular necrosis. It is helpful in cases of GOUT: MANAGEMENT
possible septic arthritis, gout or pseudogout, none of which Gout is a form of microcrystal synovitis caused by the
are suggested by this history. deposition of monosodium urate monohydrate in the
synovium. It is caused by chronic hyperuricaemia (uric acid >
AVASCULAR NECROSIS OF THE HIP 450 µmol/l)
Avascular necrosis (AVN) may be defined as death of bone Acute management
tissue secondary to loss of the blood supply. This leads to • NSAIDs or colchicine are first-line
bone destruction and loss of joint function. It most commonly
• the maximum dose of NSAID should be prescribed until 1-
affects the epiphysis of long bones such as the femur.
2 days after the symptoms have settled. Gastroprotection
(e.g. a proton pump inhibitor) may also be indicated
Causes
• colchicine* has a slower onset of action. The main side-
• long-term steroid use
effect is diarrhoea
• chemotherapy
• oral steroids may be considered if NSAIDs and colchicine
• alcohol excess are contraindicated. A dose of prednisolone 15mg/day is
• trauma usually used
• another option is intra-articular steroid injection
Features
• if the patient is already taking allopurinol it should be
• initially asymptomatic continued
• pain in the affected joint
Indications for urate-lowering therapy (ULT)
• the British Society of Rheumatology Guidelines now A. Resisted thumb flexion
advocate offering urate-lowering therapy to all patients B. Thumb extension
after their first attack of gout C. Flexion of the elbow
• ULT is particularly recommended if: D. Pronation of the forearm with the elbow flexed
• → >= 2 attacks in 12 months E. Resisted wrist extension with the elbow extended
• → tophi
ANSWER:
• → renal disease
Resisted wrist extension with the elbow extended
• → uric acid renal stones
• → prophylaxis if on cytotoxics or diuretics EXPLANATION:
Lateral epicondylitis: worse on resisted wrist
Urate-lowering therapy extension/suppination whilst elbow extended
• allopurinol is first-line
• it has traditionally been taught that urate-lowering LATERAL EPICONDYLITIS
therapy should not be started until 2 weeks after an acute Lateral epicondylitis typically follows unaccustomed activity
attack, as starting too early may precipitate a further such as house painting or playing tennis ('tennis elbow'). It is
attack. The evidence base to support this however looks most common in people aged 45-55 years and typically affects
weak the dominant arm.
• in 2017 the BSR updated their guidelines. They still
support a delay in starting urate-lowering therapy Features
because it is better for a patient to make long-term drug • pain and tenderness localised to the lateral epicondyle
decisions whilst not in pain • pain worse on wrist extension against resistance with the
• initial dose of 100 mg od, with the dose titrated every few elbow extended or supination of the forearm with the
weeks to aim for a serum uric acid of < 300 µmol/l. Lower elbow extended
initial doses should be given if the patient has a reduced • episodes typically last between 6 months and 2 years.
eGFR Patients tend to have acute pain for 6-12 weeks
• colchicine cover should be considered when starting
Management options
allopurinol. NSAIDs can be used if colchicine cannot be
• advice on avoiding muscle overload
tolerated. The BSR guidelines suggest this may need to be
• simple analgesia
continued for 6 months
• steroid injection
• the second-line agent when allopurinol is not tolerated or
• physiotherapy
ineffective is febuxostat (also a xanthine oxidase
inhibitor) Q-22
A health trust in the United Kingdom which serves a
Lifestyle modifications
population of 100,000 is planning services for patients with
• reduce alcohol intake and avoid during an acute attack
rheumatoid arthritis. How many of the population would be
• lose weight if obese
expected to have the disease?
• avoid food high in purines e.g. Liver, kidneys, seafood, oily
fish (mackerel, sardines) and yeast products A. 100
B. 300
Other points C. 1,000
• losartan has a specific uricosuric action and may be D. 2,000
particularly suitable for the many patients who have E. 10,000
coexistent hypertension
• increased vitamin C intake (either supplements or ANSWER:
through normal diet) may also decrease serum uric acid 1,000
levels
EXPLANATION:
*inhibits microtubule polymerization by binding to tubulin, The prevalence of rheumatoid arthritis in the UK population
interfering with mitosis. Also inhibits neutrophil motility and is approximately 1%
activity
RHEUMATOID ARTHRITIS: EPIDEMIOLOGY
Q-21 Epidemiology
A 47-year-old female presents with elbow pain. She has just • peak onset = 30-50 years, although occurs in all age
spent the weekend painting the house. On examination groups
there is localised pain around the lateral epicondyle and a • F:M ratio = 3:1
diagnosis of lateral epicondylitis is suspected. Which one of • prevalence = 1%
the following movements would characteristically worsen • some ethnic differences e.g. high in Native Americans
the pain? • associated with HLA-DR4 (especially Felty's syndrome)
Q-23 A. MRI
A 32-year-old gentleman is admitted with pain in his legs. On B. Plain x-ray
closer questioning, he has difficulty getting out of the chair C. Positron emission tomographic (PET) scan
and he has fallen several times in doing so. His past medical D. Bone biopsy
history includes Raynaud's which is controlled with E. CT scan
nifedipine. On examination, you notice skin changes
consistent with mechanic hands. What is the most likely ANSWER:
underlying diagnosis? MRI

A. Myasthenia gravis EXPLANATION:


B. Lambert-Eaton Myasthenic syndrome Osteomyelitis: MRI is the imaging modality of choice
C. McArdle disease
D. Anti-synthetase syndrome Please see Q-7 for Osteomyelitis
E. Drug-induced myopathy
Q-25
ANSWER: A 66-year-old female is on long-term prednisolone therapy
Anti-synthetase syndrome for polymyalgia rheumatica. What is the most appropriate
protection against osteoporosis?
EXPLANATION:
The key to this diagnosis is the proximal myopathy in A. Hormone replacement therapy
combination with mechanic hands, which are associated B. Calcitonin
with anti-synthetase syndrome. It is important to ask about C. Oral bisphosphonate
lung symptoms because there is an association with D. Calcium and vitamin D
interstitial lung disease - particularly non-specific interstitial E. Hip-protectors
pneumonia or NSIP.
ANSWER:
McArdle disease (or myophosphorylase deficiency) is Oral bisphosphonate
associated with cramping and fatigued muscles following
exercise. EXPLANATION:
Oral bisphosphonate therapy is recommended for patients
Myasthenia gravis causes fatiguability and is not associated older than 65 years who have taken, or who are likely to
with mechanic hands. remain on oral corticosteroids for more than 3 months

MYOPATHIES BISPHOSPHONATES
Features Bisphosphonates are analogues of pyrophosphate, a molecule
• symmetrical muscle weakness (proximal > distal) which decreases demineralisation in bone. They inhibit
• common problems are rising from chair or getting out of osteoclasts by reducing recruitment and promoting apoptosis.
bath
• sensation normal, reflexes normal, no fasciculation Clinical uses
• prevention and treatment of osteoporosis
Causes • hypercalcaemia
• inflammatory: polymyositis • Paget's disease
• inherited: Duchenne/Becker muscular dystrophy, • pain from bone metatases
myotonic dystrophy
• endocrine: Cushing's, thyrotoxicosis Adverse effects
• alcohol • oesophageal reactions: oesophagitis, oesophageal ulcers
(especially alendronate)
Q-24 • osteonecrosis of the jaw
A 65-year-old man with a history of type 2 diabetes mellitus • increased risk of atypical stress fractures of the proximal
and peripheral arterial disease is investigated for fatigue and femoral shaft in patients taking alendronate
pyrexia of unknown origin. He recently had an amputation of • acute phase response: fever, myalgia and arthralgia may
a toe on his left foot. A diagnosis of osteomyelitis is occur following administration
suspected in the left foot. What is the most appropriate • hypocalcaemia: due to reduced calcium efflux from bone.
investigation? Usually clinically unimportant

The BNF suggests the following counselling for patients taking


oral bisphosphonates
'Tablets should be swallowed whole with plenty of water • pyrexia: typically rises in the late afternoon/early evening
while sitting or standing; to be given on an empty stomach at in a daily pattern and accompanies a worsening of joint
least 30 minutes before breakfast (or another oral symptoms and rash
medication); patient should stand or sit upright for at least 30 • lymphadenopathy
minutes after taking tablet' • rheumatoid factor (RF) and anti-nuclear antibody (ANA)
negative
The duration of bisphosphonate treatment varies according to
the level of risk. Some authorities recommend stopping Management
bisphosphonates at 5 years if the following apply: • NSAIDs should be used to manage fever, joint pain and
• patient is < 75-years-old serositis in the first instance. They should be trialled for at
• femoral neck T-score of > -2.5 least a week before steroids are added. Steroids may
• low risk according to FRAX/NOGG control symptoms but won't improve prognosis. If
symptoms persist, the use of methotrexate, IL-1 or anti-
Q-26 TNF therapy can be considered
A 29-year-old with adult onset Still's comes to the
rheumatology clinic for review. She still has significant Q-27
symptoms including joint pains and a persistent skin rash A 43-year-old woman who has rheumatoid arthritis is
despite methotrexate and etanercept. Examination reveals a reviewed in clinic. She has responded poorly to
macular erythematous rash and active synovitis. CRP is methotrexate and consideration is being given to starting
elevated at 95 mg/l. sulfasalazine. An existing allergy to which one of the
following drugs may be a contradiction to the prescription?
Which of the following is the most appropriate next
intervention? A. Penicillin
B. Trimethoprim
A. Denosumab C. Aspirin
B. Mepolizumab D. Sulpiride
C. Rituximab E. Leflunomide
D. Tocilizumab
E. Anakinra ANSWER:
ANSWER: Aspirin
Rituximab
EXPLANATION:
EXPLANATION: Patients who are allergic to aspirin may also react to
The evidence is very strong to support the use of IL-1 sulfasalazine
inhibition for adult onset Still's disease. A review of
published data estimated the overall remission rate at over SULFASALAZINE
80%, and complete remission rate at approximately 66%. Sulfasalazine is a disease modifying anti-rheumatic drug used
Anakinra competitively inhibits the action of IL-1 by binding in the management of inflammatory arthritis, especially
to the IL-1 receptor and plasma levels correlate well with IL-1 rheumatoid arthritis. It is also used in the management of
in synovial fluid and presence of synovitis. inflammatory bowel disease.

Denosumab is a RANK ligand inhibitor used in the treatment Cautions


of osteoporosis. Mepolizumab is used in the treatment of • GP6D deficiency
severe asthma with elevated eosinophils. There is some • allergy to aspirin or sulphonamides (cross-sensitivity)
evidence to support the use of tocilizumab, an anti-IL6
monoclonal antibody in adult onset Still's, but there is Adverse effects
stronger data to support a role for tocilizumab in the • oligospermia
treatment of rheumatoid arthritis. Rituximab is usually
• Stevens-Johnson syndrome
instigated in patients who fail to respond to anakinra.
• pneumonitis / lung fibrosis
STILL'S DISEASE IN ADULTS • myelosuppression, Heinz body anaemia, megaloblastic
Adult Still's disease anaemia
• typically affects 16-35 year olds • may colour tears → stained contact lenses

Features Q-28
• arthralgia A 33-year-old female presents 6 weeks after the birth of her
• elevated serum ferritin first child with a two-week history of polyarthralgia, fever
• rash: salmon-pink, maculopapular and a skin rash. First-line investigations show:
Adverse effects
ESR 45 mm/hour • mucositis
• myelosuppression
What is the most likely diagnosis? • pneumonitis
• pulmonary fibrosis
A. Polymorphic eruption of pregnancy • liver fibrosis
B. Systemic lupus erythematous
C. Rheumatoid arthritis Pregnancy
D. Reactive arthritis • women should avoid pregnancy for at least 3 months
E. Pseudogout after treatment has stopped
• the BNF also advises that men using methotrexate need
ANSWER: to use effective contraception for at least 3 months after
Systemic lupus erythematous treatment

EXPLANATION: Prescribing methotrexate


Unlike many autoimmune diseases systemic lupus • methotrexate is a drug with a high potential for patient
erythematous (SLE) often becomes worse during pregnancy harm. It is therefore important that you are familiar with
and the puerperium guidelines relating to its use
• methotrexate is taken weekly, rather than daily
SYSTEMIC LUPUS ERYTHEMATOSUS: PREGNANCY
• FBC, U&E and LFTs need to be regularly monitored. The
Overview
Committee on Safety of Medicines recommend 'FBC and
• risk of maternal autoantibodies crossing placenta renal and LFTs before starting treatment and repeated
• leads to condition termed neonatal lupus erythematous weekly until therapy stabilised, thereafter patients should
• neonatal complications include congenital heart block be monitored every 2-3 months'
• strongly associated with anti-Ro (SSA) antibodies • folic acid 5mg once weekly should be co-prescribed,
taken more than 24 hours after methotrexate dose
Q-29 • the starting dose of methotrexate is 7.5 mg weekly
You review a 48-year-old woman who is taking methotrexate (source: BNF)
for rheumatoid arthritis. Concurrent prescription of which • only one strength of methotrexate tablet should be
other medication should be avoided? prescribed (usually 2.5 mg)
• avoid prescribing trimethoprim or cotrimoxazole
A. Erythromycin
concurrently - increases risk of marrow aplasia
B. Trimethoprim
C. Sumatriptan
Q-30
D. Lansoprazole
A 74-year-old man presents with a severe throbbing
E. Sodium valproate
headache on the right side of his head. He has now had this
pain for around 6-7 days but reports no obvious trigger.
ANSWER:
There have been no visual disturbances or episodes of limb
Trimethoprim
weakness. Neurological examination is unremarkable. The
right side of his head is tender to touch but he cannot
EXPLANATION:
remember falling. Given the likely diagnosis what is the most
There is an increased risk of haematological toxicity when
important initial step?
trimethoprim is prescribed alongside methotrexate.
A. Give high-dose oral prednisolone
METHOTREXATE
B. Arrange an urgent orbital x-ray for suspected blow-out
Methotrexate is an antimetabolite that inhibits dihydrofolate
fracture
reductase, an enzyme essential for the synthesis of purines
C. Arrange an urgent temporal artery biopsy
and pyrimidines. It is considered an 'important' drug as whilst
D. Arrange an urgent CT head
it can be very effective in controlling disease the side-effects
E. Ocular pilocarpine + intravenous acetazolamide
may be potentially life-threatening - careful prescribing and
close monitoring is essential.
ANSWER:
Give high-dose oral prednisolone
Indications
• inflammatory arthritis, especially rheumatoid arthritis EXPLANATION:
• psoriasis This patient has temporal arteritis and requires high-dose
• some chemotherapy acute lymphoblastic leukaemia oral steroids to prevent ocular complications
TEMPORAL ARTERITIS Features
Temporal arteritis is large vessel vasculitis which overlaps with • tall stature with arm span to height ratio > 1.05
polymyalgia rheumatica (PMR). Histology shows changes • high-arched palate
which characteristically 'skips' certain sections of affected • arachnodactyly
artery whilst damaging others. • pectus excavatum
• pes planus
Features
• scoliosis of > 20 degrees
• typically patient > 60 years old
• heart: dilation of the aortic sinuses (seen in 90%) which
• usually rapid onset (e.g. < 1 month)
may lead to aortic aneurysm, aortic dissection, aortic
• headache (found in 85%)
regurgitation, mitral valve prolapse (75%),
• jaw claudication (65%)
• lungs: repeated pneumothoraces
• visual disturbances secondary to anterior ischemic optic
• eyes: upwards lens dislocation (superotemporal ectopia
neuropathy
lentis), blue sclera, myopia
• tender, palpable temporal artery
• dural ectasia (ballooning of the dural sac at the
• around 50% have features of PMR: aching, morning
lumbosacral level)
stiffness in proximal limb muscles (not weakness)
• also lethargy, depression, low-grade fever, anorexia, night The life expectancy of patients used to be around 40-50 years.
sweats With the advent of regular echocardiography monitoring and
beta-blocker/ACE-inhibitor therapy this has improved
Investigations significantly over recent years. Aortic dissection and other
• raised inflammatory markers: ESR > 50 mm/hr (note ESR cardiovascular problems remain the leading cause of death
< 30 in 10% of patients). CRP may also be elevated however.
• temporal artery biopsy: skip lesions may be present
• note creatine kinase and EMG normal Q-32
A 46-year-old female presents with a burning sensation over
Treatment
the antero-lateral aspect of her right thigh. A diagnosis of
• high-dose prednisolone - there should be a dramatic
meralgia paraesthetica is suspected. Which nerve is most
response, if not the diagnosis should be reconsidered
likely to be affected?
• urgent ophthalmology review. Patients with visual
symptoms should be seen the same-day by an
A. Common peroneal nerve
ophthalmologist. Visual damage is often irreversible
B. Anterior cutaneous nerve of thigh
Q-31 C. Posterior cutaneous nerve of thigh
A 45-year-old man who is known to have Marfan's syndrome D. Lateral cutaneous nerve of thigh
presents with lower back pain. This has been present for a E. Sciatic nerve
few months now and is associated with headaches, leg pain
and intermittent episodes of urinary incontinence. What is ANSWER:
the most likely diagnosis? Lateral cutaneous nerve of thigh

A. Depression EXPLANATION:
B. Spinal stenosis Burning thigh pain - ? meralgia paraesthetica - lateral
C. Leaking aortic abdominal aneurysm cutaneous nerve of thigh compression
D. Multiple sclerosis
MERALGIA PARAESTHETICA
E. Dural ectasia
Meralgia paraesthetica comes from the Greek words meros
ANSWER: for thigh and algos for pain and is often described as a
Dural ectasia syndrome of paraesthesia or anaesthesia in the distribution of
the lateral femoral cutaneous nerve (LFCN). It is an
EXPLANATION: entrapment mononeuropathy of the LFCN, but can also be
Dural ectasia affects around 60% of patients with Marfan's iatrogenic after a surgical procedure, or result from a
syndrome. It may cause lower back pain associated with neuroma. Although uncommon, meralgia paraesthetica is not
neurological problems such as bladder and bowel rare and is hence probably underdiagnosed.
dysfunction.
Anatomy
MARFAN'S SYNDROME • The LFCN is primarily a sensory nerve, carrying no motor
Marfan's syndrome is an autosomal dominant connective fibres.
tissue disorder. It is caused by a defect in the FBN1 gene on • It most commonly originates from the L2/3 segments.
chromosome 15 that codes for the protein fibrillin-1. It affects • After passing behind the psoas muscle, it runs beneath
around 1 in 3,000 people. the iliac fascia as it crosses the surface of the iliac muscle
and eventually exits through or under the lateral aspect Q-33
of the inguinal ligament. A 65-year-old Asian female presents with generalised bone
• As the nerve curves medially and inferiorly around the pain and muscle weakness. Investigations show:
anterior superior iliac spine (ASIS), it may be subject to
repetitive trauma or pressure. Calcium 2.07 mmol/l
• Compression of this nerve anywhere along its course can Phosphate 0.66 mmol/l
lead to the development of meralgia paraesthetica. ALP 256 U/l

Epidemiology What is the most likely diagnosis?


• The majority of cases occur in people aged between 30
and 40. A. Bone tuberculosis
• In some, both legs may be affected. B. Hypoparathyroidism
• It is more common in men than women. C. Myeloma
• Occurs more commonly in those with diabetes than in the D. Osteomalacia
general population. E. Paget's disease

Risk factors 3 ANSWER:


• Obesity Osteomalacia
• Pregnancy
• Tense ascites EXPLANATION:
Osteomalacia
• Trauma
• low: calcium, phosphate
• Iatrogenic, such as pelvic osteotomy, spinal surgeries,
laparoscopic hernia repair and bariatric surgery. In some • raised: alkaline phosphatase
cases, may result from abduction splints used in the The low calcium and phosphate combined with the raised
management of Perthe's disease. alkaline phosphatase point towards osteomalacia
• Various sports have been implicated, including
OSTEOMALACIA
gymnastics, football, bodybuilding and strenuous
Basics
exercise.
• normal bony tissue but decreased mineral content
• Some cases are idiopathic.
• rickets if when growing
Patients typically present with the following symptoms in the • osteomalacia if after epiphysis fusion
upper lateral aspect of the thigh:
• Burning, tingling, coldness, or shooting pain Types
• Numbness • vitamin D deficiency e.g. malabsorption, lack of sunlight,
diet
• Deep muscle ache
• renal failure
• Symptoms are usually aggravated by standing, and
relieved by sitting • drug induced e.g. anticonvulsants
• They can be mild and resolve spontaneously or may • vitamin D resistant; inherited
severely restrict the patient for many years. • liver disease, e.g. cirrhosis

Signs: Features
• Symptoms may be reproduced by deep palpation just • rickets: knock-knee, bow leg, features of hypocalcaemia
below the ASIS (pelvic compression) and also by • osteomalacia: bone pain, fractures, muscle tenderness,
extension of the hip. proximal myopathy
• There is altered sensation over the upper lateral aspect of
the thigh. Investigation
• There is no motor weakness. • low 25(OH) vitamin D (in 100% of patients, by definition)
• raised alkaline phosphatase (in 95-100% of patients)
Investigations: • low calcium, phosphate (in around 30%)
• The pelvic compression test is highly sensitive, and often, • x-ray: children - cupped, ragged metaphyseal surfaces;
meralgia paraesthetica can be diagnosed based on this adults - translucent bands (Looser's zones or
test alone pseudofractures)
• Injection of the nerve with local anaesthetic will abolish
the pain. Using ultrasound is effective both for diagnosis Treatment
and guiding injection therapy in meralgia paraesthetica • calcium with vitamin D tablets
• Nerve conduction studies may be useful.
Q-34
Which one of the following is least recognised in polyarteritis
nodosa?

A. Cytoplasmic-antineutrophil cytoplasmic antibodies


B. Hypertension
C. Mononeuritis multiplex
D. Pyrexia
E. Renal failure

ANSWER:
Cytoplasmic-antineutrophil cytoplasmic antibodies

EXPLANATION:
Perinuclear-antineutrophil cytoplasmic antibodies are found
in around 20% of patients

POLYARTERITIS NODOSA
Polyarteritis nodosa (PAN) is a vasculitis affecting medium-
sized arteries with necrotizing inflammation leading to
aneurysm formation. PAN is more common in middle-aged
men and is associated with hepatitis B infection

Features
• fever, malaise, arthralgia
• weight loss
• hypertension
• mononeuritis multiplex, sensorimotor polyneuropathy
• testicular pain
• livedo reticularis
• haematuria, renal failure
• perinuclear-antineutrophil cytoplasmic antibodies (ANCA)
are found in around 20% of patients with 'classic' PAN
• hepatitis B serology positive in 30% of patients

Angiogram from a patient with polyarteritis nodosa. Both kidneys demonstrate


beading and numerous microaneurysms affecting the intrarenal vessels. Similar
changes are seen affecting the intrahepatic vessels with a few small
microaneurysms noted. The proximal branches of the SMA appears normal;
however there are no normal straight arteries from the jejunal arteries and lack of
normal anastomotic arcades and loops. This is associated with multiple
microaneurysms.

Q-35
A 63-year-old man presents to the Emergency Department
with a 2 day history of a painful and swollen left knee joint.
Aspiration reveals positively birefringent crystals and no
organisms are seen. Which of the following conditions are
not recognised causes of the underlying condition?

A. Haemochromatosis
B. Low magnesium
C. High phosphate
Livedo reticularis D. Acromegaly
E. Hyperparathyroidism
ANSWER: EHLER-DANLOS SYNDROME
High phosphate Ehler-Danlos syndrome is an autosomal dominant connective
tissue disorder that mostly affects type III collagen. This
EXPLANATION: results in the tissue being more elastic than normal leading to
A low phosphate predisposes to pseudogout joint hypermobility and increased elasticity of the skin.

PSEUDOGOUT Features and complications


Pseudogout is a form of microcrystal synovitis caused by the • elastic, fragile skin
deposition of calcium pyrophosphate dihydrate in the • joint hypermobility: recurrent joint dislocation
synovium • easy bruising
• aortic regurgitation, mitral valve prolapse and aortic
Risk factors dissection
• hyperparathyroidism • subarachnoid haemorrhage
• hypothyroidism • angioid retinal streaks
• haemochromatosis
• acromegaly Q-37
• low magnesium, low phosphate A 57-year-old woman with a history of polymyalgia
• Wilson's disease rheumatica has been taking prednisolone 10 mg for the past
5 months. A DEXA scan is reported as follows:
Features
• knee, wrist and shoulders most commonly affected L2 T-score -1.6 SD
• joint aspiration: weakly-positively birefringent rhomboid Femoral neck T-score -1.7 SD
shaped crystals
• x-ray: chondrocalcinosis What is the most suitable management?

Management A. No treatment
• aspiration of joint fluid, to exclude septic arthritis B. Vitamin D + calcium supplementation + repeat DEXA
• NSAIDs or intra-articular, intra-muscular or oral steroids scan in 6 months
as for gout C. Vitamin D + calcium supplementation
D. Vitamin D + calcium supplementation + hormone
Q-36 replacement therapy
A 24-year-old male comes into the GP with due to constant E. Vitamin D + calcium supplementation + oral
bruising. On examination, he has an early-diastolic murmur bisphosphonate
over the aortic region. His skin is very elastic and his joints
extend further than they normally should. He is referred for ANSWER:
genetic tests, which confirm the suspected diagnosis of Vitamin D + calcium supplementation + oral bisphosphonate
Ehlers-Danlos syndrome. Which type of collagen is primarily
affected by Ehlers-Danlos syndrome? EXPLANATION:
This patient has been taking 10mg of prednisolone for the
A. Collagen type 1 past 5 months and hence should be assessed for bone
B. Collagen type 2 protection. The T score of less than -1.5 SD is an indication
C. Collagen type 3 for a bisphosphonate. This should be co-prescribed with
D. Collagen type 4 calcium + vitamin D.
E. Collagen type 5
Please see Q-18 for Osteoporosis: Glucocorticoid Induced
ANSWER:
Collagen type 4 Q-38
A 24-year-old man is investigated for chronic back pain.
EXPLANATION: Which one of the following would most suggest a diagnosis
Ehlers-Danlos syndrome is most commonly associated with a of ankylosing spondylitis?
defect in type III collagen
Ehlers-Danlos syndrome is primarily caused by a genetic A. Reduced lateral flexion of the lumbar spine
defect in collagen type III. Collagen Type V is affected in a B. Pain gets worse during the day
less common variant of Ehlers-Danlos syndrome. C. Accentuated lumbar lordosis
Collagen type I is defective in osteogenesis imperfecta. D. Pain on straight leg raising
Collagen IV is defective in Goodpasture's syndrome. E. Loss of thoracic kyphosis
ANSWER: cardiomyopathy and severe, generalised muscular
Reduced lateral flexion of the lumbar spine hypotonia.

EXPLANATION: Please see Q-4 for McArdle’s Disease


Reduced lateral flexion of the lumbar spine is one of the
earliest signs of ankylosing spondylitis. There tends to be a Q-40
loss of lumbar lordosis and an accentuated thoracic kyphosis A 62-year-old man is reviewed on the acute medical ward
in patients with ankylosing spondylitis with a two week history of lumbar back pain and fevers. He
complains of point tenderness over the L1-2 area and finds it
Please see Q-3 for Ankylosing Spondylitis: Features difficult to mobilise.

Q-39 You arrange for an MRI of his lumbar spine which is reported
A 17-year-old male presents to General Practice with as follows:
recurrent cramping in his arms during a rowing training
session. On closer questioning, he has noticed a reduction in 'Area of hyperintensity visible at the L1-2 disc space with
his exercise tolerance recently. He reports getting a 'second vertebral end plate changes suggestive of vertebral
wind' phenomenon after minutes of rest. osteomyelitis/discitis.'

On examination, he is well with no apparent deficit. Which of the following organisms is most likely to be
responsible?
Bloods reveal:
A. Streptococcus pyogenes
Hb 142 g/l B. Salmonella enterica
Platelets 204 * 109/l C. Staphylococcus aureus
WBC 6.7 * 109/l D. Escherichia coli
Na+ 139 mmol/l E. Pseudomonas aeruginosa
K+ 4.2 mmol/l
Urea 8.9 mmol/l ANSWER:
Creatinine 148 µmol/l Staphylococcus aureus
Adjusted calcium 2.23 mmol/L
Creatine kinase 1037 IU/L EXPLANATION:
Autoimmune profile pending Staphylococcus aureus is the most common cause of
osteomyelitis
What is the most likely underlying diagnosis? In the majority of cases, osteomyelitis and discitis is caused
by Staphylococcus aureus.
A. Polymyositis
B. Dermatomyositis In patients with sickle cell anaemia and other
C. Von Gierke's disease haemoglobinopathies, Salmonella sp. become more common
D. McArdle's disease however it is usually preceded by a period of gastrointestinal
E. Pompe's disease upset.

ANSWER: The other organisms are all capable of causing osteomyelitis


McArdle's disease and discitis however disease caused by these organisms
occurs much less commonly than that caused by
EXPLANATION: Staphylococcus aureus.
McArdle's disease is characteristically associated with a
'second-wind' phenomenon and present with distal muscle Please see Q-7 for Osteomyelitis
cramping in this demographic following exercise.
Q-41
Polymyositis and dermatomyositis would, typically but not Which one of the following statements regarding ankylosing
always, cause a more marked increase in creatine kinase spondylitis is correct?
which can be up to 100 times the upper limit of normal.
A. Schober's test assesses reduced chest expansion
Von Gierke patients present with manifestations related to B. HLA-B27 is positive in 50% of patients
hypoglycemia around three to four months of age. Pompe's C. Achilles tendonitis is a recognised association
disease tends to also present in infants but with D. It is equally common in men and women
E. The typical age of presentation is between 40-50 years
ANSWER:
Achilles tendonitis is a recognised association

EXPLANATION:
HLA-B27 is positive in 90% of patients.

Please see Q-3 for Ankylosis Spondylitis: Features

Q-42
A 55-year-old man presents with pain and stiffness in his
hands. This has been getting gradually worse over the past
few months and is associated with stiffness in the mornings.

On examination, you note bilateral swelling of the


metacarpal phalangeal (MCP) and distal interphalangeal
(DIP). One of the digits is swollen along the whole length. Notice the nail changes on this image as well

What is the most likely diagnosis?

A. Osteoarthritis
B. Rheumatoid arthritis
C. Psoriatic arthritis
D. Gout
E. Reactive arthritis

ANSWER:
Psoriatic arthritis

EXPLANATION:
Inflammatory arthritis involving DIP swelling and dactylitis
points to a diagnosis of psoriatic arthritis
The morning stiffness points to an inflammatory arthritis
such as rheumatoid or psoriatic. However, DIP and dactylitis
are much more common in psoriatic arthritis, making this the
most likely diagnosis.

PSORIATIC ARTHROPATHY
Psoriatic arthropathy correlates poorly with cutaneous
psoriasis and often precedes the development of skin lesions.
Around 10-20% percent of patients with skin lesions develop
an arthropathy with males and females being equally affected

Types*
• rheumatoid-like polyarthritis: (30-40%, most common
type)
• asymmetrical oligoarthritis: typically affects hands and
feet (20-30%)
• sacroilitis
• DIP joint disease (10%)
• arthritis mutilans (severe deformity fingers/hand,
'telescoping fingers') X-ray showing some of changes in seen in psoriatic arthropathy. Note that the
DIPs are predominately affected, rather than the MCPs and PIPs as would be seen
with rheumatoid. Extensive juxta-articular periostitis is seen in the DIPs but the
Management
changes have not yet progressed to the classic 'pencil-in-cup' changes that are
• treat as rheumatoid arthritis often seen.
• but better prognosis
Q-43
Which one of the following is least associated with Behcet's
syndrome?

A. Mouth ulcers
B. Genital ulcers
C. Conjunctivitis
D. Deep vein thrombosis
E. Aseptic meningitis

ANSWER:
Conjunctivitis

EXPLANATION:
Oral ulcers + genital ulcers + anterior uveitis = Behcet's
Mouth ulcers, genital ulcers, deep vein thrombosis and
aseptic meningitis are all recognised features of Behcet's
syndrome

Ocular involvement is the most feared complication of


Behcet's syndrome. Conjunctivitis is seen rarely and is much
less common than anterior uveitis. Other ocular problems
seen include retinal vasculitis, iridocyclitis and chorioretinitis

BEHCET'S SYNDROME
Behcet's syndrome is a complex multisystem disorder
associated with presumed autoimmune mediated
inflammation of the arteries and veins. The precise aetiology
has yet to be elucidated however. The classic triad of
symptoms are oral ulcers, genital ulcers and anterior uveitis

Epidemiology
• more common in the eastern Mediterranean (e.g. Turkey)
• more common in men (complicated gender distribution
which varies according to country. Overall, Behcet's is
considered to be more common and more severe in men)
• tends to affect young adults (e.g. 20 - 40 years old)
• associated with HLA B5* and MICA6 allele
• around 30% of patients have a positive family history

Features
• classically: 1) oral ulcers 2) genital ulcers 3) anterior
uveitis
• thrombophlebitis
• arthritis
• neurological involvement (e.g. aseptic meningitis)
• GI: abdo pain, diarrhoea, colitis
• erythema nodosum, DVT
This x-ray shows changes affecting both the PIPs and DIPs. The close-up images
Diagnosis
show extensive changes including large eccentric erosions, tuft resorption and
progresion towards a 'pencil-in-cup' changes. • no definitive test
• diagnosis based on clinical findings
*Until recently it was thought asymmetrical oligoarthritis was • positive pathergy test is suggestive (puncture site
the most common type, based on data from the original 1973 following needle prick becomes inflamed with small
Moll and Wright paper. Please see the link for a comparison of pustule forming)
more recent studies
*more specifically HLA B51, a split antigen of HLA B5
Q-44 De Quervain's tenosynovitis typically causes symptoms
Which one of the following cytokines is the most important affecting the base of the thumb.
in the pathophysiology of rheumatoid arthritis?
CUBITAL TUNNEL SYNDROME
A. IFN-beta Cubital tunnel syndrome occurs due to compression of the
B. IFN-alpha ulnar nerve as it passes through the cubital tunnel.
C. IL-4
D. Tumour necrosis factor History and Examination -
E. IL-2 • Tingling and numbness of the 4th and 5th finger which
starts off intermittent and then becomes constant.
ANSWER: • Over time patients may also develop weakness and
Tumour necrosis factor muscle wasting
• Pain worse on leaning on the affected elbow
EXPLANATION: • Often a history of osteoarthritis or prior trauma to the
Rheumatoid arthritis - TNF is key in pathophysiology area.

Please see Q-10 for Tumour Necrosis Factor Investigations -


• X-ray
Q-45 • nerve conduction studies
A 67-year-old reports tingling and numbness in the little and
ring finger of his left hand. He reports the pain initially came Management -
and went but has now become constant. The pain is worse if • Avoid aggravating activity
he leans on his elbow and he recalls a previous elbow • Physiotherapy
fracture as a child. • Steroid injections
• Surgery in resistant cases
Given the clinical history what is the most likely cause of his
symptoms?
Q-46
A 44-year-old female with a history of Raynaud's
A. Cubital tunnel syndrome
phenomenon is reviewed in the rheumatology clinic. She is
B. Medial epicondylitis
currently being investigated for dysphagia. On examination
C. Radial tunnel syndrome
she is noted to have tight, shiny skin over her fingers. Which
D. Carpal tunnel syndrome
one of the following complications is she most likely to
E. De Quervains tenosynovitis
develop?
ANSWER:
A. Early onset dementia
Cubital tunnel syndrome
B. Erythema nodosum
C. Malabsorption
EXPLANATION:
D. Constrictive pericarditis
Cubital tunnel syndrome is caused by compression of the
E. Erosive joint disease
ulnar nerve and can present with tingling/numbness of the
4th and 5th finger
ANSWER:
The answer here is cubital tunnel syndrome which occurs due
Malabsorption
to compression of the ulnar nerve as it passes through the
cubital tunnel. Initial symptoms include tingling and
EXPLANATION:
numbness of the 4th and 5th finger. Often pain is worse on
This patient is likely to have CREST syndrome, a subtype of
leaning on the affected elbow and there is a history of
limited cutaneous systemic sclerosis. Malabsorption can
osteoarthritis or prior trauma to the area.
develop in these patients secondary to bacterial overgrowth
of the sclerosed small intestine
Medial epicondylitis would not cause distal symptoms in the
hand but pain of the medial elbow.
Whilst diffuse systemic sclerosis is associated with more
severe and rapid internal organ involvement it is also seen in
Radial tunnel syndrome causes aching and paraesthesia of
the limited form.
the hand with forearm pain distal to the lateral epicondyle
SYSTEMIC SCLEROSIS
Carpal tunnel syndrome is caused by compression of the
Systemic sclerosis is a condition of unknown aetiology
median nerve and presents with pain and tingling of the
characterised by hardened, sclerotic skin and other
thumb, index, middle and medial half of the ring finger
connective tissues. It is four times more common in females
There are three patterns of disease: Antibodies
• ANA positive in 90%
Limited cutaneous systemic sclerosis • RF positive in 30%
• Raynaud's may be first sign • anti-scl-70 antibodies associated with diffuse cutaneous
• scleroderma affects face and distal limbs predominately systemic sclerosis
• associated with anti-centromere antibodies • anti-centromere antibodies associated with limited
• a subtype of limited systemic sclerosis is CREST cutaneous systemic sclerosis
syndrome: Calcinosis, Raynaud's phenomenon,
oEsophageal dysmotility, Sclerodactyly, Telangiectasia Q-47
Which of the following findings is not typical in a patient
Diffuse cutaneous systemic sclerosis with antiphospholipid syndrome?
• scleroderma affects trunk and proximal limbs
predominately A. Prolonged APTT
• associated with scl-70 antibodies B. Thrombocytosis
• hypertension, lung fibrosis and renal involvement seen C. Recurrent venous thrombosis
• poor prognosis D. Recurrent arterial thrombosis
E. Livedo reticularis
Scleroderma (without internal organ involvement)
• tightening and fibrosis of skin ANSWER:
• may be manifest as plaques (morphoea) or linear Thrombocytosis

EXPLANATION:
Antiphospholipid syndrome: arterial/venous thrombosis,
miscarriage, livedo reticularis
Thrombocytopenia is associated with antiphospholipid
syndrome

ANTIPHOSPHOLIPID SYNDROME
Antiphospholipid syndrome is an acquired disorder
characterised by a predisposition to both venous and arterial
thromboses, recurrent fetal loss and thrombocytopenia. It
may occur as a primary disorder or secondary to other
conditions, most commonly systemic lupus erythematosus
(SLE)

A key point for the exam is to appreciate that


antiphospholipid syndrome causes a paradoxical rise in the
APTT. This is due to an ex-vivo reaction of the lupus
anticoagulant autoantibodies with phospholipids involved in
the coagulation cascade

Features
• venous/arterial thrombosis
• recurrent fetal loss
• livedo reticularis
• thrombocytopenia
• prolonged APTT
• other features: pre-eclampsia, pulmonary hypertension

Associations other than SLE


• other autoimmune disorders
• lymphoproliferative disorders
• phenothiazines (rare)

Management - based on BCSH guidelines


• initial venous thromboembolic events: evidence currently • infective endocarditis (= 50%)
supports use of warfarin with a target INR of 2-3 for 6 • SLE (= 20-30%)
months • systemic sclerosis (= 30%)
• recurrent venous thromboembolic events: lifelong • general population (= 5%)
warfarin; if occurred whilst taking warfarin then increase • rarely: TB, HBV, EBV, leprosy
target INR to 3-4
• arterial thrombosis should be treated with lifelong Anti-cyclic citrullinated peptide antibody
warfarin with target INR 2-3 Anti-cyclic citrullinated peptide antibody may be detectable
up to 10 years before the development of rheumatoid
Q-48 arthritis. It may therefore play a key role in the future of
You review a 40-year-old mechanic who presents with joint rheumatoid arthritis, allowing early detection of patients
pains. For the past two months he has noticed intermittent suitable for aggressive anti-TNF therapy. It has a sensitivity
pain, stiffness and swelling of the joints in his hands and similar to rheumatoid factor (around 70%) with a much higher
feet. The stiffness tends to improve during the day but the specificity of 90-95%.
pain tends to get worse. He has also noticed stiffness in his
back but cannot remember any aggravating injury. You order NICE recommends that patients with suspected rheumatoid
some blood tests (taken during an acute attack) which are arthritis who are rheumatoid factor negative should be test
reported as follows: for anti-CCP antibodies.

Rheumatoid factor Negative Q-49


Anti-cyclic citrullinated peptide antibody Positive A 45-year-old woman is referred to rheumatology
Uric acid0.3 mmol/l (0.18 - 0.48) outpatients with a 4 month history of joint pains, myalgia
ESR 41 mm/hr and generalised lethargy. An autoantibody screen reveals
she is ANA positive and anti-ribonucleoprotein positive. The
What is the most likely diagnosis? creatine kinase is elevated at 525. What is the most likely
diagnosis?
A. Reactive arthritis
B. Ankylosing spondylitis A. Systemic lupus erythematous
C. Gout B. Mixed connective tissue disease
D. Osteoarthritis C. Polymyositis
E. Rheumatoid arthritis D. Dermatomyositis
E. CREST syndrome
ANSWER:
Rheumatoid arthritis ANSWER:
Mixed connective tissue disease
EXPLANATION:
Anti-cyclic citrullinated peptide antibodies are associated EXPLANATION:
with rheumatoid arthritis Anti-ribonuclear protein (anti-RNP) = mixed connective tissue
Anti-cyclic citrullinated peptide antibody is highly specific for disease
rheumatoid arthritis.
MIXED CONNECTIVE TISSUE DISEASE
RHEUMATOID ARTHRITIS: ANTIBODIES Mixed connective tissue disease (MCTD, Sharp's syndrome) is
Rheumatoid factor a rare, heterogeneous, multi-system autoimmune disorder. It
Rheumatoid factor (RF) is a circulating antibody (usually IgM) is a distinct clinical entity, but features of systemic lupus
which reacts with the Fc portion of the patients own IgG erythematosus (SLE), systemic sclerosis (SSc) and myositis may
all be present. It is associated with anti-U1 ribonucleoprotein
RF can be detected by either (RNP) antibodies.* It is not to be confused with
• Rose-Waaler test: sheep red cell agglutination 'undifferentiated connective tissue disease'.**
• Latex agglutination test (less specific)
Epidemiology
RF is positive in 70-80% of patients with rheumatoid arthritis, • Male:female ratio 1:3
high titre levels are associated with severe progressive disease • Average age of presentation 30-40, may present in
(but NOT a marker of disease activity) children
• Rare - incidence in adult population is estimated to be
Other conditions associated with a positive RF include: 2.1/million/year in one Norwegian study
• Sjogren's syndrome (around 100%)
• Felty's syndrome (around 100%)
Presentation: Q-50
• Raynaud's phenomenon often precedes other symptoms Cristiano, 36-year-old gentleman presents to the emergency
and occurs in 90% of cases department complaining of a painful, swollen knee. On
• Polyarthralgia/arthritis examination, he is limping and unable to fully extend or flex
• Myalgia his knee. He has a tense effusion in his knee, which is warm
• 'Sausage fingers'(dactylitis) to touch and swollen. His eyes also appear red and irritated.
He reports dysuria and tells you that he was diagnosed and
Other clinically important features: treated for an STI three weeks ago. The patient records show
• Dermatological: photosensitive rash, scleroderma-like that Cristiano has previously tested HLA-B27 positive and has
changes, alopecia presented several times in the past with similar symptoms.
• Oesophageal dysfunction You perform arthrocentesis. Which one of the following sets
• Respiratory: pleuritis, pulmonary hypertension, interstitial of characteristics would you expect the synovial fluid to
lung disease show?
• Haematological: anaemia, lymphadenopathy,
splenomegaly, rarely TTP A. Cloudy yellow colour, Culture negative, Calcium
• Cardiac: pericarditis, pericardial effusion, accelerated pyrophosphate crystals, White cell count: 15,000/mm 3
coronary artery disease B. Turbid grey colour, Weakly gram-negative ovoid shaped
bacteria, No crystals, White cell count: 90,000/mm3
• Renal: glomerulonephritis (tends to be milder than SLE)
C. Turbid grey colour, Gram-negative diplococci, No
• Neuropsychiatric: seizures, mood disturbance
crystals, White cell count: 90,000/mm3
D. Cloudy yellow colour, Culture negative, No crystals,
Investigations:
White cell count: 20,000/mm3
• Exclude other connective tissue disease/vasculitis
E. Cloudy yellow colour, Culture negative, Urate crystals,
• Bloods FBC: anaemia, leucopenia, thrombocytopenia,
White cell count: 20,000/mm3
U+E: renal impairment, CRP/ESR raised
• ANA (usually) positive, anti Ds-DNA and scleroderma- ANSWER:
specific antibodies (e.g. Anti-Scl70) are negative Cloudy yellow colour, Culture negative, No crystals, White cell
• Anti-U1 RNP (an extractable nuclear antigen, ENA), must count: 20,000/mm3
be positive*.
• Organ-specific investigations, e.g. ECG, echo, CT chest, EXPLANATION:
MRI brain Reactive arthritis: develops after an infection where the
organism cannot be recovered from the joint
Management: This gentleman is suffering from reactive arthritis and has
• No large-scale trials - patients have been included in trials presented with a typical triad of urethritis, conjunctivitis and
for SLE/SSc and show similar levels of response to arthritis. Patients who are HLA-B27 positive are predisposed
immunosuppression/DMARDs to recurrences of reactive arthritis and hence should be
• Calcium channel blockers may be used for the treatment advised to take particular care to avoid infections (e.g.
of Raynaud's Chlamydia) which cause these recurrences.
• Proton pump inhibitors for reflux disease
• Endothelin receptor antagonists/prostacyclin analogues Reactive arthritis usually occurs several weeks after an
in pulmonary hypertension infection, and the organism cannot be recovered from the
• Smoking cessation, moderate exercise joint on arthrocentesis. The turbid-grey-coloured joint
aspirates with organisms detected on culture indicate a
Prognosis: septic joint and are hence incorrect. Additionally, as reactive
• 1/3 long-term remission, 1/3 have chronic symptoms, 1/3 arthritis is not a crystal-arthritis, crystals will not be found in
develop severe systemic involvement and premature the joint aspirate making the other two options incorrect.
death.
REACTIVE ARTHRITIS
*Note that anti-U1 RNP antibodies are not completely specific Reactive arthritis is one of the HLA-B27 associated
and may also be seen in definite SSc and SLE seronegative spondyloarthropathies. It encompasses Reiter's
**Undifferentiated connective tissue disease refers to syndrome, a term which described a classic triad of urethritis,
syndromes in which features of one or more 'classical' conjunctivitis and arthritis following a dysenteric illness during
connective tissue disease may be present, but do not meet the Second World War. Later studies identified patients who
diagnostic criteria. Anti-U1 RNP is absent. developed symptoms following a sexually transmitted
infection (post-STI, now sometimes referred to as sexually
acquired reactive arthritis, SARA).
Reactive arthritis is defined as an arthritis that develops Which of the following factors will predispose her to
following an infection where the organism cannot be azathioprine toxicity?
recovered from the joint.
A. Cimetidine
'Can't see, pee or climb a tree' B. Rifampicin
C. Fast acetylator status
Epidemiology D. Thiopurine methyltransferase deficiency
• post-STI form much more common in men (e.g. 10:1) E. Alcohol excess
• post-dysenteric form equal sex incidence
ANSWER:
The table below shows the organisms that are most Thiopurine methyltransferase deficiency
commonly associated with reactive arthritis:
EXPLANATION:
Post-dysenteric form Post-STI form Azathioprine - check thiopurine methyltransferase deficiency
Shigella flexneri Chlamydia trachomatis (TPMT) before treatment
Salmonella typhimurium Thiopurine methyltransferase (TPMT) deficiency is present in
Salmonella enteritidis about 1 in 200 people and predisposes to azathioprine
Yersinia enterocolitica related pancytopaenia
Campylobacter

AZATHIOPRINE
Management Azathioprine is metabolised to the active compound
• symptomatic: analgesia, NSAIDS, intra-articular steroids mercaptopurine, a purine analogue that inhibits purine
• sulfasalazine and methotrexate are sometimes used for synthesis. A thiopurine methyltransferase (TPMT) test may be
persistent disease needed to look for individuals prone to azathioprine toxicity.
• symptoms rarely last more than 12 months
Adverse effects include
Q-51 • bone marrow depression
An 28-year-old man is investigated for recurrent lower back • nausea/vomiting
pain. A diagnosis of ankylosing spondylitis is suspected. • pancreatitis
Which one of the following investigations is most useful?
A significant interaction may occur with allopurinol and hence
A. ESR lower doses of azathioprine should be used.
B. X-ray of the sacro-iliac joints
C. HLA-B27 testing Q-53
D. X-ray of the thoracic spine A 57-year-old woman presents with a three month history of
E. CT of the lumbar spine right-sided hip pain. This seems to have come on
spontaneously without any obvious precipitating event. The
ANSWER: pain is described as being worse on the 'outside' of the hip
X-ray of the sacro-iliac joints and is particularly bad at night when she lies on the right
hand side.
EXPLANATION:
X-ray of the sacro-iliac joints is the most useful investigation On examination there is a full range of movement in the hip
for diagnosis and monitoring, but changes may not be seen including internal and external rotation. Deep palpation of
for many years after the onset of symptoms the lateral aspect of the right hip joint recreates the pain.

Please see Q-11 for Ankylosing Spondylitis: Investigation and An x-ray of the right hip is reported as follows:
Management
Right hip: Minor narrowing of the joint space otherwise
Q-52 normal appearance
A 31-year-old female intolerant of methotrexate is started
on azathioprine for rheumatoid arthritis. Routine blood What is the most likely diagnosis?
monitoring shows:
A. Fibromyalgia
Hb 7.9 g/dl B. Lumbar nerve root compression
Plt 97 * 109/l C. Osteoarthritis
WBC 2.7 * 109/l D. Greater trochanteric pain syndrome
E. Meralgia paraesthetica
ANSWER: Q-54
Greater trochanteric hip syndrome A 54-year-old male presents with abdominal pain, testicular
pain, weight loss, left foot drop and livedo reticularis. You
EXPLANATION: suspect a diagnosis of polyarteritis nodosa and request a full
Greater trochanteric pain syndrome is now the preferred autoantibody screen. From the options given below, what
term for trochanteric bursitis. are you most likely to see when the results of his
autoantibody screen returns if the diagnosis is polyarteritis
Whilst the x-ray shows joint space narrowing this is not an nodosa?
uncommon finding. Osteoarthritis would also be less likely
given the palpable nature of the pain and relatively short A. Positive anti-CCP
duration of symptoms. B. Positive rheumatoid factor
C. Anti-RNP
HIP PAIN IN ADULTS D. No autoantibody is likely to be positive
The table below provides a brief summary of the potential E. Anti-Smith
causes of hip pain in adults
ANSWER:
Condition Features No autoantibody is likely to be positive
Osteoarthritis Pain exacerbated by exercise and relieved by
rest EXPLANATION:
Reduction in internal rotation is often the first PAN can be difficult to diagnose because findings can be non-
sign specific. The diagnosis should be considered in patients with
Age, obesity and previous joint problems are various combinations of symptoms, such as unexplained
risk factors
fever, arthralgia, subcutaneous nodules, skin ulcers, pain in
Inflammatory arthritis Pain in the morning
the abdomen or extremities, new foot drop or wrist drop, or
Systemic features
Raised inflammatory markers rapidly developing hypertension. The diagnosis is further
Referred lumbar spine Femoral nerve compression may cause
clarified when clinical findings are combined with certain
pain referred pain in the hip laboratory results and other causes are excluded.
Femoral nerve stretch test may be positive -
lie the patient prone. Extend the hip joint Diagnosis of PAN is confirmed by biopsy showing necrotizing
with a straight leg then bend the knee. This arteritis or by arteriography showing the typical aneurysms
stretches the femoral nerve and will cause in medium-sized arteries. Magnetic resonance angiography
pain if it is trapped
may show microaneurysms, but some abnormalities may be
Greater trochanteric pain Due to repeated movement of the
too small for it to detect. Thus, magnetic resonance
syndrome (Trochanteric fibroelastic iliotibial band
bursitis) Pain and tenderness over the lateral side of
angiography is not the test used primarily for diagnosis.
thigh
Most common in women aged 50-70 years Biopsy of clinically uninvolved tissue is often useless because
Meralgia paraesthetica Caused by compression of lateral cutaneous the disease is focal; biopsy should target sites suggested by
nerve of thigh clinical evaluation. Samples of subcutaneous tissue, sural
Typically burning sensation over antero- nerve, and muscle, if thought to be involved, are preferred to
lateral aspect of thigh samples from the kidneys or liver; kidney and liver biopsies
Avascular necrosis Symptoms may be of gradual or sudden may be falsely negative because of sampling error and may
onset cause bleeding from unsuspected microaneurysms.
May follow high dose steroid therapy or
previous hip fracture of dislocation
Laboratory tests are nonspecific. Leukocytosis up to 20,000
Pubic symphysis Common in pregnancy
dysfunction Ligament laxity increases in response to to 40,000/L, proteinuria, and microscopic haematuria are the
hormonal changes of pregnancy most common abnormalities. Patients may have
Pain over the pubic symphysis with radiation thrombocytosis, markedly elevated ESR, anaemia caused by
to the groins and the medial aspects of the blood loss or renal failure, low serum albumin, and elevated
thighs. A waddling gait may be seen serum immunoglobulins. AST and ALT are often mildly
Transient idiopathic An uncommon condition sometimes seen in elevated. Testing for hepatitis B and C should be done.
osteoporosis the third trimester of pregnancy
Groin pain associated with a limited range of
Other testing such as pANCA, cANCA, rheumatoid factor,
movement in the hip
Patients may be unable to weight bear
anti-CCP antibody ANA, C3 and C4 complement levels,
ESR may be elevated cryoglobulin levels, Anti-dsDNA and antibodies to
extractable nuclear antigens such as anti-Smith, anti-Ro/SSA,
anti-La/SSB, and anti-RNP suggest other diagnoses, such as
RA, SLE, or Sjögren syndrome.
Whilst it should be noted that positive ANCA titres (usually of • may be idiopathic or associated with connective tissue
the p-ANCA type) are found in a low percentage (<20%) of disorders or underlying malignancy (typically ovarian,
patients with classic PAN and indeed are the most likely breast and lung cancer, found in 20-25% - more if patient
positive autoantibodies to be found, it does not form part of older)
the diagnostic criteria for the diagnosis and is neither • polymyositis is a variant of the disease where skin
sensitive or specific. It is merely associated and most patients manifestations are not prominent
are likely to have negative autoantibodies. Indeed, as
previously stated, a strongly positive titre should lead one to Skin features
suspect another diagnosis. • photosensitive
• macular rash over back and shoulder
Please see Q-34 for Polyarteritis Nodosa • heliotrope rash in the periorbital region
• Gottron's papules - roughened red papules over extensor
Q-55 surfaces of fingers
A 75-year-old female presents to her general practitioner • nail fold capillary dilatation
with muscle weakness that has become progressively worse
over the last year. She can no longer walk or stand for a long Other features
period of time. She also has a purple rash on her eyelids, red • proximal muscle weakness +/- tenderness
lumps on her knees and elbows and cracked painful skin on • Raynaud's
her fingers. What autoantibody is most likely to be causing
• respiratory muscle weakness
these symptoms?
• interstitial lung disease: e.g. Fibrosing alveolitis or
organising pneumonia
A. Anti-cyclic citrullinated peptide (anti-CCP)
• dysphagia, dysphonia
B. Anti-double-stranded DNA (anti-dsDNA)
C. Antinuclear (ANA)
Investigations
D. Anti-topoisomerase (anti-Scl-70)
• the majority of patients are ANA positive, with around
E. Anti-mitochondrial (AMA)
25% anti-Mi-2 positive
ANSWER:
Q-56
Antinuclear (ANA)
A 48- year-old patient presented to her family physician
complaining of pain all over her body for the past 4 months.
EXPLANATION:
She initially thought it was because she started taking
This patient has the inflammatory condition,
aerobic exercise classes 6 months ago. However, the pain
dermatomyositis. Dermatomyositis is associated with ANA. It
has been persistently bothering her and taking over-the-
presents with symptoms/signs such as proximal muscle
counter pain relievers did not help. The pain is more
weakness, a macular rash over back/shoulders, a violet
pronounced around her shoulders and lower back. She also
periorbital rash and red papules over extensor surfaces of
complains feeling tired and not being able to sleep because
the fingers.
of the pain. She has a body mass index of 28 kg per m2 and
Anti-CCP antibodies are associated with rheumatoid has never smoked in her life. She denies any weight loss or
arthritis. Rheumatoid arthritis typically presents with joint episodes of fever. She currently lives alone and is usually
pain. able to do her household chores. Her mother who passed
away last year suffered from rheumatoid arthritis. On
Anti-dsDNA antibodies are associated with systemic lupus examination, the doctor finds that the patient has tender
erythematosus (SLE). SLE generally presents with non-specific areas on her neck, elbow regions, and knees. Which is the
symptoms such as, fatigue, fever, oral ulcers, joint pain and a most likely diagnosis in this patient?
rash.
A. Polymyalgia rheumatica
Anti-Scl-70 antibodies are associated with diffuse systemic
B. Fibromyalgia
sclerosis. This would present with proximal limb or trunk
C. Polymyositis
scleroderma.
D. Ankylosing spondylitis
AMA is associated with primary biliary cirrhosis (PBC). PBC is E. Rheumatoid arthritis
often asymptomatic in the early stages but may present as
fatigue, pruritus and jaundice. ANSWER:
Fibromyalgia
DERMATOMYOSITIS
Overview EXPLANATION:
• an inflammatory disorder causing symmetrical, proximal This patient presented with a chronic pain described as all
muscle weakness and characteristic skin lesions over her body for the past 6 months. The pain did not appear
to be originating from the joints and therefore this makes a from the European League against Rheumatism (EULAR)
diagnosis of rheumatoid arthritis unlikely. The two most published in 2007 and also a BMJ review in 2014.
likely diagnoses are fibromyalgia and polymyalgia • explanation
rheumatica. However, polymyalgia rheumatica would have • aerobic exercise: has the strongest evidence base
also presented with weight loss and fever. This patient most • cognitive behavioural therapy
likely has fibromyalgia. She also feels tired and suffers from • medication: pregabalin, duloxetine, amitriptyline
sleep disturbances, which are both common complaints in
patients suffering from fibromyalgia. Q-57
Which one of the following is not a risk factor for developing
1: This is a musculoskeletal condition most common in osteoporosis?
patients above 50 years of age. It is associated with elevated
levels of inflammatory markers such as ESR and CRP and is A. Smoking
associated with giant cells arteritis. However, serum CK and B. Obesity
muscle biopsy are normal. C. Sedentary lifestyle
2: Fibromyalgia is characterized by a widespread D. Premature menopause
musculoskeletal pain and tender at several points in the E. Female sex
body.
3: There is no history of muscle weakness in this patient. ANSWER:
Muscle weakness in the shoulder region would have a made Obesity
a diagnosis of polymyositis likely.
4: This is an inflammatory musculoskeletal condition which EXPLANATION:
most affects the axial skeleton. It is strongly associated with Low body mass, rather than obesity is associated with an
HLA-B27 histocompatibility complex. The first presentation is increased risk of developing osteoporosis
usually a pain in the lower back region due to sacroiliitis.
5: Rheumatoid arthritis is a common painful inflammatory OSTEOPOROSIS: CAUSES
musculoskeletal condition affecting the joints. It is an Advancing age and female sex are significant risk factors for
autoimmune condition which usually affects the small joints osteoporosis. Prevalence of osteoporosis increases from 2% at
of the hands and feet in a symmetric distribution. Patients 50 years to more than 25% at 80 years in women.
often develop deformities such as ulnar deviation as the
disease progresses and articular structures are destroyed. There are many other risk factors and secondary causes of
osteoporosis. We'll start by looking at the most 'important'
FIBROMYALGIA ones - these are risk factors that are used by major risk
Fibromyalgia is a syndrome characterised by widespread pain assessment tools such as FRAX:
throughout the body with tender points at specific anatomical • history of glucocorticoid use
sites. The cause of fibromyalgia is unknown. • rheumatoid arthritis
• alcohol excess
Epidemiology • history of parental hip fracture
• women are around 5 times more likely to be affected • low body mass index
• typically presents between 30-50 years old • current smoking

Features Other risk factors


• chronic pain: at multiple site, sometimes 'pain all over' • sedentary lifestyle
• lethargy • premature menopause
• cognitive impairment: 'fibro fog' • Caucasians and Asians
• sleep disturbance, headaches, dizziness are common • endocrine disorders: hyperthyroidism, hypogonadism
(e.g. Turner's, testosterone deficiency), growth hormone
Diagnosis is clinical and sometimes refers to the American deficiency, hyperparathyroidism, diabetes mellitus
College of Rheumatology classification criteria which lists 9 • multiple myeloma, lymphoma
pairs of tender points on the body. If a patient is tender in at
• gastrointestinal disorders: inflammatory bowel disease,
least 11 of these 18 points it makes a diagnosis of fibromyalgia
malabsorption (e.g. Coeliac's), gastrectomy, liver disease
more likely
• chronic kidney disease
• osteogenesis imperfecta, homocystinuria
The management of fibromyalgia is often difficult and needs
to be tailored to the individual patient. A psychosocial and
Medications that may worsen osteoporosis (other than
multidisciplinary approach is helpful. Unfortunately there is
glucocorticoids):
currently a paucity of evidence and guidelines to guide
• SSRIs
practice. The following is partly based on consensus guidelines
• antiepileptics
• proton pump inhibitors A. Joint aspiration
• glitazones B. Start infliximab
• long term heparin therapy C. Oral high-dose prednisolone
• aromatase inhibitors e.g. anastrozole D. Short course of methotrexate
E. Depomederone injection
Investigations for secondary causes
If a patient is diagnosed with osteoporosis or has a fragility ANSWER:
fracture further investigations may be warranted. NOGG Joint aspiration
recommend testing for the following reasons:
• exclude diseases that mimic osteoporosis (e.g. EXPLANATION:
osteomalacia, myeloma); Joint aspiration is mandatory in all patients with a hot,
• identify the cause of osteoporosis and contributory swollen joint to rule out septic arthritis. If this was excluded
factors; in the above patient then intra-articular or system steroid
• assess the risk of subsequent fractures; therapy may be considered.
• select the most appropriate form of treatment
SEPTIC ARTHRITIS
The following investigations are recommended by NOGG: Overview
• History and physical examination • most common organism overall is Staphylococcus aureus
• Blood cell count, sedimentation rate or C-reactive • in young adults who are sexually active Neisseria
protein, serum calcium, gonorrhoeae should also be considered
• albumin, creatinine, phosphate, alkaline phosphatase and
liver transaminases The Kocher criteria for the diagnosis of septic arthritis:
• Thyroid function tests • fever >38.5 degrees C
• Bone densitometry ( DXA) • non-weight bearing
• raised ESR
Other procedures, if indicated • raised WCC
• Lateral radiographs of lumbar and thoracic spine/DXA-
based vertebral imaging Management
• Protein immunoelectrophoresis and urinary Bence-Jones • synovial fluid should be obtained before starting
proteins treatment
• 25OHD • intravenous antibiotics which cover Gram-positive cocci
• PTH are indicated. The BNF currently recommends
flucloxacillin or clindamycin if penicillin allergic
• Serum testosterone, SHBG, FSH, LH (in men),
• antibiotic treatment is normally be given for several
• Serum prolactin
weeks (BNF states 6-12 weeks)
• 24 hour urinary cortisol/dexamethasone suppression test
• needle aspiration should be used to decompress the joint
• Endomysial and/or tissue transglutaminase antibodies
• arthroscopic lavage may be required
(coeliac disease)
• Isotope bone scan
Q-59
• Markers of bone turnover, when available
What is the mode of action of bisphosphonates?
• Urinary calcium excretion
A. Promotes osteoblasts
So from the first list we should order the following bloods as a
B. Promotes calcium absorption
minimum for all patients:
C. Antagonist of PTH
• full blood count D. Inhibit osteoclasts
• urea and electrolytes E. Promotes phosphate excretion
• liver function tests
• bone profile ANSWER:
• CRP Inhibit osteoclasts
• thyroid function tests
EXPLANATION:
Q-58 Bisphosphonates inhibit osteoclasts
A 45-year-old female with a history of rheumatoid arthritis
presents to the Emergency Department with a two day Please see Q-25 for Bisphosphonates
history of a hot, painful, swollen right elbow joint. What is
the most appropriate management?
Q-60 • bony tenderness at the medial malleolar zone (from the
A 24-year-old nulliparous female with a history of recurrent tip of the medial malleolus to the lower 6 cm of the
deep vein thrombosis presents with shortness of breath. The posterior border of the tibia)
full blood count and clotting screen reveals the following • inability to walk four weight bearing steps immediately
results: after the injury and in the emergency department

Hb 12.4 g/dl There are also Ottawa rules available for both foot and knee
Plt 137 injuries
WBC 7.5 * 109/l
Q-62
PT 14 secs Which of the following statements is true regarding psoriatic
APTT 46 secs arthropathy?
What is the most likely underlying diagnosis?
A. Skin disease always precedes joint disease
A. Third generation oral contraceptive pill use B. Approximately one-third of patients with psoriasis
B. Protein C deficiency eventually develop arthropathy
C. Antithrombin III deficiency C. The mainstay of management is analgesia,
D. Antiphospholipid syndrome physiotherapy and joint replacement
E. Activated protein C resistance D. Males and females are equally affected
E. Arthritis mutilans is the most common subtype
ANSWER:
Antiphospholipid syndrome ANSWER:
Males and females are equally affected
EXPLANATION:
Antiphospholipid syndrome: (paradoxically) prolonged APTT EXPLANATION:
+ low platelets Males and females are affected equally by psoriatic arthritis
The combination of a prolonged APTT and thrombocytopenia Please see Q-42 for Psoriatic Arthropathy
make antiphospholipid syndrome the most likely diagnosis
Q-63
Please see Q-47 for Antiphospholipid Syndrome Reactive arthritis is associated with which one of the
following HLA antigens?
Q-61
A 23-year-old female presents with a painful ankle following A. HLA-B27
an inversion injury whilst playing tennis. Which one of the B. HLA-A3
following findings is least relevant when deciding whether C. HLA-DR4
an x-ray is needed? D. HLA-B5
E. HLA-DR3
A. Swelling immediately after the injury and now
ANSWER:
B. Pain in the malleolar zone
HLA-B27
C. Tenderness at the medial malleolar zone
D. Tenderness at the lateral malleolar zone EXPLANATION:
E. Cannot walk 4 steps immediately after the injury and Please see Q-50 for Reactive Arthritis
now
Q-64
ANSWER: A 50-year-old diabetic right-handed lady presents with left
Swelling immediately after the injury and now shoulder pain. She describes a stiff shoulder often more
painful at night and has difficulty dressing or doing up her
EXPLANATION: bra. On examination, there is no point tenderness and you
ANKLE INJURY: OTTAWA RULES notice weakness in external rotation.
The Ottawa Rules with for ankle x-rays have a sensitivity
What is the most likely cause of her shoulder pain?
approaching 100%
A. Acromioclavicular degeneration
An ankle x-ray is required only if there is any pain in the B. Subacromial impingement
malleolar zone and any one of the following findings: C. Rotator cuff tear
• bony tenderness at the lateral malleolar zone (from the D. Calcific tendinopathy
tip of the lateral malleolus to include the lower 6 cm of E. Adhesive capsulitis
posterior border of the fibular)
ANSWER: Management
Adhesive capsulitis • no single intervention has been shown to improve
outcome in the long-term
EXPLANATION: • treatment options include NSAIDs, physiotherapy, oral
External rotation is classically impaired in adhesive capsulitis corticosteroids and intra-articular corticosteroids
Adhesive capsulitis presents as a painful stiff shoulder with
restriction of active and passive range of motion in Q-65
abduction, internal and external rotation. However external A 78-year-old woman is discharged following a fractured
rotation often shows the most marked restriction and is the neck of femur. On review she is making good progress but
first movement to show impairment. The stem describes consideration is given to secondary prevention of further
difficulty dressing and doing up her bra as well as weakness fractures. What is the most suitable management?
of external rotation suggesting a globally impaired range of
motion. Patients often report difficulty sleeping on the A. Arrange DEXA scan + start strontium ranelate if T-score <
affected side. Other indications that the answer is adhesive -2.5 SD
capsulitis, include coexisting diabetes, female gender and B. Start oral bisphosphonate
symptoms in the non-dominant hand, all of which are C. Arrange DEXA scan + start oral bisphosphonate if T-score
common findings in this condition < -1.0 SD
D. Arrange DEXA scan + start hormone replacement
Acromioclavicular degeneration is often associated with therapy if T-score < -2.5 SD
popping, swelling, clicking or grindings and a positive scarf E. Arrange DEXA scan + start oral bisphosphonate if T-score
test not reported in the stem < -1.5 SD

Subacromial impingement patients often complain of pain ANSWER:


on overhead activities and demonstrate a painful arc of Start oral bisphosphonate
abduction on examination - worse between 90 and 120
degrees. There may also be popping, snapping or grinding. EXPLANATION:
NICE guidelines support starting bisphosphonates without
Rotator cuff tears can occur either due to specific trauma or waiting for a DEXA scan in such scenarios
chronic impingement. Patients will normally describe
weakness as well as pain and there may be muscle wasting OSTEOPOROSIS: MANAGEMENT
and tenderness on palpation. There may be a painful arc of NICE guidelines were updated in 2008 on the secondary
movement and weakness of the affected muscle. prevention of osteoporotic fractures in postmenopausal
women.
Patients with calcific tendinopathy would normally have
tenderness on palpation of the affected area and be Key points include
reluctant to move the arm. There may be overlap with • treatment is indicated following osteoporotic fragility
symptoms of impingement syndrome making this a less likely fractures in postmenopausal women who are confirmed
answer. to have osteoporosis (a T-score of - 2.5 SD or below). In
women aged 75 years or older, a DEXA scan may not be
ADHESIVE CAPSULITIS required 'if the responsible clinician considers it to be
Adhesive capsulitis (frozen shoulder) is a common cause of clinically inappropriate or unfeasible'
shoulder pain. It is most common in middle-aged females. The • vitamin D and calcium supplementation should be offered
aetiology of frozen shoulder is not fully understood. to all women unless the clinician is confident they have
adequate calcium intake and are vitamin D replete
Associations • alendronate is first-line
• diabetes mellitus: up to 20% of diabetics may have an • around 25% of patients cannot tolerate alendronate,
episode of frozen shoulder usually due to upper gastrointestinal problems. These
patients should be offered risedronate or etidronate (see
Features typically develop over days treatment criteria below)
• external rotation is affected more than internal rotation • strontium ranelate and raloxifene are recommended if
or abduction patients cannot tolerate bisphosphonates (see treatment
• both active and passive movement are affected criteria below)
• patients typically have a painful freezing phase, an
adhesive phase and a recovery phase Treatment criteria for patients not taking alendronate
• bilateral in up to 20% of patients Unfortunately, a number of complicated treatment cut-off
• the episode typically lasts between 6 months and 2 years tables have been produced in the latest guidelines for patients
who do not tolerate alendronate
These take into account a patients age, theire T-score and the • increased risk of cardiovascular events: any history of
number of risk factors they have from the following list: cardiovascular disease or significant risk of cardiovascular
• parental history of hip fracture disease is a contraindication
• alcohol intake of 4 or more units per day • increased risk of thromboembolic events: a Drug Safety
• rheumatoid arthritis Update in 2012 recommended it is not used in patients
with a history of venous thromboembolism
It is very unlikely that examiners would expect you to have • may cause serious skin reactions such as Stevens Johnson
memorised these risk tables so we've not included them in syndrome
the revision notes but they may be found by following the
NICE link. The most important thing to remember is: Denosumab
the T-score criteria for risedronate or etidronate are less than
• human monoclonal antibody that inhibits RANK ligand,
the others implying that these are the second line drugs
which in turn inhibits the maturation of osteoclasts
if alendronate, risedronate or etidronate cannot be taken
• given as a single subcutaneous injection every 6 months
then strontium ranelate or raloxifene may be given based on
• initial trial data suggests that it is effective and well
quite strict T-scores (e.g. a 60-year-old woman would need a
tolerated
T-score < -3.5)
the strictest criteria are for denosumab
Teriparatide
Supplementary notes on treatment • recombinant form of parathyroid hormone
Bisphosphonates • very effective at increasing bone mineral density but role
• alendronate, risedronate and etidronate are all licensed in the management of osteoporosis yet to be clearly
for the prevention and treatment of post-menopausal defined
and glucocorticoid-induced osteoporosis
• all three have been shown to reduce the risk of both Hormone replacement therapy
vertebral and non-vertebral fractures although • has been shown to reduce the incidence of vertebral
alendronate, risedronate may be superior to etidronate in fracture and non-vertebral fractures
preventing hip fractures • due to concerns about increased rates of cardiovascular
• ibandronate is a once-monthly oral bisphosphonate disease and breast cancer it is no longer recommended
for primary or secondary prevention of osteoporosis
Vitamin D and calcium unless the woman is suffering from vasomotor symptoms
• poor evidence base to suggest reduced fracture rates in
the general population at risk of osteoporotic fractures - Hip protectors
may reduce rates in frail, housebound patients • evidence to suggest significantly reduce hip fractures in
nursing home patients
Raloxifene - selective oestrogen receptor modulator (SERM) • compliance is a problem
• has been shown to prevent bone loss and to reduce the
risk of vertebral fractures, but has not yet been shown to Falls risk assessment
reduce the risk of non-vertebral fractures • no evidence to suggest reduced fracture rates
• has been shown to increase bone density in the spine and • however, do reduce rate of falls and should be considered
proximal femur in management of high risk patients
• may worsen menopausal symptoms
• increased risk of thromboembolic events
• may decrease risk of breast cancer

Strontium ranelate
• 'dual action bone agent' - increases deposition of new
bone by osteoblasts (promotes differentiation of pre-
osteoblast to osteoblast) and reduces the resorption of
bone by inhibiting osteoclasts
• concerns regarding the safety profile of strontium have
been raised recently. It should only be prescribed by a
specialist in secondary care
• due to these concerns the European Medicines Agency in
2014 said it should only be used by people for whom
there are no other treatments for osteoporosis
MRI showing osteoporotic fractures of the 8th and 10th thoracic vertebrae.
Q-66 • leukaemia
One of your patients who has a family history of Marfan's • type I cryoglobulinaemia, cold agglutinins
syndrome has recently been diagnosed with the condition. • use of vibrating tools
What is the most important investigation to monitor their • drugs: oral contraceptive pill, ergot
condition? • cervical rib

A. Urea and electrolytes Management


B. Echocardiography • first-line: calcium channel blockers e.g. nifedipine
C. Spirometry • IV prostacyclin infusions: effects may last several
D. Electrocardiogram weeks/months
E. DEXA scan
Q-68
ANSWER: A 34-year-old is diagnosed with chronic fatigue syndrome.
Echocardiography Which one of the following interventions is most useful?

EXPLANATION: A. Graded exercise therapy


Please see Q-31 for Marfan’s Syndrome B. Psychodynamic psychotherapy
C. Graded physiotherapy
Q-67 D. Advice to avoid alcohol and caffeine
A 47-year-old female is referred to the rheumatology clinic E. Low-dose fluoxetine
due to cold fingers. Which connective tissue disease is most
strongly associated with Raynaud's phenomenon? ANSWER:
Graded exercise therapy
A. Systemic lupus erythematous
B. Rheumatoid arthritis EXPLANATION:
C. Systemic sclerosis CHRONIC FATIGUE SYNDROME
D. Sjogren's syndrome Diagnosed after at least 4 months of disabling fatigue
E. Polyarteritis nodosa affecting mental and physical function more than 50% of the
time in the absence of other disease which may explain
ANSWER: symptoms
Systemic sclerosis
Epidemiology
EXPLANATION: • more common in females
Raynaud's phenomenon is associated with all the above • past psychiatric history has not been shown to be a risk
conditions but is most strongly linked to systemic sclerosis. factor
Around 2% of women and 6% of men with Raynaud's
phenomenon develop systemic sclerosis Fatigue is the central feature, other recognised features
include
RAYNAUD'S
• sleep problems, such as insomnia, hypersomnia,
Raynaud's phenomena may be primary (Raynaud's disease) or
unrefreshing sleep, a disturbed sleep-wake cycle
secondary (Raynaud's phenomenon)
• muscle and/or joint pains
• headaches
Raynaud's disease typically presents in young women (e.g. 30
• painful lymph nodes without enlargement
years old) with symmetrical attacks
• sore throat
Factors suggesting underlying connective tissue disease • cognitive dysfunction, such as difficulty thinking, inability
• onset after 40 years to concentrate, impairment of short-term memory, and
• unilateral symptoms difficulties with word-finding
• rashes • physical or mental exertion makes symptoms worse
• presence of autoantibodies • general malaise or 'flu-like' symptoms
• features which may suggest rheumatoid arthritis or SLE, • dizziness
for example arthritis or recurrent miscarriages • nausea
• digital ulcers, calcinosis • palpitations
• very rarely: chilblains
Investigation
Secondary causes • NICE guidelines suggest carrying out a large number of
• connective tissue disorders: scleroderma (most common), screening blood tests to exclude other pathology e.g. FBC,
rheumatoid arthritis, SLE
U&E, LFT, glucose, TFT, ESR, CRP, calcium, CK, ferritin*, • Serum 25OHD < 30 nmol/l: treatment recommended
coeliac screening and also urinalysis • Serum 25OHD 30-50 nmol/l: treatment is advised in
patients with: fragility fracture, osteoporosis, symptoms
Management suggestive of vitamin D deficiency, reduced exposure to
• cognitive behaviour therapy - very effective, number sunlight, raised PTH, conditions associated with
needed to treat = 2 malabsorption
• graded exercise therapy - a formal supervised program, • Serum 25OHD > 50 nmol/l: provide reassurance and give
not advice to go to the gym advice on maintaining adequate vitamin D levels
• 'pacing' - organising activities to avoid tiring through safe sunlight exposure and diet
• low-dose amitriptyline may be useful for poor sleep
VITAMIN D SUPPLEMENTATION
• referral to a pain management clinic if pain is a
Vitamin D supplementation has been a hot topic for a number
predominant feature
of years now. The muddied waters are now slightly clearer
following the release of the following:
Better prognosis in children
• 2012: letter by the Chief Medical Officer regarding
vitamin D supplementation
*children and young people only
• 2013: National Osteoporosis Society (NOS) release UK
Q-69 Vitamin D guideline
A 66-year-old female with COPD and a background of The following groups should be advised to take vitamin D
multiple previous courses of steroids presents to her GP for a supplementation:
routine bloods screen. She also has hypertension and angina. • all pregnant and breastfeeding women should take a daily
Her blood results are as follows: supplement containing 10µg of vitamin D
• all children aged 6 months - 5 years. Babies fed with
25-OH Vitamin D 16 nmol/l (>50) formula milk do not need to take a supplement if they are
Adjusted calcium 2.34 mmol/l taking more than 500ml of milk a day, as formula milk is
fortified with vitamin D
Her dietary intake of calcium is good. Which answer is most • adults > 65 years
appropriate in view of her past medical history and vitamin • 'people who are not exposed to much sun should also
D levels? take a daily supplement'
A. Give calcium and vitamin D combined Testing for vitamin D deficiency
B. Reassurance The key message is that not many people warrant a vitamin D
C. Repeat in 1 year and if vitamin D <10 commence test. The NOS guidelines specify that testing may be
treatment appropriate in the following situtations:
D. Commence oral bisphosphonate • patients with bone diseases that may be improved with
E. Oral vitamin D loading then maintenance vitamin D treatment e.g. known osteomalacia or Paget's
disease
ANSWER: • patients with bone diseases, prior to specific treatment
Oral vitamin D loading then maintenance where correcting vitamin deficiency is appropriate e,g,
prior to intravenous zolendronate or denosumab
EXPLANATION: • patients with musculoskeletal symptoms that could be
This patient has vitamin D deficiency and thus the correct attributed to vitamin D deficiency e.g. bone pain
answer is to load with vitamin D and then continue on ?osteomalacia
maintenance. In patients with good calcium intake and
normal serum calcium, giving oral calcium may actually be Patients with osteoporosis should always be given
detrimental. This is due to evidence suggesting adverse calcium/vitamin D supplements so testing is not considered
cardiovascular outcomes, which is thought to be related to necessary. People who are at higher risk of vitamin D
accelerated tissue and vascular calcification. Thus given her deficiency (see above) should be treated anyway so again
background of angina, this should be avoided. testing is not necessary.

Measurement of serum 25-OH vitamin D is the best way of Q-70


estimating vitamin D status. A 44-year-old woman presents with pain in her right hand
and forearm which has been getting worse for the past few
Optimal: > 75nmol/l weeks. There is no history of trauma. The pain is
Adequate: 50-75nmol/l concentrated around the thumb and index finger and is
Insufficiency: 30-49nmol/l often worse at night. Shaking her hand seems to provide
Deficiency: < 30nmol/l some relief. On examination there is weakness of the
abductor pollicis brevis and reduced sensation to fine touch Features - attacks typically last 1-3 days
at the index finger. What is the most likely diagnosis? • pyrexia
• abdominal pain (due to peritonitis)
A. C6 entrapment neuropathy • pleurisy
B. Thoracic outlet syndrome • pericarditis
C. Carpal tunnel syndrome • arthritis
D. Cervical rib • erysipeloid rash on lower limbs
E. Pancoast's tumour
ANSWER: Management
Carpal tunnel syndrome • colchicine may help

EXPLANATION: Q-73
More proximal symptoms would be expected with a C6 A 35-year-old woman who has severe Raynaud's disease is
entrapment neuropathy e.g. weakness of the biceps muscle reviewed in clinic. Three months ago she was started on
or reduced biceps reflex. nifedipine. Unfortunately this has had a minimal effect on
her symptoms and has resulted in ankle oedema. What is the
Patients with carpal tunnel syndrome often get relief from
most appropriate next step in management?
shaking their hands and this may be an important clue in
exam questions.
A. Aspirin
B. Sympathectomy
Please see Q-1 for Carpal Tunnel Syndrome
C. Intravenous prostacyclin
D. Methotrexate
Q-71
E. Intravenous infliximab
A 25-year-old man presents with back pain. Which one of the
following may suggest a diagnosis of ankylosing spondylitis? ANSWER:
Intravenous prostacyclin
A. Rapid onset
B. Gets worse following exercise EXPLANATION:
C. Bone tenderness Please see Q-67 for Raynaud’s
D. Pain at night Q-74
E. Improves with rest An 86-year-old woman presented to the emergency
department complaining of a 2-day history of difficulty
ANSWER: weight bearing and pain in the right knee. She is known to
Pain at night have atrial fibrillation, for which she is currently
anticoagulated with warfarin. Her INR 2 weeks ago was seen
EXPLANATION: to be 4.4, and she subsequently had her warfarin dose
Please see Q-3 for Ankylosis Spondylitis: Features adjusted. On examination, you note that her right knee is
red, hot and tender with a fluctuant effusion. What is the
Q-72 next most appropriate step?
Which one of the following is most useful in the
management of Familial Mediterranean Fever? A. Intravenous flucloxacillin
B. 5mg oral vitamin K
A. Prednisolone C. Commence colchicine
B. Erythromycin D. Aspirate the joint
C. Cyclophosphamide E. Discharge the patient with advice to rest, ice, compress
D. Colchicine and elevate the leg.
E. Benzylpenicillin
ANSWER:
ANSWER: Aspirate the joint
Colchicine EXPLANATION:
The red, hot, tender joint is an important presentation in
EXPLANATION: acute rheumatology. The three classic differentials for this
FAMILIAL MEDITERRANEAN FEVER presentation are septic arthritis, gout and pseudogout. In
Familial Mediterranean Fever (FMF, also known as recurrent this lady, in view of the previously high INR, joint
polyserositis) is an autosomal recessive disorder which haematoma is also an important consideration. It is not
typically presents by the second decade. It is more common in possible to differentiate these conditions clinically, so joint
people of Turkish, Armenian and Arabic descent aspiration is the most important next step. Culture will
enable a positive diagnosis of septic arthritis, whilst ANSWER:
microscopy and gross appearance the diagnosis of crystal Avascular necrosis of the femoral head
arthropathy.
EXPLANATION:
If there is a high clinical index of suspicion of septic arthritis, Initial x-rays are often normal in patients with avascular
the patient should be commenced on IV antibiotics. necrosis, but it would be unlikely that metastatic deposits
However, this should be done after joint aspiration so that significant enough to cause pain would not be shown.
more targetted therapy can be performed subsequently.
Please see Q-19 for Avascular Necrosis of the Hip
Warfarin reversal is not necessary in this case due to the low-
risk nature of joint aspiration and the fact that her previous Q-77
INR was only 4.4 with a subsequent reduction in dose. A 34-year-old man is reviewed in clinic. He has recently had
Colchicine is a useful medication in the acute management of his annual echocardiogram showing no change in the dilation
gout, but the diagnosis is yet to be established. Discharging of his aortic sinuses or mitral valve prolapse. You note he is
the patient at this point is premature. tall with pectus excavatum and arachnodactyly. His
condition is primarily due to a defect in which one of the
Please see Q-58 for Septic Arthritis following proteins?

Q-75 A. Polycystin-1
A 60-year-old man with a past medical history of B. Fibrillin
osteoarthritis presents with a swollen, red and hot left knee C. Type IV collagen
joint. He is unable to move it due to the pain. On D. Type I collagen
examination he is pyrexial with a temperature of 38.7 C and E. Elastin
a blood sample shows a white cell count of 22.8 *109/l.
Following joint aspiration, what is the most appropriate ANSWER:
antibiotic therapy? Fibrillin

A. IV flucloxacillin + benzylpenicillin EXPLANATION:


B. IV gentamicin + benzylpenicillin Although fibrillin is the primary protein affected (due to a
C. IV flucloxacillin defect in the fibrillin-1 gene) it should be noted that fibrillin
D. IV vancomycin + cefotaxime is used as a substrate of elastin
E. IV gentamicin + rifampicin + benzylpenicillin
Please see Q-31 for Marfan’s Syndrome
ANSWER:
IV flucloxacillin Q-78
You see a 28-year-old man with severe back pain. He has had
EXPLANATION: lumbar back pain for 2 years on and off but today the pain
Septic arthritis: IV flucloxacillin became much worse when he lent over to pick up a sock
yesterday. He complains of pain down the posterolateral
Please see Q-58 for Septic Arthritis side of his right leg. He also says when he touches this area it
feels numb. He says that his right leg feels weak.
Q-76
A 27-year-old woman is referred to orthopaedics. Three On examination, he has sensory loss of the posterolateral
years she had surgery and chemotherapy for thyroid cancer. aspect of his right leg and lateral aspect of his foot. He has
Follow up scans to date have shown no evidence of any weakness in plantar flexion of his right foot, a reduced ankle
disease recurrence. For the past two months she has been reflex on the right, and a positive sciatic nerve stretch test.
experiencing gradually increasing pain in her right hip which
is worse on exercising. On examination passive movement of You believe he has slipped a disc. Where is the sit of the
the hip is painful in all directions, especially internal compression?
rotation. An x-ray ordered by her GP has been reported as
normal. What is the most likely diagnosis? A. L2
B. L3
A. Trochanteric bursitis C. L4
B. Avascular necrosis of the femoral head D. L5
C. Primary hyperparathyroidism E. S1
D. Metastatic deposits
E. Hypoparathyroidism ANSWER:
S1
Investigation
EXPLANATION: • rheumatoid factor (RF) positive in nearly 100% of patients
S1 lesion features = Sensory loss of posterolateral aspect of • ANA positive in 70%
leg and lateral aspect of foot, weakness in plantar flexion of • anti-Ro (SSA) antibodies in 70% of patients with PSS
foot, reduced ankle reflex, positive sciatic nerve stretch test • anti-La (SSB) antibodies in 30% of patients with PSS
The correct answer here is an S1 lesion. An S1 lesion causes • Schirmer's test: filter paper near conjunctival sac to
sensory loss of the posterolateral aspect of the leg and measure tear formation
lateral aspect of the foot, weakness in plantar flexion of the • histology: focal lymphocytic infiltration
foot, reduced ankle reflexes, and a positive sciatic nerve • also: hypergammaglobulinaemia, low C4
stretch test.
Management
An L2 lesion would cause pain that radiates to the anterior • artificial saliva and tears
thigh but also would involve the posterior pelvis and possibly • pilocarpine may stimulate saliva production
the buttocks. Weakness would involve the Iliopsoas muscle,
so a patient might complain of weak hip flexion (eg when Q-80
climbing the stairs). This muscle also pulls the leg forward A 41-year-old man presents with persistent fatigue for the
when walking so weakness will reduce the length of the past 8 months. Which one of the following features is least
stride and cause a limp. Therefore, option 1 is wrong. consistent with a diagnosis of chronic fatigue syndrome?
Please see Q-2 for Lower Back Pain: Prolapsed Disc A. Dizziness
B. Painful lymph nodes without enlargement
Q-79 C. Having a busy day improves the symptoms
A 45-year-old woman with a history of primary Sjogren's D. Palpitations
syndrome is reviewed in clinic. Her main problem is a dry E. Headaches
mouth, which unfortunately has not responded to artificial
saliva. Which one of the following medications is most likely ANSWER:
to be beneficial? Having a busy day improves the symptoms
A. Rivastigmine EXPLANATION:
B. Neostigmine Physical or mental exertion usually makes the symptoms
C. Clonidine worse
D. Atropine
E. Pilocarpine Please see Q-68 for Chronic Fatigue Syndrome
ANSWER: Q-81
Pilocarpine An 18-year-old Armenian gentleman is admitted with
abdominal pains, fever and hip arthralgia. He has had
EXPLANATION: recurrent abdominal pains in the past but without a
SJOGREN'S SYNDROME confirmed diagnosis. A prior single port laparoscopy
Sjogren's syndrome is an autoimmune disorder affecting revealed no apparent deficit.
exocrine glands resulting in dry mucosal surfaces. It may be
primary (PSS) or secondary to rheumatoid arthritis or other On examination, you note the left lung base is stony dull to
connective tissue disorders, where it usually develops around percussion with a pericardial rub on auscultation of the
10 years after the initial onset. Sjogren's syndrome is much heart.
more common in females (ratio 9:1). There is a marked
increased risk of lymphoid malignancy (40-60 fold). What is the likely underlying diagnosis?
Features A. Familial Mediterranean Fever
• dry eyes: keratoconjunctivitis sicca B. Pancreatitis
• dry mouth C. Primary effusion lymphoma
• vaginal dryness D. Systemic lupus erythematosus
• arthralgia E. Castleman's disease
• Raynaud's, myalgia
• sensory polyneuropathy ANSWER:
• recurrent episodes of parotitis Familial Mediterranean Fever
• renal tubular acidosis (usually subclinical)
EXPLANATION:
The answer is Familial Mediterranean Fever which typically
presents in a Turkish, Armenian, and Arabic demographic, as
stated in the question, with a polyserositis.

Pancreatitis, primary effusion lymphoma, systemic lupus


erythematosus, Castleman's disease are all associated with
polyserositis. The lack of findings on laparoscopy and other
surgical findings weakens the diagnosis of pancreatitis. The
absence of lymphadenopathy makes primary effusion
lymphoma and Castleman's disease less likely. The stem
makes no mention of a rash, photosensitivity, renal Discoid lupus erythematous affecting the scalp
involvement, autoantibodies or pancytopaenia which makes
lupus much less likely, also. Q-83
Potential complications of Paget's disease include each of
Please see Q-72 for Familial Mediterranean Fever the following except:

Q-82 A. Deafness
Which one of the following statements concerning discoid B. Cerebral calcification
lupus is correct? C. Skull thickening
D. Bone sarcoma
A. Commonly progresses to SLE E. Fractures
B. Causes non-scarring alopecia
C. Characterised by follicular keratin plugs ANSWER:
D. Is rarely photosensitive Cerebral calcification
E. Typically presents in older males
EXPLANATION:
ANSWER: Please see Q-13 for Paget’s Disease of the Bone
Characterised by follicular keratin plugs
Q-84
EXPLANATION: A 44-year-old woman is seen in the rheumatology clinic. She
Discoid lupus erythematosus is characterised by follicular has been referred with Raynaud's phenomenon. During the
keratin plugs review of systems she mentions that her GP is organising an
endoscopy to investigate dyspepsia. On examination she is
DISCOID LUPUS ERYTHEMATOUS noted to have tight, shiny skin over her fingers. Which one of
Discoid lupus erythematosus is a benign disorder generally the following complications is she most likely to develop?
seen in younger females. It very rarely progresses to systemic
lupus erythematosus (in less than 5% of cases). Discoid lupus A. Bronchiectasis
erythematosus is characterised by follicular keratin plugs and B. Angiodysplasia
is thought to be autoimmune in aetiology C. Arterial hypertension
D. Chronic kidney disease
Features E. Pulmonary hypertension
• erythematous, raised rash, sometimes scaly
• may be photosensitive ANSWER:
• more common on face, neck, ears and scalp Pulmonary hypertension
• lesions heal with atrophy, scarring (may cause scarring
alopecia), and pigmentation EXPLANATION:
This patient is likely to have CREST syndrome. Unfortunately
Management pulmonary hypertension is one of the more common late
• topical steroid cream complications seen in such patients.
• oral antimalarials may be used second-line e.g.
hydroxychloroquine Please see Q-46 for Systemic Sclerosis
• avoid sun exposure
Q-85
A 58-year-old woman complains of aches and pains in her
bones. Her family have noticed she is generally weak and
lethargic. A series of blood tests are requested:
Calcium 2.04 mmol/l RHEUMATOID ARTHRITIS: MANAGEMENT
Albumin 39 g/l The management of rheumatoid arthritis (RA) has been
Phosphate 0.63 mmol/l revolutionised by the introduction of disease-modifying
Alkaline phosphatase 271 U/l therapies in the past decade.
Vitamin D3 15 nmol/l (75-200 nmol/l)
Parathyroid hormone 10.8 pmol/l (0.8 - 8.5 pmol/l) Patients with evidence of joint inflammation should start a
combination of disease-modifying drugs (DMARD) as soon as
What is the most appropriate management? possible. Other important treatment options include
analgesia, physiotherapy and surgery.
A. Arrange a liver ultrasound
B. Refer for a technetium-MIBI subtraction scan Initial therapy
C. Arrange a DEXA scan • In 2018 NICE updated their rheumatoid arthritis
D. Arrange serum electrophoresis and a skeletal survery guidelines. They now recommend DMARD monotherapy
E. Start vitamin D3 supplementation +/- a short-course of bridging prednisolone. In the past
dual DMARD therapy was advocated as the initial step.
ANSWER:
Start vitamin D3 supplementation DMARDs
• methotrexate is the most widely used DMARD.
EXPLANATION: Monitoring of FBC & LFTs is essential due to the risk of
The low calcium, phosphate and vitamin D levels combined myelosuppression and liver cirrhosis. Other important
with a raised alkaline phosphatase and parathyroid hormone side-effects include pneumonitis
level is entirely consistent with osteomalacia, or vitamin D • sulfasalazine
deficiency. The treatment of choice is therefore vitamin D3 • leflunomide
supplementation. • hydroxychloroquine

Remember blood values are normal in osteoporosis. TNF-inhibitors


Myeloma ('Arrange serum electrophoresis and a skeletal • the current indication for a TNF-inhibitor is an inadequate
survery') and primary hyperparathyroidism ('Refer for a response to at least two DMARDs including methotrexate
technetium-MIBI subtraction scan') are associated with • etanercept: recombinant human protein, acts as a decoy
hyper- rather than hypocalcaemia. receptor for TNF-α, subcutaneous administration, can
cause demyelination, risks include reactivation of
Please see Q-33 for Osteomalacia tuberculosis
• infliximab: monoclonal antibody, binds to TNF-α and
Q-86 prevents it from binding with TNF receptors, intravenous
A 24-year-old female is investigated for intermittent pain administration, risks include reactivation of tuberculosis
and swelling of the metacarpal phalangeal joints for the past • adalimumab: monoclonal antibody, subcutaneous
3 months. An x-ray shows loss of joint space and soft-tissue administration
swelling. Rheumatoid factor is positive and a diagnosis of
rheumatoid arthritis is made. What is the most appropriate Rituximab
management to slow disease progression?
• anti-CD20 monoclonal antibody, results in B-cell
depletion
A. Infliximab
• two 1g intravenous infusions are given two weeks apart
B. Rituximab
• infusion reactions are common
C. Methotrexate + infliximab + short-course of
prednisolone
Abatacept
D. Methotrexate + short-course of prednisolone
• fusion protein that modulates a key signal required for
E. Diclofenac
activation of T lymphocytes
ANSWER: • leads to decreased T-cell proliferation and cytokine
Methotrexate + short-course of prednisolone production
• given as an infusion
EXPLANATION: • not currently recommend by NICE
In 2018 NICE updated their rheumatoid arthritis guidelines.
They now recommend disease-modifying antirheumatic drug Q-87
(DMARD) monotherapy with a short-course of bridging A 30-year-old woman is diagnosed with systemic lupus
prednisolone. In the past dual DMARD therapy was erythematosus after presenting with lethargy, arthralgia and
advocated as the initial step. a facial rash. Her rheumatologists starts her on
hydroxychloroquine. Which one of the following is it most rheumatoid arthritis, allowing early detection of patients
important to monitor? suitable for aggressive anti-TNF therapy. It has a sensitivity
similar to rheumatoid factor (70-80%, see below) with a
A. Blood pressure much higher specificity of 90-95%.
B. Blood sugar
C. QT interval on ECG Please see Q-48 for Rheumatoid Arthritis: Antibodies
D. Visual acuity
E. Peak expiratory flow rate Q-89
A 45-year-old man presents with a painful swelling on the
ANSWER: posterior aspect of his elbow. There is no history of trauma.
Visual acuity On examination an erythematous tender swelling is noted.
What is the most likely diagnosis?
EXPLANATION:
Hydroxychloroquine - may result in a severe and permanent A. Synovial cyst
retinopathy B. Haemarthrosis
C. Septic arthritis
HYDROXYCHLOROQUINE D. Gout
Hydroxychloroquine is used in the management of E. Olecranon bursitis
rheumatoid arthritis and systemic/discoid lupus
erythematosus. It is pharmacologically very similar to ANSWER:
chloroquine which is used to treat certain types of malaria. Olecranon bursitis

Adverse effects EXPLANATION:


• bull's eye retinopathy - may result in severe and ELBOW PAIN
permanent visual loss The table below details some of the characteristic features of
• recent data suggest that retinopathy caused by conditions causing elbow pain:
hydroxychloroquine is more common than previously
thought and the most recent RCOphth guidelines (March Features
2018) suggest colour retinal photography and spectral
domain optical coherence tomography scanning of the
macula • pain and tenderness localised to the
lateral epicondyle
Monitoring • pain worse on resisted wrist extension
• the BNF advises: 'Ask patient about visual symptoms and with the elbow extended or supination
of the forearm with the elbow
monitor visual acuity annually using the standard reading
extended
chart'
• episodes typically last between 6
months and 2 years. Patients tend to
Q-88 have acute pain for 6-12 weeks
Lateral epicondylitis
The presence of anti-cyclic citrullinated peptide antibody is (tennis elbow)
suggestive of which one of the following conditions?
Medial epicondylitis Features
(golfer's elbow)
A. Systemic lupus erythematous
B. Rheumatoid arthritis
• pain and tenderness localised to the
C. Type 1 diabetes mellitus medial epicondyle
D. Addison's disease • pain is aggravated by wrist flexion and
E. Dermatomyositis pronation
• symptoms may be accompanied by
ANSWER: numbness / tingling in the 4th and 5th
Rheumatoid arthritis finger due to ulnar nerve involvement

EXPLANATION: Radial tunnel Most commonly due to compression of the


Anti-cyclic citrullinated peptide antibodies are associated syndrome posterior interosseous branch of the radial nerve. It
with rheumatoid arthritis is thought to be a result of overuse.

Features
Anti-cyclic citrullinated peptide antibody may be detectable
up to 10 years before the development of rheumatoid
arthritis. It may therefore play a key role in the future of
• symptoms are similar to lateral
Features Poor prognostic features
• rheumatoid factor positive
• poor functional status at presentation
• pain and tenderness localised to the • HLA DR4
lateral epicondyle • X-ray: early erosions (e.g. after < 2 years)
• pain worse on resisted wrist extension • extra articular features e.g. nodules
with the elbow extended or supination
of the forearm with the elbow
• insidious onset
extended • anti-CCP antibodies
• episodes typically last between 6
months and 2 years. Patients tend to In terms of gender there seems to be a split in what the
Lateral epicondylitis have acute pain for 6-12 weeks established sources state is associated with a poor prognosis.
(tennis elbow) However both the American College of Rheumatology and the
epicondylitis making it difficult to recent NICE guidelines (which looked at a huge number of
diagnose prognosis studies) seem to conclude that female gender is
• however, the pain tends to be around 4-5 associated with a poor prognosis.
cm distal to the lateral epicondyle
• symptoms may be worsened by Q-91
extending the elbow and pronating the A 54-year-old male presents with weakness of his upper
forearm
arms. On examination he is found to have a macular rash
over his back and the extensor aspects of his upper arms. He
Cubital tunnel Due to the compression of the ulnar nerve. is a heavy smoker and his sodium is 121 mmol/l. What is the
syndrome
most likely underlying diagnosis?
Features

A. Addison's disease
B. Polymyositis
• initially intermittent tingling in the 4th
and 5th finger
C. Overlap syndrome
• may be worse when the elbow is resting
D. Dermatomyositis
on a firm surface or flexed for extended E. Hypothyroidism
periods
• later numbness in the 4th and 5th finger ANSWER:
with associated weakness Dermatomyositis

Olecranon bursitis Swelling over the posterior aspect of the elbow. EXPLANATION:
There may be associated pain, warmth and This man may have an underlying small cell lung cancer
erythema. It typically affects middle-aged male causing Syndrome of Inappropriate Antidiuretic Hormone
patients. Secretion.

Q-90 Please see Q-55 for Dermatomyositis


Which of the following is associated with a good prognosis in
rheumatoid arthritis? Q-92
A 59-year-old man with a history of gout presents with a
A. Rheumatoid factor negative swollen and painful first metatarsophalangeal joint. He
B. HLA DR4 currently takes allopurinol 400mg od as gout prophylaxis.
C. Anti-CCP antibodies What should happen to his allopurinol therapy?
D. Rheumatoid nodules
E. Insidious onset A. Stop and recommence 4 weeks after acute inflammation
has settled
ANSWER: B. Reduce allopurinol to 100mg od until acute attack has
Rheumatoid factor negative settled
C. Stop and switch to colchicine prophylaxis
EXPLANATION: D. Stop and recommence 2 weeks after acute inflammation
RHEUMATOID ARTHRITIS: PROGNOSTIC FEATURES has settled
A number of features have been shown to predict a poor E. Continue allopurinol in current dose
prognosis in patients with rheumatoid arthritis, as listed
below ANSWER:
Continue allopurinol in current dose
EXPLANATION: EXPLANATION:
Please see Q-20 for Gout: Management This patient has features consistent with Marfan's syndrome.
Whilst glaucoma and retinal detachment are seen they are
Q-93 not as common as superotemporal ectopia lentis (upwards
You are doing the annual review of a 50-year-old woman lens dislocation), which is seen in around 50% of patients.
who has rheumatoid arthritis. Which one of the following Inferonasal ectopia lentis is characteristic of homocystinuria
complications is most likely to occur as a result of her
disease? Mitral valve prolapse may cause a late-systolic murmur. This
feature helps distinguish the above description from that of a
A. Chronic lymphocytic leukaemia patient with homocystinuria
B. Hypertension
C. Colorectal cancer Please see Q-31 for Marfan’s Syndrome
D. Type 2 diabetes mellitus
E. Ischaemic heart disease Q-95
A 34-year-old businessman presented to the hospital with a
ANSWER: low-grade fever, blood-streaked sputum and a dry cough for
Ischaemic heart disease 7 weeks. He had been travelling extensively in India, staying
in cheap and unsanitary accommodations. You decide to
EXPLANATION: send him for further testing which confirms a diagnosis of
Rheumatoid arthritis: patients have an increased risk of IHD tuberculosis. He is started on the correct antibiotics. Two
weeks later, he presents to your outpatient clinic with joint
RHEUMATOID ARTHRITIS: COMPLICATIONS pain, fatigue and a new rash. The rash is confined to his face
A wide variety of extra-articular complications occur in and shaped like a butterfly. You suspect drug-induced lupus,
patients with rheumatoid arthritis (RA): which one of these drugs is responsible for his condition?
• respiratory: pulmonary fibrosis, pleural effusion,
pulmonary nodules, bronchiolitis obliterans, A. Rifampicin
methotrexate pneumonitis, pleurisy B. Isoniazid
• ocular: keratoconjunctivitis sicca (most common), C. Ethambutol
episcleritis, scleritis, corneal ulceration, keratitis, steroid- D. Streptomycin
induced cataracts, chloroquine retinopathy E. Pyridoxine (Vitamin B6)
• osteoporosis
• ischaemic heart disease: RA carries a similar risk to type 2 ANSWER:
diabetes mellitus Isoniazid
• increased risk of infections
• depression EXPLANATION:
Isoniazid can cause drug-induced lupus
Less common
• Felty's syndrome (RA + splenomegaly + low white cell This patient has TB and is started on anti-TB drugs. There are
count) a few side-effect profiles which are specific to these drugs.
• amyloidosis
Rifampicin- this is a CYP450 enzyme inducer and can stain
Q-94 secretions such as sweat or urine an orange-red colour.
A 43-year-old who is noted to have a high-arched palate,
arachnodactyly and a late-systolic murmur presents with Isoniazid- this is a CYP450 enzyme inhibitor and can cause
visual problems. Which one of the following eye disorders is drug-induced lupus and peripheral neuropathy. Therefore
most associated with his underlying condition? pyridoxine is given which is a vitamin B6 and this reduces the
risk of neuropathies.
A. Superotemporal ectopia lentis
B. Inferonasal ectopia lentis Streptomycin- this is an aminoglycoside which is toxic to
C. Retinitis pigmentosa kidney and ears.
D. Acute glaucoma
E. Retinal detachment Pyridoxine- This is a supplement and does not cause drug-
induced lupus.
ANSWER:
Superotemporal ectopia lentis DRUG-INDUCED LUPUS
In drug-induced lupus not all the typical features of systemic
lupus erythematosus are seen, with renal and nervous system
involvement being unusual. It usually resolves on stopping the Q-97
drug. Which of the following is true regarding rheumatoid factor?

Features A. It is usually an IgM molecule reacting against patient's


• arthralgia own IgG
• myalgia B. High titres are not associated with severe disease
• skin (e.g. malar rash) and pulmonary involvement (e.g. C. Rose-Waaler test involves agglutination of IgG coated
pleurisy) are common latex particles
• ANA positive in 100%, dsDNA negative D. 80% of SLE patients are RF positive
• anti-histone antibodies are found in 80-90% E. 50% of patients with Sjogren's syndrome are RF positive
• anti-Ro, anti-Smith positive in around 5%
ANSWER:
It is usually an IgM molecule reacting against patient's own
IgG

EXPLANATION:
Rheumatoid factor is an IgM antibody against IgG

Please see Q-48 for Rheumatoid Arthritis: Antibodies

Q-98
A 73-year-old man presents pain in his right thigh. This has
been getting progressively worse for the past 9 months
A woman with drug-induced lupus despite being otherwise well. An x-ray is reported as follows:

Most common causes X-ray right femur Radiolucency of subarticular region


• procainamide suggestive of osteolysis. Some areas of patchy sclerosis
• hydralazine
Bloods tests show:
Less common causes
• isoniazid Calcium 2.38 mmol/l
• minocycline Phosphate 0.85 mmol/l
• phenytoin Alkaline phosphatase 544 u/L
Prostate specific antigen 4.4 ng/ml
Q-96
Low levels of which one of the following types of What is the most appropriate action?
complement are associated with the development of
systemic lupus erythematous? A. Vitamin D supplementation
B. Check serum testosterone
A. C4 C. Referral to an orthopaedic surgeon
B. C5 D. Referral to a urologist
C. C6 E. IV bisphosphonates
D. C7
E. C8 ANSWER:
IV bisphosphonates
ANSWER:
C4 EXPLANATION:
This patient has Paget's disease as evidenced by an isolated
EXPLANATION: rise in ALP and characteristic x-ray changes. As he has bone
SLE: C3 & C4 low pain he should be treated with bisphosphonates. A PSA of
4.4 ng/ml is probably normal in a 73-year-old man and is
Low levels of C4a and C4b have been shown to be associated certainly not consistent with metastatic prostate cancer.
with an increased risk of developing systemic lupus
erythematous Please see Q-13 for Paget’s Disease of the Bone

Please see Q-12 for Systemic Lupus Erythematosus:


Investigations
Q-99 Please see Q-90 for Rheumatoid Arthritis: Prognostic
A 57-year-old female has noticed that the skin on her hands Features
has become very tight and that her fingers sometimes turn
blue. She has also had difficulty swallowing both solids and Q-101
liquids. What autoantibody is most associated with these A 54-year-old man with a history of type 2 diabetes mellitus
symptoms? presents with a history of right shoulder pain. On
examination there is limited movement of the right shoulder
A. Anti-centromere in all directions. What is the most likely diagnosis?
B. Anti-topoisomerase (anti-Scl-70)
C. Anti-double-stranded DNA (anti-dsDNA) A. Adhesive capsulitis
D. Anti-cyclic citrullinated peptide (anti-CCP) B. Dermatomyositis
E. Anti-mitochondrial (AMA) C. Avascular necrosis
ANSWER: D. Lhermitte's syndrome
Anti-centromere E. Diabetic amyotrophy

EXPLANATION: ANSWER:
This female has limited cutaneous systemic sclerosis. This is Adhesive capsulitis
indicated as the scleroderma is limited to the distal
extremities only, however, the face may also be involved EXPLANATION:
also. Limited systemic sclerosis is associated with anti- Diabetic amyotrophy affects the lower limbs
centromere antibodies.
Please see Q-64 for Adhesive Capsulitis
Anti-Scl-70 antibodies are associated with diffuse systemic
sclerosis. In diffuse systemic sclerosis, scleroderma involves Q-102
the trunk and proximal limbs. Perinuclear antineutrophil cytoplasmic antibodies (pANCA)
are most strongly associated with which condition?
Anti-dsDNA antibodies are associated with systemic lupus
erythematosus (SLE). SLE generally presents with non-specific A. Goodpasture's syndrome
symptoms such as, fatigue, fever, oral ulcers, joint pain and a B. Churg-Strauss syndrome
rash. C. Polyarteritis nodosa
D. Granulomatosis with polyangiitis
Anti-CCP antibodies are associated with rheumatoid E. Autoimmune hepatitis
arthritis. Rheumatoid arthritis typically presents with joint
pain. ANSWER:
Churg-Strauss syndrome
AMA is associated with primary biliary cirrhosis (PBC). PBC is
often asymptomatic in the early stages but may present as EXPLANATION:
fatigue, pruritus and jaundice. cANCA = granulomatosis with polyangiitis; pANCA = Churg-
Strauss + others
Please see Q-46 for Systemic Sclerosis
Please see Q-9 for ANCA
Q-100
Which one of the following is most consistently associated Q-103
with a poor prognosis in rheumatoid arthritis? A 33-year-old man who is suspected of having ankylosing
spondylitis has a lumbar spine x-ray. Which one of the
A. Anti-CCP antibodies following features is most likely to be present?
B. HLA DR2 allele
C. Rapid onset A. Wedge shaped discs
D. Being a smoker B. Sclerosis
E. Female sex C. 'Rugger-Jersey' spine
D. Osteophytes
ANSWER: E. Subchondral cysts
Anti-CCP antibodies
ANSWER:
EXPLANATION: Sclerosis
See below for further information on the effect of gender on
prognosis.
EXPLANATION: Q-105
Ankylosing spondylitis - x-ray findings: subchondral erosions, Which one of the following statements is true regarding
sclerosis and squaring of lumbar vertebrae cytoplasmic anti-neutrophil cytoplasmic antibodies (cANCA)?
Please see Q-3 for Ankylosing Spondylitis: Features A. Targeted against myeloperoxidase
Q-104 B. Associated with granulomatosis with polyangiitis
A 72-year-old woman presents with a severe right-sided C. Can be used to monitor activity in autoimmune
headache and some blurring of vision that comes and goes. haemolytic anaemia
She has no significant past medical history. On examination, D. Is more commonly seen in ulcerative colitis than
there is tenderness on palpation of her right temporal perinuclear ANCA
region. Visual acuity is 6/36 in her right eye and 6/9 in her E. Positive in > 90% of hepatitis C associated vasculitis
left.
ANSWER:
Investigations: Associated with granulomatosis with polyangiitis
Erythrocyte sedimentation rate 154mm/hr (1-36)
C-reactive protein 116mg/L (<10) EXPLANATION:
cANCA = granulomatosis with polyangiitis; pANCA = Churg-
What drug should be given initially? Strauss + others
A. Cyclosporine (intravenous)
B. Leflunomide (oral) Please see Q-9 for ANCA
C. Methotrexate (oral)
D. Prednisolone (oral) Q-106
E. Methylprednisolone (intravenous) A 48-year-old female with left sided pins and needles in her
left thumb, index and middle finger for several months, has
ANSWER: been diagnosed with carpal tunnel syndrome. What features
Methylprednisolone (intravenous) in her past medical history has predisposed her to the
condition?
EXPLANATION:
This patient has temporal arteritis, also known as giant cell A. High blood pressure
arteritis (GCA). B. Prednisolone use
C. Obesity
The American College of Rheumatology 1990 criteria requires D. Hormone replacement therapy
3 of the following for GCA diagnosis: E. Previous contraceptive pill use
1. Age >50 y/o
2. New onset localised headache ANSWER:
3. Temporal artery tenderness or decreased pulsation Obesity
4. ESR >50mm/hr
5. Temporal artery biopsy positive EXPLANATION:
Obesity is a proven independent risk factor in those under 63
Uncomplicated GCA (no visual involvement and/or years. The others are not proven risk factors. ref: Bland JD.
jaw/tongue claudication) should be treated with oral The relationship of obesity, age, and carpal tunnel syndrome:
prednisolone 40-60mg daily until symptoms and more complex than was thought? Muscle Nerve. 2005
investigations normalise. Oct;32(4):527-32.

Complicated GCA (with visual involvement and/or Prednisolone use, hormone replacement therapy and using
jaw/tongue claudication), as in this scenario, should be given the contraceptive pill are not proven risk factors and no
IV methylprednisolone 500-1000mg for 3 days before connection has been found between them and increased risk
starting oral prednisolone. carpal tunnel syndrome.

As GCA requires long-term steroid therapy bone sparing Please see Q-1 for Carpal Tunnel Syndrome
agents (a bisphosphonate and vitamin D) and a
gastroprotective drug (e.g omeprazole) should be prescribed. Q-107
Also, low dose aspirin should be considered as it has been A 62-year-old man with lung cancer is suspected of having
shown to reduce the rate of visual loss and cerebrovascular dermatomyositis. Which one of the following antibodies is
accidents in GCA. most likely to be positive?

Please see Q-30 for Temporal Arteritis


A. Anti-nuclear antibodies that when he walks for more than a few minutes his legs
B. Anti-centromere bodies become weak and numb. If he sits down and leans forward
C. Anti-scl-70 antibodies the symptoms go. He finds standing exacerbates the
D. Anti-Jo-1 antibodies symptoms. He is worried as he is losing his independence as
E. Anti-Mi-2 antibodies he does not feel stable on his feet, he now has a walking aid.
His wife thinks he has a more stooped posture than 12
ANSWER: months ago.
Anti-nuclear antibodies
His past medical history includes tablet controlled
EXPLANATION: hypertension. He has never smoked and has a normal BMI.
Dermatomyositis antibodies: ANA most common, anti-Mi-2
most specific On examination, he has a wide-based gate. Neurological
examination of his lower limbs is normal. His peripheral
DERMATOMYOSITIS: INVESTIGATIONS AND MANAGEMENT pulses feel normal.
Investigations
• elevated creatine kinase What diagnosis is most likely given the presentation and
• EMG examination findings?
• muscle biopsy
• ANA positive in 60% A. Ankylosing spondylitis
• anti-Mi-2 antibodies are highly specific for B. Spinal stenosis
dermatomyositis, but are only seen in around 25% of C. Sciatica
patients D. Vascular claudication
• anti-Jo-1 antibodies are not commonly seen in E. Non-specific lower back pain
dermatomyositis - they are more common in polymyositis
where they are seen in a pattern of disease associated ANSWER:
with lung involvement, Raynaud's and fever Spinal stenosis

Management EXPLANATION:
• prednisolone Spinal stenosis is the most likely diagnosis in a patient with
gradual onset leg and back pain, weakness and numbness
Q-108 which is brought on by walking (with a normal clinical
A 54-year-old man is diagnosed as having gout. You are examination)
discussing ways to help prevent future attacks. Which one of
the following is most likely to precipitate an attack of gout? This man's presentation is most consistent with a diagnosis
of spinal stenosis. Spinal stenosis typically presents with
A. Chocolate gradual onset leg and back pain, weakness and numbness
B. Brazil nuts which is brought on by walking. Patients often say that the
C. Eggs pain is relieved by sitting and leaning forward and is better if
D. Sardines walking up a hill. Physical examination findings are
E. Smoking frequently normal in patients with lumbar spinal stenosis.
Therefore, the correct answer is 2.
ANSWER:
Sardines The main differential diagnosis is vascular claudication but
he has normal pulses peripherally. Therefore, option 4 is
EXPLANATION: wrong.
Foods to avoid include those high in purines e.g. Liver,
kidneys, seafood, oily fish (mackerel, sardines) and yeast Sciatica is more likely to present with unilateral leg pain.
products Therefore, option 3 is incorrect.

Please see Q-20 for Gout: Management Non-specific lower back pain is common but does not present
with lower limb symptoms. Therefore, option 5 is wrong.
Q-109
You see a 70-year-old male patient with back pain. He says LUMBAR SPINAL STENOSIS
he has had lower back pain for about 12 months which is Lumbar spinal stenosis is a condition in which the central
slowly getting worse. It now radiates to his buttocks, thighs canal is narrowed by tumour, disk prolapse or other similar
and legs bilaterally (but his left leg is worse than the right). degenerative changes.
He describes the pain as 'cramping' and 'burning'. He says
Patients may present with a combination of back pain, Please see Q-57 for Osteoporosis: Causes
neuropathic pain and symptoms mimicking claudication. One
of the main features that may help to differentiate it from Q-111
true claudication in the history is the positional element to the A 75-year-old male presents to the Emergency Department
pain. Sitting is better than standing and patients may find it with a month-long history of right-sided hearing loss and
easier to walk uphill rather than downhill. The neurogenic left-sided bony hip pain. Blood results show:
claudication type history makes lumbar spinal stenosis a likely
underlying diagnosis, the absence of such symptoms makes it ALP 440 IU/l
far less likely. Calcium 2.20 mmol/l
Phosphate 1.05 mmol/l
Pathology Total Protein 65 g/l
Degenerative disease is the commonest underlying cause.
Degeneration is believed to begin in the intervertebral disk Other results are unremarkable. What is the most
where biochemical changes such as cell death and loss of appropriate initial management step in the treatment of this
proteoglycan and water content lead to progressive disk condition?
bulging and collapse. This process leads to an increased stress
transfer to the posterior facet joints, which accelerates A. Risedronate
cartilaginous degeneration, hypertrophy, and osteophyte B. Teriparatide
formation; this is associated with thickening and distortion of C. Prednisolone
the ligamentum flavum. The combination of the ventral disk D. Vitamin D
bulging, osteophyte formation at the dorsal facet, and E. Calcitonin
ligamentum flavum hyptertrophy combine to
circumferentially narrow the spinal canal and the space ANSWER:
available for the neural elements. The compression of the Risedronate
nerve roots of the cauda equina leads to the characteristic
clinical signs and symptoms of lumbar spinal stenosis. EXPLANATION:
This question requires you first of all to diagnose the
Diagnosis condition and then identify the correct management.
MRI scanning is the best modality for demonstrating the canal
narrowing. Historically a bicycle test was used as true vascular The constellation of bony pain, unilateral hearing loss, and
claudicants could not complete the test. an isolated raised ALP should point you in the direction of
Paget's disease of the bone. In symptomatic Paget's disease,
Treatment treatment is with a nitrogen-containing bisphosphonate such
Laminectomy as alendronate, risedronate, or zoledronic acid. In patients
who cannot tolerate these, calcitonin is second-line therapy.
Q-110
A 50-year-old man with no past medical history is Unless contraindicated, all patients on bisphosphonates
investigated for ongoing back pain. He is found to have a should be given supplements of calcium and Vitamin D to
vertebral collapse secondary to osteoporosis. What is the avoid symptomatic hypocalcaemia.
most appropriate test to determine the cause of his
osteoporosis? There is no role for prednisolone in the treatment of Paget's
disease of the bone.
A. Thyroid function tests
B. Prostate specific antigen Teriparatide is a recombinant form of human parathyroid
C. Oestrogen level hormone (PTH) that may be used in the treatment of
D. Prolactin level osteoporosis.
E. Testosterone level
Please see Q-13 for Paget’s Disease of the Bone
ANSWER:
Testosterone level Q-112
A 38-year-old woman is reviewed. She has a diagnosis of
EXPLANATION: rheumatoid arthritis. She has recently been switched from
Osteoporosis in a man - check testosterone methotrexate to leflunomide. Monitoring of the full blood
count and liver function tests has been arranged. What else
Whilst thyrotoxicosis is a known cause of osteoporosis, should be monitored during treatment?
testosterone deficiency is much more likely in a middle-aged
male
A. Peak expiratory flow rate ANSWER:
B. Blood pressure Osteopaenia in vertebrae, osteoporosis in femoral neck
C. QT interval on ECG
D. Blood glucose EXPLANATION:
E. Urine for microscopic haematuria OSTEOPOROSIS: DEXA SCAN
Basics
ANSWER:
• T score: based on bone mass of young reference
Blood pressure
population
EXPLANATION: • T score of -1.0 means bone mass of one standard
Leflunomide may cause hypertension deviation below that of young reference population
• Z score is adjusted for age, gender and ethnic factors
LEFLUNOMIDE
Leflunomide is a disease modifying anti-rheumatic drug T score
(DMARD) mainly used in the management of rheumatoid • -1.0 = normal
arthritis. It has a very long half-life which should be • -1.0 to -2.5 = osteopaenia
remembered considering it's teratogenic potential. • < -2.5 = osteoporosis
Contraindications
• pregnancy - the BNF advises: 'Effective contraception Q-114
essential during treatment and for at least 2 years after A 43-year-old woman presents with pain in the right elbow.
treatment in women and at least 3 months after This has been present for the past month and she reports no
treatment in men (plasma concentration monitoring obvious trigger. On examination she reports pain on wrist
required' extension against resistance whilst the elbow is extended.
What is the most likely diagnosis?
• caution should also be exercised with pre-existing lung
and liver disease
A. Cubital tunnel syndrome
B. Lateral epicondylitis
Adverse effects
C. Carpal tunnel syndrome
• gastrointestinal, especially diarrhoea
D. Medial epicondylitis
• hypertension
E. Pronator syndrome
• weight loss/anorexia
• peripheral neuropathy ANSWER:
• myelosuppression Lateral epicondylitis
• pneumonitis
EXPLANATION:
Monitoring Please see Q-21 for Lateral Epicondylitis
• FBC/LFT and blood pressure
Q-115
Stopping Which one of the following is most likely to indicate an
• leflunomide has a very long wash-out period of up to a underlying connective tissue disorder in a patient with
year which requires co-administration of cholestyramine Raynaud's phenomenon?
Q-113
A. Chilblains
A 54-year-old woman who has had two Colle's fractures in
B. Bilateral symptoms
the past three years has a DEXA scan: C. Female patient
D. Onset at 18 years old
T-score
E. Recurrent miscarriages
L2-4 -1.4

Femoral neck -2.7


ANSWER:
Recurrent miscarriages
What does the scan show?
EXPLANATION:
A. Osteoporosis in both the vertebrae and femoral neck Raynaud's disease (i.e. primary) presents in young women
B. Osteoporosis in vertebrae, osteopaenia in femoral neck with bilateral symptoms
C. Osteopaenia in both the vertebrae and femoral neck
D. Osteopaenia in vertebrae, osteoporosis in femoral neck A history of recurrent miscarriages could indicate systemic
E. Normal bone density in vertebrae, osteoporosis in lupus erythematous or anti-phospholipid syndrome.
femoral neck Chilblains (pernio) are itchy, painful purple swellings which
occur on the fingers and toes after exposure to the cold. They • the 2008 NICE guidelines suggest it is not recommended
are occasionally associated with underlying connective tissue • a 2008 Drug and Therapeutics Bulletin review advised
disease but this is rare that whilst glucosamine provides modest pain relief in
knee osteoarthritis it should not be prescribed on the
Please see Q-67 for Raynaud’s NHS due to limited evidence of cost-effectiveness

Q-116 Q-117
A 58-year-old woman with a history of left hip osteoarthritis Which one of the following drugs has been associated with
presents for review. She is currently taking co-codamol an increased risk of atypical stress fractures of the proximal
30/500 for pain on a regular basis but this is unfortunately femoral shaft?
not controlling her symptoms. There is no past medical
history of note, in particular no asthma or gastrointestinal A. Spironolactone
problems. What is the most suitable next step in B. Alendronate
management? C. Quetiapine
D. Venlafaxine
A. Switch to regular oral tramadol E. Clopidogrel
B. Add topical ibuprofen
C. Add oral ibuprofen + proton pump inhibitor ANSWER:
D. Add oral etoricoxib Alendronate
E. Add oral diclofenac
EXPLANATION:
ANSWER: Please see Q-25 for Bisphosphonates
Add oral ibuprofen + proton pump inhibitor
Q-118
EXPLANATION: A 70-year-old woman presents with loss of vision in her left
NICE recommend co-prescribing a PPI with NSAIDs in all eye. For the past two weeks she has painful frontal
patients with osteoarthritis headaches and has been feeling generally lethargic. On
Topical NSAIDs are only indicated for osteoarthritis of the examination visual acuity is 6/9 in the right eye but on the
knee or hand. left side only hand movements can be made seen.
Fundoscopy of the left side reveals a pale and oedematous
OSTEOARTHRITIS: MANAGEMENT optic disc. What is the most likely diagnosis?.
NICE published guidelines on the management of
osteoarthritis (OA) in 2014 A. Acute angle closure glaucoma
• all patients should be offered help with weight loss, given B. Central retinal artery occlusion
advice about local muscle strengthening exercises and C. Multiple sclerosis
general aerobic fitness D. Methanol poisoning
• paracetamol and topical NSAIDs are first-line analgesics. E. Temporal arteritis
Topical NSAIDs are indicated only for OA of the knee or
hand ANSWER:
• second-line treatment is oral NSAIDs/COX-2 inhibitors, Temporal arteritis
opioids, capsaicin cream and intra-articular
corticosteroids. A proton pump inhibitor should be co- EXPLANATION:
prescribed with NSAIDs and COX-2 inhibitors. These drugs This patient has likely developed anterior ischemic optic
should be avoided if the patient takes aspirin neuropathy on the left side
• non-pharmacological treatment options include supports
and braces, TENS and shock absorbing insoles or shoes Please see Q-30 for Temporal Arteritis
• if conservative methods fail then refer for consideration
of joint replacement Q-119
Which one of the following features is least commonly seen
What is the role of glucosamine? in patients with pseudoxanthoma elasticum?
• normal constituent of glycosaminoglycans in cartilage and
synovial fluid A. Increased risk of ischaemic heart disease
• a systematic review of several double blind RCTs of B. Retinitis pigmentosa
glucosamine in knee osteoarthritis reported significant C. Gastrointestinal haemorrhage
short-term symptomatic benefits including significantly D. Autosomal recessive inheritance
reduced joint space narrowing and improved pain scores E. 'Plucked chicken skin' appearance
• more recent studies have however been mixed
ANSWER: ANSWER:
Retinitis pigmentosa Interstitial Lung Disease

EXPLANATION: EXPLANATION:
PSEUDOXANTHOMA ELASTICUM Polymyositis is a type of inflammatory myopathy related to
Pseudoxanthoma elasticum is an inherited condition (usually dermatomyositis and inclusion body myositis. The hallmark
autosomal recessive*) characterised by an abnormality in of polymyositis is weakness and/or loss of muscle mass in
elastic fibres the proximal musculature, and absent skin involvement. The
Features classic laboratory finding is a raised creatinine kinase (CK)
and the Anti-Jo-1 antibody.
• retinal angioid streaks
• 'plucked chicken skin' appearance - small yellow papules
Interstitial lung disease (ILD) has a major effect on morbidity
on the neck, antecubital fossa and axillae
and mortality in patients with polymyositis. ILD is considered
• cardiac: mitral valve prolapse, increased risk of ischaemic
to be a major risk factor for premature death in patients with
heart disease
myositis.
• gastrointestinal haemorrhage
The anti-Jo-1 antibody is a predictor of the presence of ILD at
diagnosis, with up to 70% of patients with the antibody
*there are reports of autosomal dominant inheritance in a
having concurrent ILD. The 5-year survival rate is between 60
minority of cases
and 80%.
Q-120
A 50-year-old woman complains of pain in her right elbow. POLYMYOSITIS
This has been present for the past four weeks and is maximal Overview
around 4-5cm distal from the lateral aspect of the elbow • inflammatory disorder causing symmetrical, proximal
joint. The pain is made worse by extending the elbow and muscle weakness
pronating the forearm. What is the most likely diagnosis? • thought to be a T-cellmediated cytotoxic process directed
against muscle fibres
A. Lateral epicondylitis • may be idiopathic or associated with connective tissue
B. Radial tunnel syndrome disorders
C. De Quervain's tenosynovitis • associated with malignancy
D. Cubital tunnel syndrome • dermatomyositis is a variant of the disease where skin
E. Medial epicondylitis manifestations are prominent, for example a purple
(heliotrope) rash on the cheeks and eyelids
ANSWER: • typically affects middle-aged, female:male 3:1
Radial tunnel syndrome
Features
EXPLANATION: • proximal muscle weakness +/- tenderness
Please see Q-5 for Elbow Pain • Raynaud's
• respiratory muscle weakness
Q-121 • interstitial lung disease: e.g. fibrosing alveolitis or
A 30-year-old woman presents with bilateral proximal organising pneumonia
muscle weakness. She is noted to have blanching of the • dysphagia, dysphonia
fingers, especially in cold weather.
Investigations
Anti-Jo-1 positive • elevated creatine kinase
ANA positive • other muscle enzymes (lactate dehydrogenase (LD),
CK 2000 U/L aldolase, AST and ALT) are also elevated in 85-95% of
ESR 60 mm/hr patients
EMG myopathic changes • EMG
• muscle biopsy
The presence of which of the following conveys the worst • anti-Jo-1 antibodies are seen in pattern of disease
prognosis? associated with lung involvement, Raynaud's and fever
A. Dysphagia
B. Skin rash Q-122
C. CK >2000 U/L Osteopetrosis is due to a defect in:
D. Interstitial lung disease
E. Raynaud's phenomenon
A. Osteoclast function Whilst the development of any new problem following the
B. PTH receptors introduction of a new drug warrants medical review it is
C. Osteoblast function particularly important to warn patients starting
D. Calcium resorption in proximal tubule bisphosphonates about symptoms which could suggest an
E. Calcium absorption oesophageal reaction, especially with alendronate

ANSWER: Please see Q-25 for Bisphosphonates


Osteoclast function
Q-125
EXPLANATION: A 19-year-old man presents with a spontaneous
OSTEOPETROSIS pneumothorax. He is noted to be of slim build with a tall
Overview stature and a high arched palate. His family who attends the
• also known as marble bone disease hospital with him have a similar appearance. You suspect
• rare disorder of defective osteoclast function resulting in Marfan's syndrome.
failure of normal bone resorption
• results in dense, thick bones that are prone to fracture This disease is caused by a mutation in the gene that
• bone pains and neuropathies are common. encodes which protein?
• calcium, phosphate and ALP are normal
• stem cell transplant and interferon-gamma have been A. Collagen
used for treatment B. Elastin
C. Fibrillin-1
Q-123 D. Tropoelastin
A 28-year-old woman who has rheumatoid arthritis is to be E. Actin
started on etanercept injections after failing to respond to
methotrexate and sulfsalazine. Which one of the following ANSWER:
adverse effects is associated with the use of etanercept? Fibrillin-1

A. Triggering Churg-Strauss syndrome EXPLANATION:


B. Thrombocytosis Marfan's syndrome is caused by a mutation in a protein
C. Tendonitis called fibrillin-1
D. Cardiomyopathy
E. Reactivation of tuberculosis Mutation of FBN1 that encodes Fibrillin-1 is responsible for
Marfan's syndrome.
ANSWER:
Reactivation of tuberculosis Please see Q-31 for Marfan’s Syndrome

EXPLANATION: Q-126
TNF-α inhibitors may reactivate TB A 76-year-old female presents with a 1 month history of left
sided temporal headaches and jaw claudication. Biopsy of
Please see Q-86 for Rheumatoid Arthritis: Management left temporal artery is negative.

Q-124 Hb 130 g/l


A 72-year-old female known to have osteoporosis is started Platelets 359 * 109/l
on alendronate. Which one of the following side-effects is it WBC 10 * 109/l
most important to warn her about? CRP 89 mg/l
A. Sore throat
B. Heartburn What is the next best step in management?
C. Headache
D. Diarrhoea A. Observation
E. Palpitations B. Commence prednisolone
C. Biopsy the right temporal artery
ANSWER: D. CT brain
Heartburn E. Ultrasound of left temporal artery

EXPLANATION: ANSWER:
Bisphosphonates can cause a variety of oesophageal Commence prednisolone
problems
EXPLANATION: ANSWER:
Jaw claudication is a very specific sign for temporal arteritis. Hydroxychloroquine is considered safe during pregnancy
A negative temporal artery biopsy can occur in up to 50
percent of patients, often because the sampled region was EXPLANATION:
not involved in the pathologic process. Therefore, it is not RHEUMATOID ARTHRITIS: PREGNANCY
sensitive enough to rule out temporal arteritis. Therefore, Rheumatoid arthritis (RA) typically develops in women of a
because vision is threatened, a brief course of steroids should reproductive age. Issues surrounding conception are
be initiated. therefore commonly encountered. There are no current
published guidelines regarding how patients considering
Please see Q-30 for Temporal Arteritis conception should be managed although expert reviews are
largely in agreement.
Q-127
A 25-year-old woman presents with a three day history of Key points
dysuria and a painful left knee. During the review of • patients with early or poorly controlled RA should be
symptoms she mentions a bout of diarrhoea and crampy advised to defer conception until their disease is more
abdominal pain three weeks ago. She is normally fit and well stable
and takes no regular medication. Her father died of • RA symptoms tend to improve in pregnancy but only
colorectal cancer in his sixth decade. On examination the left resolve in a small minority. Patients tend to have a flare
knee is red, swollen and hot to touch. What is the most likely following delivery
diagnosis? • methotrexate is not safe in pregnancy and needs to be
stopped at least 3 months before conception
A. Reactive arthritis secondary to Salmonella spp. • leflunomide is not safe in pregnancy
B. Reactive arthritis secondary to Chlamydia trachomatis • sulfasalazine and hydroxychloroquine are considered safe
C. Rheumatoid arthritis in pregnancy
D. Ulcerative colitis • interestingly studies looking at pregnancy outcomes in
E. Gonococcal arthritis patients treated with TNF-α blockers do not show any
significant increase in adverse outcomes. It should be
ANSWER: noted however that many of the patients included in the
Reactive arthritis secondary to Salmonella spp. study stopped taking TNF-α blockers when they found out
they were pregnant
EXPLANATION: • low-dose corticosteroids may be used in pregnancy to
Urethritis + arthritis + conjunctivitis = reactive arthritis control symptoms
Important for meLess important • NSAIDs may be used until 32 weeks but after this time
Two of the classic three features of reactive arthritis should be withdrawn due to the risk of early close of the
(urethritis, arthritis and conjunctivitis) are present in this ductus arteriosus
patient. The family history of colorectal cancer is of no • patients should be referred to an obstetric anaesthetist
particular significance. Symptoms of reactive arthritis due to the risk of atlanto-axial subluxation
typically appear 1-4 weeks following the initial infection, in
this case a diarrhoeal illness. Q-129
A 30-year-old man with a history of mitral valve prolapse,
Please see Q-50 for Reactive Arthritis recurrent pneumothorax, lower back pain secondary to
scoliosis and pectus excavatum is considering starting a
Q-128 family. Given the likely diagnosis, what is the mode of
A 28-year-old woman with rheumatoid arthritis asks for inheritance of this condition?
advice about conception. Which one of the following
statements is true? A. X-linked recessive
A. Methotrexate may be continued during pregnancy as B. Mitochondrial
long as the woman takes folic acid 5mg daily C. Autosomal codominant
B. NSAIDs should be avoided in the first and second D. Autosomal recessive
trimester E. Autosomal dominant
C. Woman with rheumatoid should be encouraged to
conceive as soon as possible (ideally within 1 year) after ANSWER:
diagnosis to minimise the risk of complications Autosomal dominant
D. TNF-α blockers are absolutely contraindicated in
pregnancy EXPLANATION:
E. Hydroxychloroquine is considered safe during pregnancy Please see Q-31 for Marfan’s Syndrome
Q-130 EXPLANATION:
A 45-year-old man who is known to have haemochromatosis Gout is one of the commonest forms of inflammatory
presents with a swollen and painful right knee. An x-ray arthritis. The prevalence appears to be rapidly increasing
shows no fracture but extensive chondrocalcinosis. Given worldwide. It is mediated by the crystallization of uric acid
the likely diagnosis, which one of the following is most likely within the joints. Urate crystals are deposited predominantly
to present in the joint fluid? in the superficial portions of the articular cartilage. These
characteristic cartilaginous deposits are not readily
A. Raised hyaluronic acid levels demonstrated with conventional diagnostic imaging
B. Monosodium urate crystals including CT or MRI.
C. Bipyramidal oxalate crystals
D. Negatively birefringent calcium carbonate crystals Imaging modalities such as plain XR, CT, MRI and bone
E. Positively birefringent rhomboid-shaped crystals scintigraphy can provide helpful diagnostic clues. However,
disadvantages include lack of specificity (bone scan, MRI),
ANSWER: considerable cost (MRI) and the inability to assess early soft
Positively birefringent rhomboid-shaped crystals tissue changes such as effusion, early erosions, synovial
hypertrophy and hypervascularity or small tophi
EXPLANATION: (roentgenography). Typical well-defined, punched out,
Pseudogout - positively birefringent rhomboid shaped periarticular erosions with overhanging edges are not seen
crystals radiographically until 6-12 yrs after the initial acute attack.
The most reliable method of diagnosis is invasive needle
Please see Q-35 for Pseudogout aspiration and identification of crystals on polarizing
microscopy. However, many physicians do not perform
Q-131 synovial fluid analysis, and therapy is often initiated with an
A 31-year-old woman who had rheumatoid arthritis assumed diagnosis.
diagnosed 5 years ago asks for advice as she is considering
starting a family. She currently has quiescent rheumatoid The most useful characteristic lesion is the double-contour
arthritis which is well controlled on methotrexate. Which sign a hyperechoic, irregular band over the superficial
one of the following drugs is it safest to use if she is planning margin of the joint cartilage, produced by deposition of
on becoming pregnant? monosodium urate crystals on the surface of the hyaline
cartilage, which increases the interface of the cartilage
A. Leflunamide surface, reaching a thickness similar to the subchondral
B. Sulfasalazine bone.
C. Methotraxate
D. Rituximab In contrast to gout, calcium pyrophosphate crystals tend to
E. Gold aggregate in the middle layer of the hyaline cartilage,
parallel to the bony cortex, as a hyperechoic, irregular line
ANSWER: embedded in the anechoic-appearing hyaline cartilage, with
Sulfasalazine a normal hyaline cartilage surface. Chondrocalcinosis can
thus be readily distinguished from gout.
EXPLANATION:
Please see Q-128 for Rheumatoid Arthritis: Pregnancy The double contour sign is not characteristically observed
with any of the other answers.
Q-132
A 71-year-old male undergoes an ultrasound scan for an GOUT: FEATURES
acutely painful right knee associated with swelling. An Gout is a form of inflammatory arthritis. Patients typically
ultrasound scan conforms the presence of an effusion and have episodes lasting several days when their gout flares and
states the 'double contour' sign was observed. What is the are often symptom-free between episodes. The acute
most likely diagnosis? episodes typically develop maximal intensity with 12 hours/
The main features it presents with are:
A. Jaccoud's arthropathy • pain: this is often very significant
B. Gout • swelling
C. Septic arthritis • erythema
D. Haemarthrosis
E. Osteoarthritis Around 70% of first presentations affect the 1st
metatarsophalangeal (MTP) joint. Attacks of gout affecting
ANSWER: this area were historically called podagra. Other commonly
Gout affected joints include:
• ankle Q-133
• wrist A 28-year-old man is diagnosed with having ankylosing
• knee spondylitis. He presented with a six month history of back
pain. On examination there is reduced lateral flexion of the
If untreated repeated acute episodes of gout can damage the spine but no evidence of any other complications. Which one
joints resulting in a more chronic joint problem. of the following is he most likely to offered as first-line
treatment?
Radiological features of gout include:
• joint effusion is an early sign A. Exercise regime + NSAIDs
• well-defined 'punched-out' erosions with sclerotic B. Exercise regime + infliximab
margins ina juxta-articular distribution, often with C. Physiotherapy + sulfasalazine
overhanging edges D. Physiotherapy + etanercept
• relative preservation of joint space until late disease E. Exercise regime + paracetamol
• eccentric erosions
• no periarticular osteopenia (in contrast to rheumatoid ANSWER:
arthritis) Exercise regime + NSAIDs
• soft tissue tophi may be seen
EXPLANATION:
The anti-TNF drugs are currently only used for patients with
severe ankylosing spondylitis which has failed to respond to
NSAIDs.

Please see Q-11 for Ankylosing Spondylitis: Investigation and


Management

Q-134
A 69-year-old man presents with an acute episode of gout on
his left first metatarsal-phalangeal joint. What is the most
likely underlying mechanism?

A. Sedentary lifestyle
B. Decreased renal excretion of uric acid
C. Increased endogenous production of uric acid
D. Starvation
E. Too much protein in diet
X ray of a patient with gout affecting his feet. It demonstrates juxta-articular
erosive changes around the 1st MTP joint with overhanging edges and associated ANSWER:
with a moderate soft tissue swelling. The joint space is maintained.
Decreased renal excretion of uric acid

EXPLANATION:
The vast majority of gout is due to decreased renal excretion
of uric acid
Important for meLess important
Decreased renal excretion of uric acid is thought to account
for 90% of cases of primary gout. Secondary risk factors such
as alcohol intake and medications should also be
investigated

GOUT: PREDISPOSING FACTORS


Gout is a form of microcrystal synovitis caused by the
deposition of monosodium urate monohydrate in the
synovium. It is caused by chronic hyperuricaemia (uric acid >
X-ray of a patient with gout affecting his hands. There are multiple periarticular
0.45 mmol/l)
erosions bilaterally with adjacent large soft tissue masses and relatively preserved
joint spaces. In the right hand, these findings are most prominent at the 1st Decreased excretion of uric acid
interphalangeal, 2nd-4th proximal interphalangeal, 1st-3rd metacarpophalangeal • drugs*: diuretics
and carpometacarpal joints. In the left hand, the findings are most prominent at
the ulnar styloid, scapholunate joint, first and fifth carpometacarpal joints, second
• chronic kidney disease
and fifth metacarpophalangeal joints and 1st interphalangeal joint. • lead toxicity
Increased production of uric acid Please see Q-116 for Osteoarthritis: Management
• myeloproliferative/lymphoproliferative disorder
Q-137
• cytotoxic drugs
Approximately what percentage of patients with psoriasis
• severe psoriasis develop an associated arthropathy?
Lesch-Nyhan syndrome
A. 0.5%
• hypoxanthine-guanine phosphoribosyl transferase
B. 30-40%
(HGPRTase) deficiency
C. 4-5%
• x-linked recessive therefore only seen in boys
D. 1%
• features: gout, renal failure, neurological deficits, learning
E. 10-20%
difficulties, self-mutilation
ANSWER:
*aspirin in a dose of 75-150mg is not thought to have a
10-20%
significant effect on plasma urate levels - the British Society
for Rheumatology recommend it should be continued if
EXPLANATION:
required for cardiovascular prophylaxis
Please see Q-42 for Psoriatic Arthropathy
Q-135
Q-138
A 31-year-old female with a history of SLE gives birth
A 33-year-old woman presents with back pain which radiates
following a 39 week pregnancy. The newborn is noted to be
down her right leg. This came on suddenly when she was
bradycardic. Which one of the following autoantibodies are
bending down to pick up her child. On examination straight
associated with congenital heart block?
leg raising is limited to 30 degrees on the right hand side due
to shooting pains down her leg. Sensation is reduced on the
A. Anti-Ro
dorsum of the right foot, particularly around the big toe and
B. Anti-Sm
foot dorsiflexion is also weak. The ankle and knee reflexes
C. Anti-RNP
appear intact. A diagnosis of disc prolapse is suspected.
D. Anti-dsDNA
Which nerve root is most likely to be affected?
E. Anti-Jo 1
A. L2
ANSWER:
B. L3
Anti-Ro
C. L4
D. L5
EXPLANATION:
E. S1
SLE - antibodies associated with congenital heart block =
anti-Ro
ANSWER:
L5
Please see Q-28 for Systemic Lupus Erythematosus:
Pregnancy
EXPLANATION:
L5 lesion features = loss of foot dorsiflexion + sensory loss
Q-136
dorsum of the foot
A 57-year-old man presents with pain in his right knee. An x-
ray shows osteoarthritis. He has no past medical history of
Please see Q-2 for Lower Back Pain: Prolapsed Disc
note. What is the most suitable treatment option for the
management of his pain?
Q-139
A 54-year-old female is reviewed in the rheumatology clinic
A. Oral diclofenac with omeprazole
due to dry eyes and arthralgia. A diagnosis of primary
B. Oral glucosamine
Sjogren's syndrome is suspected. Which one of the following
C. Oral diclofenac
features is least associated with this condition?
D. Oral ibuprofen
E. Oral paracetamol A. Renal tubular acidosis
B. Xerostomia
ANSWER: C. Sensory polyneuropathy
Oral paracetamol D. Dilated cardiomyopathy
E. Raynaud's phenomenon
EXPLANATION:
Osteoarthritis - paracetamol + topical NSAIDs (if knee/hand) ANSWER:
first-line Dilated cardiomyopathy
EXPLANATION: EXPLANATION:
Please see Q-79 for Sjogren’s Syndrome Hyperthyroidism is associated with an increased risk of
osteoporosis. Patients with hypothyroidism who take
Q-140 excessive amounts of levothyroxine may also be at risk of
Which one of the following features is least typical of osteoporosis
polymyalgia rheumatica?
Please see Q-57 for Osteoporosis: Causes
A. Elevated creatinine kinase
B. Low-grade fever Q-142
C. Morning stiffness in proximal limb muscles A 34-year-old woman with a history of antiphospholipid
D. Polyarthralgia syndrome presents with a swollen and painful leg. Doppler
E. Anorexia ultrasound confirms a deep vein thrombosis (DVT). She had a
previous DVT 4 months ago and was taking warfarin (with a
ANSWER: target INR of 2-3) when the DVT occurred. How should her
Elevated creatinine kinase anticoagulation be managed?

EXPLANATION: A. Life-long warfarin, increase target INR to 3 - 4


POLYMYALGIA RHEUMATICA B. Add in life-long low-dose aspirin
Pathophysiology C. A further 6 months warfarin, target INR 2 - 3
• overlaps with temporal arteritis D. A further 6 months warfarin, target INR 3 - 4
• histology shows vasculitis with giant cells, E. Life-long warfarin, target INR 2 - 3
characteristically 'skips' certain sections of affected artery
whilst damaging others ANSWER:
• muscle bed arteries affected most in polymyalgia Life-long warfarin, increase target INR to 3 - 4
rheumatica
EXPLANATION:
Features The evidence base is scanty here but most clinicians would
• typically patient > 60 years old increase the target INR to 3-4 if a patient has had a further
• usually rapid onset (e.g. < 1 month) thrombosis with an INR of 2-3. Please see the BCSH
• aching, morning stiffness in proximal limb muscles (not guidelines
weakness)
• also mild polyarthralgia, lethargy, depression, low-grade Please see Q-47 for Antiphospholipid Syndrome
fever, anorexia, night sweats
Q-143
Investigations A 70-year-old woman is reviewed. She sustained a fracture
• ESR > 40 mm/hr of her wrist one year ago, following which a DEXA scan was
• note CK and EMG normal performed. This showed a T-score of -2.8 SD. Calcium and
vitamin D supplementation was started along with oral
• reduced CD8+ T cells
alendronate. This however was stopped due to oesophagitis.
In accordance with NICE guidelines, what is the most suitable
Treatment
next management step?
• prednisolone e.g. 15mg/od - dramatic response
A. Start hormone replacement therapy
Q-141
B. Start raloxifene
Which one of the following is least recognised as a risk factor
C. Start teriparatide
for developing osteoporosis?
D. Switch to risedronate
E. Refer for hip protectors
A. Cushing's syndrome
B. Turner's syndrome
ANSWER:
C. Hyperparathyroidism
Switch to risedronate
D. Hypothyroidism
E. Diabetes mellitus
EXPLANATION:
The 2008 NICE guidelines suggest switching to risedronate or
ANSWER:
etidronate in patients unable to tolerate alendronate
Hypothyroidism
Please see Q-65 for Osteoporosis: Management
Q-144 ANSWER:
Patients with Sjogren's syndrome are at an increased risk of Flucloxacillin
which one of the following malignancies?
EXPLANATION:
A. Myeloma The diagnosis is septic arthritis. The most common bacteria
B. Oesophageal cancer being Staph.aureus. Patients with rheumatoid arthritis are
C. Intraocular melanoma particularly prone to septic arthritis. The rash was a heat
D. Squamous cell skin cancer rash and a red-herring. Flucloxacillin is the first line
E. Non-Hodgkin's lymphoma treatment for non-penicillin allergic septic arthritis.

ANSWER: Vancomycin is reserved for MRSA positive septic arthritis.


Non-Hodgkin's lymphoma
Ceftriaxone is used in gonococcal arthritis.
EXPLANATION:
Patients with Sjogren's syndrome have an increased risk of Ciprofloxacin and clarithromycin are not used in septic
lymphoid malignancies arthritis (unless microbiology specifically dictates based on
sensitivities).
Please see Q-79 for Sjogren’s Syndrome
Please see Q-58 for Septic Arthritis
Q-145
Looser's zones x-ray are most characteristically associated Q-147
with which one of the following conditions? Which of the following features are not typically seen in a
patient with adult onset Still's disease?
A. Primary hyperparathyroidism
B. Hypoparathyroidism A. Maculopapular rash
C. Osteomalacia B. Rheumatoid factor
D. Paget's disease C. Pyrexia
E. Osteoporosis D. High ferritin level
E. Lymphadenopathy
ANSWER:
Osteomalacia ANSWER:
Rheumatoid factor
EXPLANATION:
Please see Q-33 for Osteomalacia EXPLANATION:
Adult onset Still's disease is typically rheumatoid factor
Q-146 negative
You are working in the emergency department. A 30-year-
old lady with rheumatoid arthritis has been referred by her Please see Q-26 for Still’s Disease in Adults
GP with a two day history of worsening pain and swelling in
the left wrist. It has now become so bad in the last few hours Q-148
that she can barely move it. She returned from Thailand two A 27-year-old woman presents with painful genital
weeks ago where she was travelling with some friends. She ulceration. She has had recurrent attacks for the past four
had no problems with her wrist when travelling, but does years. Oral aciclovir has had little effect on the duration of
remember getting a rash on her chest and arms one day her symptoms. She has also noticed for the past year almost
when it was particularly hot. This only lasted a day. Her weekly attacks of mouth ulcers which again are slow to heal.
rheumatoid arthritis is usually well controlled on Her only past medical history of note is being treated for
methotrexate. She has no drug allergies. You aspirate the thrombophlebitis two years ago. What is the most likely
wrist which returns a turbid fluid and send it for gram stain. diagnosis?
Given the likely diagnosis which intravenous antibiotic do
you start her on? A. Behcet's syndrome
B. Polyarteritis nodosa
A. Vancomycin C. Systemic lupus erythematosus
B. Flucloxacillin D. Sarcoidosis
C. Ceftriaxone E. Herpes simplex virus type 2
D. Ciprofloxacin
E. Clarithromycin ANSWER:
Behcet's syndrome
EXPLANATION: May be acute or chronic
Please see Q-43 for Behcet’s Syndrome Pain worse in the morning and on standing
On examination there may be pain over the facets.
Facet joint The pain is typically worse on extension of the back
Q-149
A 65-year-old man presents with bilateral leg pain that is Ankylosing Typically a young man who presents with lower back
spondylitis pain and stiffness
brought on by walking. His past medical history includes
Stiffness is usually worse in morning and improves with
peptic ulcer disease and osteoarthritis. He can typically walk activity
for around 5 minutes before it develops. The pain subsides Peripheral arthritis (25%, more common if female)
when he sits down. He has also noticed that leaning Peripheral Pain on walking, relieved by rest
forwards or crouching improves the pain. Musculoskeletal arterial disease Absent or weak foot pulses and other signs of limb
and vascular examination of his lower limbs is ischaemia
unremarkable. What is the most likely diagnosis? Past history may include smoking and other vascular
diseases
A. Inflammatory arachnoiditis
B. Peripheral arterial disease Q-150
C. Raised intracranial pressure A 34-year-old kitchen worker presents with a two week
D. Spinal stenosis history of pain in her right wrist. She has recently emigrated
E. Lumbar vertebral crush fracture from Ghana and has no past medical history of note. On
examination she is tender over the base of her right thumb
ANSWER: and also over the radial styloid process. Ulnar deviation of
Spinal stenosis the wrist recreates the pain. What is the most likely
diagnosis?
EXPLANATION:
This is a classic presentation of spinal stenosis. Whilst A. Rheumatoid arthritis
peripheral arterial disease is an obvious differential the B. Osteoarthritis of the carpometacarpal joint
characteristic relieving factors of the pain and normal C. De Quervain's tenosynovitis
vascular examination point away from this diagnosis. D. Carpal tunnel syndrome
E. Systemic lupus erythematosus
LOWER BACK PAIN
ANSWER:
Lower back pain (LBP) is one of the most common De Quervain's tenosynovitis
presentations seen in practice. Whilst the majority of
presentations will be of a non-specific muscular nature it is EXPLANATION:
worth keeping in mind possible causes which may need DE QUERVAIN'S TENOSYNOVITIS
specific treatment. De Quervain's tenosynovitis is a common condition in which
the sheath containing the extensor pollicis brevis and
Red flags for lower back pain abductor pollicis longus tendons is inflamed. It typically affects
• age < 20 years or > 50 years females aged 30 - 50 years old
• history of previous malignancy
• night pain Features
• history of trauma • pain on the radial side of the wrist
• systemically unwell e.g. weight loss, fever • tenderness over the radial styloid process
• abduction of the thumb against resistance is painful
The table below indicates some specific causes of LBP: • Finkelstein's test: with the thumb is flexed across the
palm of the hand, pain is reproduced by movement of the
wrist into flexion and ulnar deviation
May be acute or chronic
Pain worse in the morning and on standing
On examination there may be pain over the facets. Management
Facet joint The pain is typically worse on extension of the back • analgesia
Spinal stenosis Usually gradual onset • steroid injection
Unilateral or bilateral leg pain (with or without back • immobilisation with a thumb splint (spica) may be
pain), numbness, and weakness which is worse on effective
walking. Resolves when sits down. Pain may be described
• surgical treatment is sometimes required
as 'aching', 'crawling'.
Relieved by sitting down, leaning forwards and crouching
down Q-151
Clinical examination is often normal Which one of the following features is least commonly seen
Requires MRI to confirm diagnosis in drug-induced lupus?
A. Glomerulonephritis evidence to date is that the radiographic progression of AS is
B. Arthralgia unaltered with the use of these agents. The only therapy
C. Myalgia showing promise for a disease modifying effect in this regard
D. Malar rash so far has been sustained use of NSAIDs.
E. Pleurisy
Please see Q-11 for Ankylosing Spondylitis: Investigation and
ANSWER: Management
Glomerulonephritis
Q-154
EXPLANATION: A 40-year-old woman who is known to have systemic lupus
Glomerulonephritis is unusual in drug-induced lupus erythematosus is reviewed with an exacerbation of wrist
pain. Which one of the following is the most useful marker
Please see Q-95 for Drug-Induced Lupus for monitoring disease activity?

Q-152 A. C-reactive protein


Which one of the following is least associated with the B. C2 levels
development of gout? C. Anti-nuclear antibody titres
D. Anti-dsDNA titres
A. Psoriasis E. Anti-Sm titres
B. Lesch-Nyhan syndrome
C. Lymphoma ANSWER:
D. Lithium toxicity Anti-dsDNA titres
E. Renal failure
EXPLANATION:
ANSWER: Please see Q-12 for Systemic Lupus Erythematosus:
Lithium toxicity Investigations

EXPLANATION: Q-155
Please see Q-134 for Gout: Predisposing Factors A 54-year-old man is recovering following his first episode of
gout. He still has some residual pain in the big toe. He has no
Q-153 risk factors for the development of gout and there is no
You are the ST1 working in the rheumatology out-patient evidence of gouty tophi on examination.
clinic. Your next patient is a 25-year-old man who was
diagnosed with ankylosing spondylitis (AS) 12 months ago. What is the most suitable point to start uric acid-lowering
Despite regular physiotherapy and trials of two different therapy?
non-steroidal anti-inflammatory drugs (NSAIDs) he remains
symptomatic and asks you about the potential benefits of A. Immediately
TNF-inhibitor therapy. You should tell him that TNF- B. Once his symptoms of gout have fully settled
inhibitors will improve all of the following except: C. 4 weeks after his symptoms of gout have fully settled
D. If one further attack of gout in the next 12 months
A. Quality of life E. If more than 4 episodes of gout in a 1 year period
B. Radiological progression
C. Spinal mobility ANSWER:
D. Extra-articular features Once his symptoms of gout have fully settled
E. Early morning stiffness
EXPLANATION:
ANSWER: Gout: offer allopurinol after first attack of gout has settled
Radiological Progression Important for meLess important
The 2017 British Society for Rheumatology guidelines no
EXPLANATION: longer advocate a delay before starting urate-lowering
Radiographic damage in AS is quantified by the number of therapy (ULT) but do give the following advice:
syndesmophytes, squaring, erosions and sclerosis developing
at vertebral corners. Quantified radiographic damage has Commencement of ULT is best delayed until inflammation
been shown to correlate well with spinal mobility and overall has settled as ULT is better discussed when the patient is not
physical function. However, unlike rheumatoid arthritis and in pain
psoriatic arthritis, where TNF-inhibitors have demonstrated
significant effect on progression of structural damage, the Please see Q-20 for Gout: Management
Q-156 density to individuals of a similar age. Abnormal Z-scores
A 35-year-old female is admitted to the ward following a fall confirm a diagnosis of secondary osteoporosis.
from standing position. She complains of severe pain in her
left groin. Her left leg appears to be shortened and FRAX and Q fracture are scoring systems used to quantify the
externally rotated. risk of osteoporotic fractures.

A hip X-ray is performed: Coeliac disease is a relatively common disease, and is a


known cause of malnutrition and secondary osteoporosis.
Left hip x-ray Intertrochanteric fracture of left femoral However, there are many other causes to consider.
neck Therefore, the best answer is to perform a Z-score. If this
confirms the diagnosis, then a full secondary osteoporosis
The patient is referred onto orthopaedics who perform a screen should be performed (e.g. check calcium, vitamin D,
hemiarthroplasty. thyroid function, hormonal levels, coeliac screen).

Following this injury you investigate with a DEXA scan: There is no indication to perform an MRI whole body.

Right hip T-score -3.1 OSTEOPOROSIS


Lumbar spine T-score -2.8 Risk factors
• Family history
What additional test is useful for confirming a diagnosis of • Female sex
secondary osteoporosis? • Increasing age
• Deficient diet
A. FRAX • Sedentary lifestyle
B. Q fracture • Smoking
C. Z-score • Premature menopause
D. MRI whole body • Low body weight
E. Coeliac screen • Caucasians and Asians
ANSWER: Diseases which predispose
Z-score
• Endocrine: glucocorticoid excess (e.g. Cushing's, steroid
therapy), hyperthyroidism, hypogonadism (e.g. Turner's,
EXPLANATION:
testosterone deficiency), growth hormone deficiency,
A Z-score is helpful in diagnosing secondary osteoporosis and
hyperparathyroidism, diabetes mellitus
should always be used for children, young adults, pre-
• Multiple myeloma, lymphoma
menopausal women and men under the age of 50
• Gastrointestinal problems: inflammatory bowel disease,
Important for meLess important
malabsorption (e.g. Coeliacs), gastrectomy, liver disease
This patient has suffered a fragility fracture. A fragility
• Rheumatoid arthritis
fracture is defined as a fracture resulting from any fall from a
standing height or less. The human body should be able to • Long term heparin therapy
sustain a fall from this height without a fracture unless there • Chronic renal failure
is an underlying cause that makes the bones fragile e.g. • Osteogenesis imperfecta, homocystinuria
osteoporosis. In a young female, we should be concerned
about an underlying disease of the bone. Q-157
A 60-year-old woman who has recently started treatment for
The T score less than -2.5 has confirmed a diagnosis of polymyalgia rheumatica presents with a five day history of
osteoporosis. The T-score is defined as the number of headaches and reduced vision on the right side since this
standard deviations above or below the mean for a healthy morning There is no eye pain but the there is a 'large, dark
30-year-old adult of the same sex and ethnicity as the shadow' covering the superior visual field on the right side.
patient. On examination she has a tender, palpable right temporal
artery. What is the most likely explanation for the reduced
A Z-score is helpful in diagnosing secondary osteoporosis and vision?
should always be used for children, young adults, pre-
menopausal women and men under the age of 50. A Z-score A. Anterior ischemic optic neuropathy
is similar to a T-score, however it is the number of standard B. Central retinal vein occlusion
deviations above or below the mean for the patient's age, C. Optic neuritis
sex and ethnicity. It is therefore comparing the patients bone D. Ophthalmic arteritis
E. Central retinal artery occlusion
Q-160
ANSWER: A 33-year-old female is admitted to the Emergency
Anterior ischemic optic neuropathy Department due to right-sided weakness. She has a past
history of deep vein thrombosis following the birth of her
EXPLANATION: daughter. The only other past history of note is two
Please see Q-30 for Temporal Arteritis miscarriages. A CT head confirms an ischaemic stroke in the
left middle cerebral artery territory. What is the likely
Q-158 finding on echocardiography?
A 72-year-old woman is reviewed in the osteoporosis clinic.
She had a hip fracture 5 years ago after which she was A. Normal
started on alendronate. This had to be stopped due to B. Dilated cardiomyopathy
persistent musculoskeletal pain. Risedronate also had to be C. Bicuspid aortic valve
stopped for similar reasons. Strontium ranelate was D. Atrial septal defect
therefore started but this was stopped following the E. Ventricular septal defect
development of a deep vein thrombosis. Her current T-score
is -4.1. It has therefore been decided to start a trial of ANSWER:
denosumab. What is the mechanism of action of this drug? Normal

A. Monoclonal antibody against osteoprotegerin EXPLANATION:


B. Selective oestrogen receptor modulator This is a typical MRCP question. On first sight this question
C. Inhibits RANK ligand, which in turn inhibits the appears to be pointing towards a paradoxical embolus.
maturation of osteoclasts However, given the history of miscarriages and DVT a
D. Promotes differentiation of pre-osteoblasts to diagnosis of antiphospholipid syndrome is more likely.
osteoblasts + inhibits osteoclasts
E. Pyrophosphate analog which inhibits osteoclast activity Please see Q-47 for Antiphospholipid Syndrome

ANSWER: Q-161
Inhibits RANK ligand, which in turn inhibits the maturation of Which one of the following statements regarding systemic
osteoclasts lupus erythematous is true?

EXPLANATION: A. It is linked with HLA A5


Please see Q-65 for Osteoporosis: Management B. Onset is typically between 20-40 years old
C. It is more common in Caucasians
Q-159 D. The female:male ratio is 3:1
In line with NICE guidelines on the secondary prevention of E. The incidence has decreased in the past 30 years
osteoporotic fractures in postmenopausal women, which
one of the following patients should not automatically be ANSWER:
started on treatment? Onset is typically between 20-40 years old

A. A 81-year-old woman who has had a fractured neck of EXPLANATION:


femur SYSTEMIC LUPUS ERYTHEMATOSUS
B. A 64-year-old women with a wedge fractures of her Epidemiology
thoracic spine. DEXA scan shows a T-score of -3.1 SD • much more common in females (F:M = 9:1)
C. A 55-year-old women who had a Colles fracture of the • more common in Afro-Caribbeans* and Asian
wrist. DEXA scan shows a T-score of -3.3 SD communities
D. A 64-year-old women with a BMI of 18 kg/m2. She has a • onset is usually 20-40 years
wedge fractures of her thoracic spine. DEXA scan shows • incidence has risen substantially during the past 50 years
a T-score of -2.7 SD (3 fold using American College of Rheumatology criteria)
E. A 71-year-old women who had a Colles fracture of the
wrist. DEXA scan shows a T-score of -2.1 SD Pathophysiology
• autoimmune disease
ANSWER: • associated with HLA B8, DR2, DR3
A 71-year-old women who had a Colles fracture of the wrist. • thought to be caused by immune system dysregulation
DEXA scan shows a T-score of -2.1 SD leading to immune complex formation
• immune complex deposition can affect any organ
EXPLANATION: including the skin, joints, kidneys and brain
Please see Q-65 for Osteoporosis: Management
*It is said the incidence in black Africans is much lower than in A. Rheumatoid arthritis
black Americans - the reasons for this are unclear B. Psoriatic arthritis
C. Gout
Q-162 D. Reactive arthritis
A 51-year-old male presents with an acute onset of swelling E. Gonococcal arthritis
and pain in his right knee. Aspiration shows negatively
birefringent crystals with no organisms seen. His pain fails to ANSWER:
settle with NSAIDs. What is the most appropriate next step
in his management?
EXPLANATION:
A. Repeat joint aspiration and intra-articular corticosteroid The rash on the soles is keratoderma blenorrhagica. His
B. Allopurinol reactive arthritis may be secondary to either gastrointestinal
C. IV flucloxacillin infection or Chlamydia.
D. Diuretics
E. Low dose methotrexate Please see Q-6 for Reactive Arthritis: Features

ANSWER: Q-165
Repeat joint aspiration and intra-articular corticosteroid A 55-year-old woman presents with a four week history of
shoulder pain. There has been no obvious precipitating
EXPLANATION: injury and no previous experience. The pain is worse on
Please see Q-20 for Gout: Management movement and there is a grating sensation if she moves the
arm too quickly. She also gets pain at night, particularly
Q-163 when she lies on the affected shoulder. On examination
A 68-year-old presents with a painful swollen left knee which there is no obvious erythema or swelling. Passive abduction
has failed to settle after a weeks rest. There is no history of is painful between 60 and 120 degrees. She is unable to
trauma. On examination he has a moderate sized effusion. A abduct the arm herself past 70-80 degrees. Flexion and
plain radiograph is reported as follows: extension are preserved. What is the most likely diagnosis?

Some loss of joint space A. Adhesive capsulitis (frozen shoulder)


Linear calcification of the articular cartilage B. Supraspinatus tendonitis
C. Acromioclavicular joint injury
What is the most likely diagnosis? D. Glenohumeral arthritis
E. Superior labral lesion
A. Pseudogout
B. Rheumatoid arthritis ANSWER:
C. Sarcoidosis Supraspinatus tendonitis
D. Gout
E. Osteoarthritis EXPLANATION:
This patient has a classic 'painful arc' which is a sign of
ANSWER: shoulder impingement, most commonly secondary to
Pseudogout supraspinatus tendonitis.

EXPLANATION: Rotator cuff muscles


This x-ray describes chondrocalcinosis. Non-specific changes
such as loss of joint space are common in this age group and SItS - small t for teres minor
pseudogout itself may cause osteoarthritic-like changes.
Supraspinatus
Please see Q-35 for Pseudogout Infraspinatus
teres minor
Q-164 Subscapularis
A 25-year-old man presents with a painful, swollen left knee.
He returned 4 weeks ago from a holiday in Spain. There is no Muscle Notes
history of trauma and he has had no knee problems Supraspinatus aBDucts arm before deltoid
previously. On examination he has a swollen, warm left knee Most commonly injured
with a full range of movement. His ankle joints are also Infraspinatus Rotates arm laterally
painful to move but there is no swelling. On the soles of both teres minor aDDucts & rotates arm laterally
feet you notice a waxy yellow rash. What is the most likely Subscapularis aDDuct & rotates arm medially
diagnosis?
Q-166 supination/pronation. Patients can also complain of hand
Which of the following is least likely to be associated with paraesthesia or aching at the wrist.
ankylosing spondylitis?
Cubital tunnel syndrome patients experience tingling and
A. Apical fibrosis numbness in the 4th and 5th finger.
B. Achilles tendonitis
C. Amyloidosis In olecranon bursitis, there would be presence of swelling
D. Achalasia over the posterior elbow.
E. Heart block
Cervical radiculopathy would cause a burning pain radiating
ANSWER: from shoulder to fingers. There may be reduced cervical
Achalasia range of motion or pain as clues in the history.

EXPLANATION: Please see Q-21 for Lateral Epicondylitis


Ankylosing spondylitis features - the 'A's
• Apical fibrosis Q-168
• Anterior uveitis Which one of the following best describes rheumatoid
• Aortic regurgitation factor?
• Achilles tendonitis
• AV node block A. IgG against the Fc portion of IgM
• Amyloidosis B. IgM against the Fc portion of IgA
C. IgM against the Fc portion of IgM
Please see Q-3 for Ankylosis Spondylitis: Features D. IgM against the Fc portion of IgG
E. IgG against the Fc portion of IgA
Q-167
A 22-year-old gymnast reports pain in the lateral forearm ANSWER:
that is worse on straightening the wrist or fingers. She also IgM against the Fc portion of IgG
sometimes feels an unusual sensation in her hand a bit like
pins and needles. On examination, she is tender distal to the EXPLANATION:
common extensor origin with no pain overlying the lateral Rheumatoid factor is an IgM antibody against IgG
epicondyle itself.
Please see Q-48 for Rheumatoid Arthritis: Antibodies
What is the most likely cause of her forearm pain?
Q-169
A. Lateral epicondylitis A 56-year-old lady is referred to rheumatology clinic due to
B. Radial tunnel syndrome severe Raynaud's phenomenon associated with arthralgia of
C. Cubital tunnel syndrome the fingers. On examination you note shiny and tight skin of
D. Olecranon bursitis the fingers with a number of telangiectasia on the upper
E. Cervical radiculopathy torso and face. She is also currently awaiting a gastroscopy
to investigate heartburn. Which one of the following
ANSWER: antibodies is most specific for the underlying condition?
Radial tunnel syndrome
A. Anti-Jo 1antiobodies
EXPLANATION: B. Rheumatoid factor
Radial tunnel syndrome presents similarly to lateral C. Anti-Scl-70 antibodies
epicondylitis however pain is typically distal to the D. Anti-centromere antibodies
epicondyle and worse on elbow extension/forearm E. Anti-nuclear factor
pronation
Important for meLess important ANSWER:
Radial tunnel syndrome can be difficult to distinguish from Anti-centromere antibodies
lateral epicondylitis as both conditions present with lateral
elbow pain. Radial tunnel syndrome however typically EXPLANATION:
presents with tenderness distal to the common extensor Limited (central) systemic sclerosis = anti-centromere
origin in comparison to lateral epicondylitis where there is antibodies
pain over the common extensor origin. It is most common in
gymnasts, racquet players and golfers who frequently This lady has some features of CREST syndrome. Although
hyperextend at the wrist or carry out frequent ANA is positive in 90% of patients with systemic sclerosis,
anti-centromere antibodies are the most specific test for
limited cutaneous systemic sclerosis Na+ 138 mmol/l
K+ 4.0 mmol/l
Please see Q-46 for Systemic Sclerosis Urea 3.4 mmol/l
Creatinine 77 µmol/l
Q-170
A 50-year-old female with a history of rheumatoid presents Free T4 12.2 pmol/l
with a suspected septic knee joint. A diagnostic aspiration is TSH 1.25 mu/l
performed and sent to microbiology. Which of the following CRP 9 mg/l
organisms is most likely to be responsible?
What is the most likely diagnosis?
A. Staphylococcus aureus
B. Staphylococcus epidermidis A. Systemic lupus erythematosus
C. Escherichia coli B. Acne rosacea
D. Neisseria gonorrhoeae C. Fibromyalgia
E. Streptococcus pneumoniae D. Lyme Disease
E. HIV seroconversion illness
ANSWER:
Staphylococcus aureus ANSWER:
Systemic lupus erythematosus
EXPLANATION:
Septic arthritis - most common organism: Staphylococcus EXPLANATION:
aureus The malar rash, arthralgia, lethargy and history of mental
health points towards a diagnosis of SLE. Remember that the
Please see Q-58 for Septic Arthritis CRP (in contrast to the ESR) is typically normal in SLE.

Q-171 SYSTEMIC LUPUS ERYTHEMATOSUS: FEATURES


Which one of the following statements regarding raloxifene Systemic lupus erythematosus (SLE) is a multisystem,
in the management of osteoporosis is incorrect? autoimmune disorder. It typically presents in early adulthood
and is more common in women and people of Afro-Caribbean
A. Has been shown to prevent bone loss and to reduce the origin.
risk of vertebral fractures
B. Is a selective oestrogen receptor modulator General features
C. May worsen menopausal symptoms • fatigue
D. Increases risk of thromboembolic events • fever
E. Increases the risk of breast cancer • mouth ulcers
• lymphadenopathy
ANSWER:
Increases the risk of breast cancer Skin
• malar (butterfly) rash: spares nasolabial folds
EXPLANATION: • discoid rash: scaly, erythematous, well demarcated rash
Raloxifene may actually decrease the risk of breast cancer in sun-exposed areas. Lesions may progress to become
pigmented and hyperkeratotic before becoming atrophic
Please see Q-65 for Osteoporosis: Management • photosensitivity
• Raynaud's phenomenon
Q-172 • livedo reticularis
A 25-year-old woman presents for review. She has a history
• non-scarring alopecia
of depression and is currently prescribed citalopram. Despite
returning from a recent holiday in Spain she complains of
Musculoskeletal
feeling tired all the time. On examination you notice a
• arthralgia
slightly raised red rash on the bridge of her nose and cheeks.
• non-erosive arthritis
Although she complains of having 'stiff joints' you can find
no evidence of arthritis. You order some basic blood tests:
Cardiovascular
• pericarditis: the most common cardiac manifestation
Hb 12.7 g/dl
Platelets 130 * 109/l • myocarditis
WBC 3.3 * 109/l
Respiratory A. Osteoarthritis
• pleurisy B. De Quervain's tenosynovitis
• fibrosing alveolitis C. Gout
D. Rheumatoid arthritis
Renal E. Primary hyperparathyroidism
• proteinuria
• glomerulonephritis (diffuse proliferative ANSWER:
glomerulonephritis is the most common type) Osteoarthritis

Neuropsychiatric EXPLANATION:
• anxiety and depression The trapeziometacarpal joint (base of thumb) is the most
• psychosis common site of hand osteoarthritis.
• seizures
Please see Q-116 for Osteoarthritis: Management
Q-173
Which one of the following is most recognised as a risk factor Q-176
for developing osteoporosis? A 36-year-old female presents with her second unprovoked
pulmonary embolus 3 months after ceasing warfarin after
A. Osteogenesis imperfecta her first pulmonary embolus. She has a series of tests done
B. Marfan's syndrome to evaluate for the presence of thrombophilia, in particular
C. Myotonic dystrophy anti-phospholipid syndrome. What is the greatest predictor
D. Duchenne muscular dystrophy of future thrombosis in patients with anti-phospholipid
E. Ehler-Danlos syndrome syndrome?

ANSWER: A. Lupus anticoagulant


Osteogenesis imperfecta B. Anticardiolipin antibody
C. Beta-2-glycoprotein-1 antibody
EXPLANATION: D. Anti-prothrombin antibody
Please see Q-57 for Osteoporosis: Causes E. Annexin antibody
Q-174
ANSWER:
A 31-year-old woman presents as her fingers intermittently
Lupus anticoagulant
turn white and become painful. She describes the fingers
first turning white, then blue and finally red. This is generally
EXPLANATION:
worse in the winter months but it is present all year round.
Lupus anticoagulant is the strongest risk factor for
Wearing gloves does not help. Clinical examination of her
thrombosis of all the listed options. Lupus anticoagulants
hands, other joints and skin is unremarkable. Which one of
have an odds ratio for thrombosis 5 to 16 times higher than
the following treatments may be beneficial?
controls. Anticardiolipin antibodies, beta-2-glycoprotein 1
A. Amitriptyline antibodies and the other options listed, are not considered as
B. Aspirin prothrombotic as lupus anticoagulant.
C. Pregabalin Please see Q-47 for Antiphospholipid Syndrome
D. Propranolol
E. Nifedipine Q-177
An 80-year-old woman is started on oral alendronate
ANSWER: following a fractured neck of femur. How would you explain
Nifedipine how to take the tablet?
EXPLANATION: A. Take it on a full stomach to minimise gastric irritation
This lady has Raynaud's disease. and avoid lying down for 30 minutes afterwards
B. Dissolve tablet in water and take just before breakfast +
Please see Q-67 for Raynaud’s sit-upright for 30 minutes following
Q-175 C. Take during main evening meal + sit-upright for 2 hours
A 66-year-old female presents with pain at the base of her following
left thumb. She has no past medical history of note. On D. Take at least 30 minutes before breakfast with plenty of
examination there is diffuse tenderness and swelling of her water + sit-upright for 30 minutes following
left first carpometacarpal joint. What is the most likely E. Take at least 30 minutes after a main meal + sit-upright
diagnosis? for 30 minutes following
ANSWER: testing is not part of the workup for dermatomyositis.
Take at least 30 minutes before breakfast with plenty of water Screening for cardiac involvement may be undertaken in
+ sit-upright for 30 minutes following idiopathic inflammatory myopathies but is not the most
important next step. VTE prophylaxis is not indicated.
EXPLANATION:
Bisphosphonates can cause a variety of oesophageal Please see Q-55 for Dermatomyositis
problems
Q-180
Please see Q-25 for Bisphosphonates A 61-year-old man is noted to have thickened patches of skin
over his knuckles and extensor surfaces consistent with
Q-178 Gottron's papules. His creatinine kinase levels are also
A 24-year-old with a history of Crohn's disease is started on elevated. A diagnosis of dermatomyositis is suspected.
azathioprine. What is the mechanism of action of Which one of the following types of autoantibody is most
azathioprine? specific for this condition?

A. Inhibits purine synthesis A. Anti-scl-70 antibodies


B. Inhibits inosine monophosphate dehydrogenase B. Anti-Jo-1 antibodies
C. Mercaptopurine antagonist C. Anti-nuclear antibodies
D. Thiopurine methyltransferase inhibitor D. Anti-Mi-2 antibodies
E. Causes cross-linking in DNA E. Anti-centromere bodies

ANSWER: ANSWER:
Inhibits purine synthesis Anti-Mi-2 antibodies

EXPLANATION: EXPLANATION:
Please see Q-52 for Azathioprine Dermatomyositis antibodies: ANA most common, anti-Mi-2
most specific
Q-179
A 64-year-old female presents with difficulty brushing her Please see Q-107 for Dermatomyositis: Investigations and
hair and standing up after sitting in her chair. There is a Management
purple heliotropic rash above both eyes and a symmetrical
erythematous papular rash over both hands. Blood tests Q-181
identify a CK of 9543 and anti-nuclear antibody testing is A 76-year-old gentleman is seen in Rheumatology clinic for a
positive. You make a diagnosis of dermatomyositis and painful left first metatarsophalangeal joint. He has a past
initiate treatment with steroids. medical history of chronic kidney disease stage 4, heart
failure and poorly controlled type 2 diabetes.
What is the most important next step?
His GP suspects an acute flare-up of gout and would like to
A. Arrange CK to be re-checked by GP in 3 months commence treatment. What is the most appropriate
B. HLA testing medication to initiate?
C. Troponin I (high sensitivity)
D. Urgent malignancy screen A. Naproxen
E. VTE prophylaxis B. Prednisolone
C. Colchicine
ANSWER: D. Allopurinol
Urgent malignancy screen E. Ibuprofen

EXPLANATION: ANSWER:
In patients with a new diagnosis of dermatomyositis, urgent Colchicine
malignancy screen is needed
EXPLANATION:
Almost a quarter of individuals diagnosed with The best medication for this patient would be colchicine
dermatomyositis have an underlying malignancy (typically titrated to his renal function. BNF recommends reducing the
ovarian, breast and lung cancer), this prevalence increases in dose or increasing the dosage interval if eGFR 10-
the elderly. 50ml/minute/1.73m²; avoid if eGFR less than
10mL/minute/1.73m².
Serial CK measurement can be part of monitoring response
to treatment but is not the most important next step. HLA
NSAIDs would be contraindicated due to his chronic kidney Q-184
disease, prednisolone would worsen his already poorly What is the most common cardiac defect seen in Marfan's
controlled diabetes, and allopurinol would not be indicated syndrome
for an acute flare.
A. Mitral valve prolapse
Please see Q-20 for Gout: Management B. Coarctation of the aorta
C. Bicuspid aortic valve
Q-182 D. Dilation of the aortic sinuses
A 54-year-old man presents to the Emergency Department E. Ventricular septal defect
with a 2 day history of an swollen, painful left knee.
Aspirated joint fluid shows calcium pyrophosphate crystals. ANSWER:
Which of the following blood tests is most useful in revealing Dilation of the aortic sinuses
an underlying cause?
EXPLANATION:
A. Transferrin saturation Whilst mitral valve prolapse is seen in Marfan's syndrome,
B. ACTH dilation of the aortic sinuses is more common
C. ANA
D. Serum ferritin Please see Q-31 for Marfan’s Syndrome
E. LDH
Q-185
ANSWER: A 54-year-old farm worker presents for review. She has
Transferrin saturation recently been diagnosed with osteoarthritis of the hand but
has no other past medical history of note. Despite regular
EXPLANATION: paracetamol she is still experiencing considerable pain,
This is a typical presentation of pseudogout. An elevated especially around the base of both thumbs. What is the most
transferrin saturation may indicate haemochromatosis, a suitable next management step?
recognised cause of pseudogout
A. Add oral diclofenac + lansoprazole
A high ferritin level is also seen in haemochromatosis but can B. Switch paracetamol for co-codamol 8/500
be raised in a variety of infective and inflammatory C. Add topical ibuprofen
processes, including pseudogout, as part of an acute phase D. Add oral ibuprofen
response E. Add oral glucosamine

Please see Q-35 for Pseudogout ANSWER:


Add topical ibuprofen
Q-183
Which one of the following antibodies is most specific for EXPLANATION:
systemic lupus erythematous? Osteoarthritis - paracetamol + topical NSAIDs (if knee/hand)
first-line
A. Anti-neutrophil cytoplasmic antibodies
B. Anti-nuclear antibodies The 2008 NICE guidelines suggest the use of paracetamol
C. Anti-Sm antibodies and topical NSAIDs first-line
D. Anti-RNP antibodies
E. Anti-cardiolipin antibodies Please see Q-116 for Osteoarthritis: Management

ANSWER: Q-186
Anti-Sm antibodies A 67-year-old woman who is taking long-term prednisolone
for polymyalgia rheumatica presents with progressive pain in
EXPLANATION: her right hip joint. A diagnosis of avascular necrosis is
SLE: ANA is 99% sensitive - anti-Sm & anti-dsDNA are 99% suspected. Which investigation is most likely to be
specific diagnostic?

Please see Q-12 for Systemic Lupus Erythematosus: Radionuclide bone scan
Investigations MRI
Plain x-ray
CT
DEXA scan
ANSWER: ANSWER:
MRI Colchicine

EXPLANATION: EXPLANATION:
In early avascular necrosis a radionuclide bone scan is less Colchicine is useful in patients with renal impairment who
sensitive than MRI and the findings may be nonspecific. MRI develop gout as NSAIDs are relatively contraindicated. The
is therefore the investigation of choice. BNF advises to reduce the dose by up to 50% if creatinine
clearance is less than 50 ml/min and to avoid if creatinine
Please see Q-19 for Avascular Necrosis of the Hip clearance is less than 10 ml/min.

Q-187 Co-codamol 30/500 may be used as an adjunct but would not


Which one of the following conditions is least associated provide relief as monotherapy.
with HLA-B27?
Prednisolone is an option but would adversely affect his
A. Reiter's syndrome diabetic control.
B. Psoriatic arthritis
C. Ankylosing spondylitis Please see Q-20 for Gout: Management
D. Crohn's disease
E. Sacroiliitis Q-189
A 20-year-old woman is reviewed in the rheumatology clinic.
ANSWER: She has been referred due to a three month history of
Crohn's disease arthralgia, lethargy, muscle pains and Raynaud's
phenomenon. On examination she is noted to have slightly
EXPLANATION: swollen hands but no significant synovitis. A number of
There is an indirect association between HLA-B27 and blood tests are ordered:
Crohn's as some patients may develop enteropathic arthritis,
but this is the least common association of the above Hb 12.9 g/dl
Platelets 282 * 109/l
SERONEGATIVE SPONDYLOARTHROPATHIES WBC 6.2 * 109/l
Common features
• associated with HLA-B27 Rheumatoid factor Negative
• rheumatoid factor negative - hence 'seronegative' ANA Positive
• peripheral arthritis, usually asymmetrical Anti-dsDNA antibodies Negative
• sacroiliitis CRP 25 mg/l
• enthesopathy: e.g. Achilles tendonitis, plantar fasciitis ESR 39 mm/hr
• extra-articular manifestations: uveitis, pulmonary fibrosis Creatine kinase 675 ng/mL (50-200)
(upper zone), amyloidosis, aortic regurgitation
Given the likely diagnosis, which other antibodies are most
Spondyloarthropathies likely to be present?
• ankylosing spondylitis
• psoriatic arthritis A. Anti-Scl-70
• Reiter's syndrome (including reactive arthritis) B. Anti-centromere
• enteropathic arthritis (associated with IBD) C. Anti-Jo
D. Anti-RO
Q-188 E. Anti-RNP
A 64-year-old man with chronic kidney disease stage 3
secondary to type 2 diabetes mellitus presents with pain and ANSWER:
swelling at the right first metatarsophalangeal joint. On Anti-RNP
examination the joint is hot, erythematous and tender to
EXPLANATION:
touch, although he can still flex the big toe. What is the most
Anti-ribonuclear protein (anti-RNP) = mixed connective tissue
appropriate initial management?
disease
A. Colchicine
This patient has typical features of mixed connective tissue
B. Prednisolone
disease (e.g. arthralgia, myositis and Raynaud's). To confirm
C. Co-codamol 30/500
the diagnosis anti-RNP antibodies need to be
D. Allopurinol
E. Indomethacin
Please see Q-49 for Mixed Connective Tissue Disease
Q-190 tear in the inner wall of the aorta, the intima, leading to an
A 34-year-old female with a history of discoid lupus accumulation of blood within the wall of the blood vessel
erythematous is reviewed in clinic. The erythematous, scaly (false lumen as opposed to the true lumen of the aorta). This
rash on her face has not responded to topical steroid can occur due to high stress on the aortic wall (chronic
creams. What is the most appropriate next step in uncontrolled hypertension) or a weakening of the aortic
management? wall. This man does not have a history of hypertension but
he has signs of a genetically inherited condition strongly
A. UV light therapy associated with aortic dissection. He also has an uncle who
B. Oral hydroxychloroquine died young of a heart complication which could be an aortic
C. Topical dapsone dissection due to Marfan syndrome. This condition is known
D. Oral prednisolone as Marfan syndrome and is inherited in an autosomal
E. Topical hydroxychloroquine dominant pattern.

ANSWER: 1: This occurs due to normal aging and is responsible for the
Oral hydroxychloroquine appearance of wrinkles and the decreased elasticity of the
skin in the elderly population.
EXPLANATION: 2: This is the mechanism via which vitamin C deficiency leads
Discoid lupus erythematous - topical steroids → oral to scurvy.
hydroxychloroquine 3: In Marfan syndrome, there is a defection of the
glycoprotein fibrillin which normally envelopes elastin and as
Please see Q-82 for Discoid Lupus Erythematosus a result individuals with the condition has a range of signs
symptoms such as joint hypermobility, chest deformities and
Q-191 long toes and finger amongst others.
A 27-year-old man was brought to the hospital after he 4: This is usually seen in a genetically inherited condition
complained of a sudden-onset chest pain while playing called Menkes disease. The condition is inherited in an X-
football. He said that this is the first time he is having such a linked recessive pattern and involves an accumulation of
pain. He remembers that one of his uncles had a similar pain copper in some body tissues.
when he was young and he died early due to a heart 5: This is characteristic of alpha-1-antitrypsin deficiency
problem. On examination, he has a pulse rate of 87 beats per whereby there is a deficiency of the enzyme alpha-1-
minute, a respiratory rate of 22 breaths per minute and antitrypsin which normally functions to inhibit elastase. The
blood pressure of 101/74 mmHg. The doctor also notices excessive degradation of collagen leads to pan-acinar
that his fingers are longer than normal and that his little emphysema and liver impairment.
finger and thumb overlapped each when the man was asked
to hold the opposite wrist. He has no significant past medical Please see Q-31 for Marfan’s Syndrome
and surgical history and is not currently taking any regular
medications. He denies any illicit drug use but smokes about
half a pack of cigarette a day since he was 17 years old.
Which of the following best explains disease mechanism in
this man?

A. Decrease in the production of collagen


B. Impaired hydroxylation of lysine and proline amino acids
on collagen
C. A defect of the glycoprotein structure which usually
wraps around elastin
D. Copper deficiency affecting the normal function of lysyl
oxidase enzyme
E. Overactive elastase enzyme leading to an excessive
breakdown of collagen

ANSWER:
A defect of the glycoprotein structure which usually wraps
around elastin

EXPLANATION:
This young man presented with the sign and symptoms of an
aortic dissection. Aortic dissection occurs when the there is a

You might also like